You are on page 1of 76

UPSC

PRELIMINARY EXAMINATION

(PAPER-II)

CSAT

PREVIOUS YEARS QUESTIONS

(YEAR 2011 TO 2020)


THEME Æ COMPREHENSION
COMPREHENSION

2020
Passage–2
Many opportunities to harness the highly skewed,
Directions for the following 7 (seven) items: Read
seasonal and spatial distribution of monsoon flows,
the following five passages and answer the items
which occur in a four-month period from June to
that follow. Your answers to these items should
September annually, have been lost. Since these
be based on the passages only.
few months account for most of the rainfall and
consequent freshwater availability, the need for
Passage–1 holding rainwater in reservoirs, for subsequently
releasing it for use over the year, is a necessity
Private investment in general is volatile. Foreign nobody can afford to overlook. Climate change
private investment is more volatile because the will continue to affect weather conditions and
available investment avenues are significantly create water shortages and excesses. While
greater (i.e., the entire world). Therefore, the millions suffer from droughts and floods, waters
responsibility of providing employment cannot be in the country’s many rivers flow unutilized, and
left to Foreign Direct Investment (FDI). The are discharged into the sea every year.
current FDI inflows are (volatile over time and
across sectors and regions, which is a necessary
consequence of their search for the highest returns. 2. With reference to the above passage,
The adverse consequences are unstable which of the following could be the most
employment and an (accentuation of income and rational and practical implications for
regional inequalities. A probable positive India?
consequence of foreign investment is the inflow
1. Inter-linking of rivers should be
of new technology and its subsequent diffusion.
undertaken.
However, the technology diffusion is not at all
certain because the existing state of physical and 2. A network of dams and canals should
human capital in India may prove inadequate for be built across the country for proper
the diffusion. distribution of water.
3. Farmers should be provided easy
loans for digging borewells.
1. With reference to the above passage, the
following assumptions have been made: 4. Usage of water for agriculture should
be regulated by law.
1. Relying on foreign investment in the
long run is not an economically sound 5. Distribution of river water among
policy. regions should be regulated by the
Union Government.
2. Policies must be undertaken to reduce
volatility in foreign private Select the correct answer using the code
investment. given below.
3. Policies must be undertaken to (a) 1 and 2
strengthen domestic private (b) 2, 4 and 5
investment.
(c) 1, 3 and 4
4. Public investment should be given
(d) 2, 3 and 5
priority over private investment.
5. Substantial public investment in
education and health should be Passage–3
undertaken. People will invest in education whenever they are
Which of the above assumptions is/are granted the economic freedom to fully enjoy its
valid? benefits. Again, this is for the obvious reason that
the return on education increases as the level of
(a) 1, 2 and 4
economic freedom rises. When people, thanks to
(b) 1, 3 and 5 lower tax rates, are allowed to retain most of the
(c) 2, 4 and 5 higher income that they gain from each incremental
level of education, it makes eminent sense to invest
(d) 3 only
in education. On the other hand, when the

2
COMPREHENSION

government decides to tax the higher income of (c) Water shortage in our cities is a
educated individuals at even higher rates, it makes perennial problem that cannot be
very little sense to invest in educating oneself solved.
further. The same incentives apply to parents who (d) In view of the water crisis in our
decide on whether to invest in their children’s cities, there is an urgent need to limit
education. the population of cities by adopting
an upper limit of population size.
3. With reference to the above passage, the
following assumptions have been made: 5. With reference to the above passage, the
1. Lower tax rates in a country following assumptions have been made:
invariably translate into greater 1. Rich cities only can ensure sustainable
investments in higher education. delivery of water.
2. Investment in the education of 2. Sustainable delivery of water in cities
children ensures their economic means much more than supplying
freedom. water to households.
3. Economic freedom has a positive Which of the above assumptions is/are
impact on building up human capital. valid?
Which of the above assumptions is/are (a) 1 only
valid?
(b) 2 only
(a) 1 only
(c) Both 1 and 2
(b) 2 only
(d) Neither 1 nor 2
(c) 3 only
(d) 1, 2 and 3
Passage–5
In India, agriculture still engages about half of its
Passage–4 workforce, and about 85 per cent of its farms are
Our urban bodies cannot possibly ensure small and marginal. Compared to China and
sustainable delivery of water in our cities unless Vietnam, which have experienced fast structural
financing mechanisms are put in place. Water and rural transformation, India’s story is of slow
delivery requires heavy investment in collecting transformation. As a result, poverty reduction in
it from a natural source, treating it to make it India was at a much slower pace during 1988-2014,
potable, and laying a distribution network of pipes compared to China and Vietnam. India’s poverty
for delivery to the users. It also requires reduction was slow during 1988-2005, but during
investments in sewerage infrastructure and sewage 2005-2012, it accelerated dramatically—almost
treatment plants so that the sewers can carry the three times faster than during the earlier period.
wastewater to these plants to ensure that no What did India do during this period? Research
untreated sewage is discharged back into natural reveals that the relative price scenario changed
water bodies. If our cities were rich enough to significantly (by more than 50%) in favour of
meet the entire cost, water could be delivered free. agriculture in the wake of rising global prices. This
They are not. boosted private investments in agriculture by
more than 50%. As a result, agri-GDP growth
touched 41% during 2007-2012 as against 2.4%
4. What is the most logical and crucial during 2002-2007. The net surplus of agri-trade
message conveyed by the passage? touched $25 billion in 2013-2014; real farm wages
(a) Urban local bodies must recover costs rose by 7% per annum. All this led to
through user charges. unprecedented fall in poverty.
(b) Urban local bodies are not efficient
enough to meet the water 6. With reference to the above passage, the
requirements of our cities. following assumptions have been made:

3
COMPREHENSION

1. Structural and rural transformation is A right is a substantive right only if it works in all
impossible when farms are mainly situations, and for everyone. A right to free
small and marginal. expression for an individual about her exploitation,
2. A good price incentive can trigger for instance, is meaningless without actual
investments in agriculture. availability of security that guarantees that private
force cannot be used to thwart this right. The role
3. India needs to build value chains for of the State, therefore, is not just to abstain from
high-value agri-products like preventing rightful free expression, but also to
livestock and horticulture. actively ensure that private parties are not able to
4. Higher global prices of agricultural block it.
commodities are essential for India’s
poverty reduction.
9. On the basis of the above passage, the
Which of the above assumptions are valid? following assumptions have been made:
(a) 1 and 3 1. State should have some institutions
(b) 2 and 4 to ensure its appropriate role in a
digital society.
(c) 2 and 3
2. State should ensure that private
(d) 3 and 4
parties do not violate the citizens’
right to privacy.
7. Which one of the following statements best 3. Digital economy is not compatible
reflects the critical message of the passage? with the idea of not violating the
(a) India should create large-scale off- citizens’ privacy.
farm rural employment to reduce Which of the above assumptions is/are
poverty in the near future. valid?
(b) India should create a large number (a) 1 and 2
of farmer producer companies.
(b) 3 only
(c) Private investment in agriculture
(c) 1 and 3
should be given priority over public
investment. (d) 2 only
(d) Inclusive agricultural growth is key
to reduce poverty in the near future. Passage–3
One of the biggest ironies around water is that it
8. Which one of the following is best implied comes from rivers and other wetlands. Yet it is
in the passage? seen as divorced from them. While water is used
as a resource, public policy does not always grasp
(a) Strong measures need to be taken to
that it is a part of the natural ecosystem. Efforts at
reduce the rural to urban migration
engineering water systems are thus efforts at
of labour.
augmenting water supply rather than
(b) The working condition in strengthening the capacities of ecological systems.
construction and unregistered
manufacturing needs to be improved.
10. Which one of the following is the most
(c) Service sector has been reducing the
logical and rational inference that can be
problem of unemployment.
made from the above passage?
(d) Increased social sector spending is
(a) Rivers and other wetlands should be
imperative for large-scale job
protected under Ramsar Convention.
creation.
(b) Engineering water systems should be
modernized and further augmented.
Passage–2
(c) Wetlands need to be reinforced as
In India, the current focus on the right to privacy more than just open sources of water.
is based on some new realities of the digital age.

4
COMPREHENSION

(d) Water supply should not be free of various aspects, including socio-economic impacts,
cost so as to prevent its misuse or so that the potential of the technology can be
overuse. harnessed while minimizing negative impacts.
Given the importance of biotechnology in
developing varieties that can help in climate
Passage–4 change mitigation and adaptation, not using
Asset allocation is the most important investment biotechnology as a part of the climate change action
decision we will ever make and sadly, most of us plan cannot be an option. Domestic regulation of
do not give that decision, the importance it biotechnology cannot be viewed in isolation of
deserves. We are adamant about seeking trade policy and obligations under various
predictability with our future. We tend to think international treaties and conventions.
of investing in risky assets as extremely volatile
and value eroding. We also dislike fluctuating
returns and the loss of control of investment. We 12. With reference to the above passage, the
think our money is best left idle, unproductive but following assumptions have been made:
safe. There is no asset that is risk-free. We could 1. Biotechnology regulation is an
lose our jobs, our homes can lose value, our banks evolving process.
can go bankrupt, our bonds can default, the 2. Participation of people is needed in
government can collapse and companies we chose policy decisions regarding
fondly may cease to exist. But we cannot live life biotechnology regulation.
assuming that all these extreme events are waiting
to happen, and all at the same time. All these 3. Biotechnology regulation should take
extreme forms of risks we know will not manifest into account socio-economic aspects
at the same time. in decision-making.
4. Wider involvement of political
executive in biotechnology regulation
11. Which one of the following statements best improves its effectiveness in dealing
implies the suggestion given by the author with the country’s trade policies and
of the passage? international obligations.
(a) Distribute your wealth across Which of the above assumptions are valid?
different kinds of assets so that your
risks would be minimized. (a) 1, 2 and 4 only
(b) Risk-taking behaviour should be a (b) 1 and 3 only
necessary component of your (c) 2, 3 and 4 only
personality if you want to generate
(d) 1, 2, 3 and 4
wealth.
(c) While making investments, find a
trustworthy asset management 13. Which one of the following statements best
organization which would manage implies the crux of the passage?
your wealth for you. (a) Precautionary principle is not given
(d) You should know that investing your importance in current debate on
money is a risky business. developing GM crops.
(b) Biotechnology is not currently used
in climate change mitigation and
Passage–5
adaptation mechanisms.
Although most of the Genetically Modified (GM)
(c) Biotechnology’s role is not confined
crops cultivated now are genetically engineered
to the current priorities of developing
for a single trait, in future, crops genetically
GM crops.
engineered for more than one trait will be the
norm. Thus, biotechnology’s role in agriculture (d) The negative impacts of
and the regulation of the same cannot be biotechnology are not properly
understood solely in the context of the current understood.
generation of GM crops. Instead, there is a need
to take a comprehensive look, taking into account

5
COMPREHENSION

Directions for the following 6 (six) items: Read


the following five passages and answer the items 15. Which one of the following is the most
that follow. Your answers to these items should logical and rational inference that can be
be based on the passages only. made from the above passage?
(a) India’s commitment to reduce
Passage–1 emissions by 33% is unlikely to be
Bank credit to the industrial sector has started achieved.
shrinking. Its decline has been a serious concern (b) India should import gas rather than
as credit growth is essential to revive investment. invest in renewable resources.
The problem’s origins lie in the incomplete reforms (c) Getting renewable resources to
of the last 25 years. An institutional change that market too soon may be costly.
should have followed the 1991 reforms should
have been setting up of a resolution corporation (d) India should put in more efforts in
for banks. In a market economy with booms and the exploration of natural gas.
busts, banks should be allowed to be set up and
to fail. Today, we cannot shut down banks because
16. With reference to the above passage, the
there is no proper system to shut them down. Weak
following assumptions have been made:
loss-making banks continue to need more capital.
1. Governments often provide
inefficient and costly subsidies for
14. Which one of the following is the most technologies that may not be ready
logical and rational inference that can be in the near future.
made from the above passage?
2. India’s commitment of reducing
(a) Indian banking system is not able to emissions by 33% by 2030 shall be on
help the country in its economic the basis of gas-based economy.
growth.
Which of the above assumptions is/are
(b) Economic reforms that started in 1991 valid?
have not helped in improving the
(a) 1 only
economy to expected levels.
(b) 2 only
(c) India lacks the institutional
mechanism to deal with the failure of (c) Both 1 and 2
banks. (d) Neither 1 nor 2
(d) Encouraging the foreign investments
in our industrial sector is a good
alternative to this sector’s Passage–3
dependence on banks for credit. Genome editing is different from genome
modification. Genome editing typically involves
finding the part of a plant genome that could be
Passage–2 changed to render it less vulnerable to disease, or
India has tremendous potential for solar energy. resistant to certain herbicides, or to increase
We all realize that we have to stop burning fossil yields. Researchers use ‘molecular scissors’ to
fuels to meet our energy needs. dissect the genome and repair it, which is a process
that occurs naturally when plants are under attack
But certain renewable resources are still going
from diseases and can throw up new mutations
through their cost curves and learning curves to
that enable the plant to survive future attacks. This
get the required amount of output. The Indian
evolutionary process can effectively be speeded
Government has strongly committed to its targets
up now that it is possible to examine plant
of reducing emissions by 33 per cent by 2030, and
genomes in detail in laboratories, and create
towards this it has initiated a strong push towards
mechanisms through which the relevant genes can
a gas-based economy and has also invested heavily
be altered very precisely.
in renewable energy} However, business houses
are wary of investing too heavily in renewable
energy at a time when the technology is not yet 17. With reference to the above passage, the
ready. following assumptions have been made:

6
COMPREHENSION

1. Genome editing does not require the that “all persons are equally entitled to freedom
transfer of genes from one plant to of conscience and the right freely to profess,
another. practise and propagate religion”. What people fail
2. Through genome editing, the chosen to notice is that this proclamation is prefixed with
genes can be altered precisely in a the words “subject to public order, morality, health
manner akin to the natural process and to the other provisions of this Part”, which
that helps plants to adapt to the set conditions precedent for the legal protection
environmental factors. of religious practices of any community. The
closing words of this prefatory rider in Article 25
Which of the above assumptions is/are virtually constitute a subordination clause placing
valid? other fundamental rights mentioned in Part III
(a) 1 only over and above the right to religious freedom.
Among those other fundamental rights is the right
(b) 2 only
to equality before law and equal protection of
(c) Both 1 and 2 laws—assured at the outset and elaborated in later
(d) Neither 1 nor 2 articles to mean, inter alia, that the State shall not
deny equal protection of laws to any person or
group to persons on the basis of religion alone.
Passage–4
Many people understand the connection between 19. What is the most logical inference from the
solid waste management and health in terms of above passage?
the consequences of unattended heaps of dry
garbage which become home for flies and other (a) State shall not interfere with the
vermin. However, there is another aspect that is religious affairs of the citizens.
not well-understood, that is, what happens when (b) Religious freedom under the
unscientific solid waste management combines Constitution is open to State
with poor drainage and dumping of untreated intervention.
sewage into drains which are meant to carry storm
(c) Religious freedom of the citizens is
water during rains. The result is choked drains
not covered under fundamental
which are full of stagnant water breeding
rights.
mosquitoes, resulting in the spread of water-borne
diseases. (d) Religious practices of any community
are immune to State laws.
18. In the context of India, which one of the
following statements best reflects the
critical message of the passage? Directions for the following 6 (six) items: Read
(a) In India, the drainage networks are the following five passages and answer the items
not separate for sewerage and storm that follow. Your answers to these items should
water. be based on the passages only.
(b) Urban local bodies do not have
enough resources and legislative Passage–1
authority to deal with the problems
of waste management. Spanish ships in the late 16th century first brought
the potato tuber from South America to Europe
(c) Solid waste management should be whereby in the early 19th century, it had become
integrated with the maintenance of a reliable backup to cereal crops, particularly in
drainage and sewerage networks. the cold, rain-soaked soils of Ireland. The Irish
(d) Bad management of solid waste and were soon almost wholly dependent on the potato
sewerage systems by our as their staple food. And they were planting
municipalities is the reason for primarily one prodigious variety, the ‘Lumper’
drinking water shortages in our cities. potato, whose genetic frailty would be cruelly
exposed by the fungus ‘Phytophthora infestans’.
In 1845, spores of the deadly fungus began
Passage–5 spreading across the country, destroying nearly
In Part III of the Constitution, which assures people all the Lumpers in its path. The resulting famine
certain fundamental rights, Article 25 proclaims killed or displaced millions.

7
COMPREHENSION

5. Authorities should ensure the


20. Which one of the following statements best vaccination as prescribed.
reflects the critical message of the passage? Select the correct answer using the code
(a) For introducing any foreign plant into given below.
a country, the soil and climate (a) 1, 2, 3 and 4
conditions of that country should be (b) 2, 3, 4 and 5
suitable.
(c) 1 only
(b) As a staple food of a country, tuber
crops like potato cannot replace (d) 3 and 5 only
cereal crops.
(c) Some of the fungal infections of plants Passage–3
cannot be prevented or stopped from
The pulse variety ‘Pusa Arhar 16’ has the potential
spreading across large areas.
to be grown in the paddy-growing regions of
(d) Relying on a homogeneous food Punjab, Haryana and Uttar Pradesh and eventually
source is not desirable. in all of India. Its yield (about 2000 kg/hectare)
will be significantly greater than those of the
existing varieties and because its size will be
Passage–2 uniform, it will be amenable to mechanical
India is at once among the fastest growing global harvesting, an attractive feature for farmers in
economies and home to the largest number of northern India who currently use this technology
malnourished children in the world. There are for paddy. Most important, Arhar straw, unlike
regions where malnutrition is not the exception paddy straw, is green and can be ploughed back
but the norm. And across the country, malnutrition into the soil. In paddy straw, the problem is the
is the cause of death for roughly half the 1.3 million high silica content, which does not allow for easy
children who die before their fifth birthday each decomposition. In the case of Arhar, the farmer,
year. Even those children who survive suffer even after combine harvesting, just needs to run a
permanently from the damage that has already rotovator to cut the leftover straw into pieces,
been done to their bodies and minds from not which can be ploughed back and will decompose
getting enough of the right foods and nutrients. very fast. All this is difficult with leftover paddy
Around 44 million children under 5 are stunted. stalks that cannot be easily salvaged or ploughed
That makes it harder for them to learn in school back. Farmers, therefore, choose the easiest option
and subsequently earn a living as adults. Their of simply burning it.
lifetime earnings potential is almost a quarter less
than that of their healthy peers.
22. Which of the following are the most
rational inferences that can be made from
21. With reference to the above passage, the passage?
which of the following is/are the most 1. Farmers’ income will be higher with
rational and practical implication / pulse cultivation than with paddy
implications? cultivation.
1. India’s Public Distribution System 2. Pulse cultivation causes less pollution
should be monitored by the Union as compared to paddy cultivation.
Government.
3. Pulse straw can be used to improve
2. Girls should be encouraged to delay soil quality.
marriage and first pregnancy.
4. In the context of northern Indian
3. Mothers should be encouraged to agriculture, paddy straw has no
breastfeed their children immediately usefulness.
after birth.
5. Mechanized agriculture is the main
4. The supply of safe drinking water and cause for stubble burning.
proper sanitation facilities to all
should be ensured. Select the correct answer using the code
given below.
(a) 2, 3 and 5

8
COMPREHENSION
(b) 1, 4 and 5 prospects. These problems must be resolved if
(c) 2 and 3 only economic growth has to be sustained and
transformed into meaningful development.
(d) 1 and 4 only
Passage–4
24. Which of the following is/are the most
In India, authorities always look to store the rational and logical inference / inferences
maximum amount of water in reservoirs during that can be made from the passage?
the monsoon season, which is then used for
irrigation and generation of electricity during the 1. It is essential to rethink and redefine
summer months. It is an internationally accepted the economic role of the State in the
practice that the water level of a reservoir should quest for development.
be kept below a certain level before the onset of 2. India has not made effective
monsoon season. This is so that when monsoon implementation of its policies in social
rains come, there is space to store the excess sectors nor made sufficient
rainwater and also so that water can be released investments in them.
in a regulated manner. But the authorities store
Select the correct answer using the code
the maximum amount of water in reservoirs even
given below.
before the close of the monsoon, only to ensure
greater electricity generation and irrigation (a) 1 only
(b) 2 only
23. With reference to the above passage, the (c) Both 1 and 2
following assumptions have been made: (d) Neither 1 nor 2
1. High risks involved in holding
maximum water in reservoirs are due
to our over-dependence on 25. With reference to the above passage, the
hydropower projects. following assumptions have been made:

2. Storage capacity of dams should not 1. India’s economy needs to be greatly


be fully used before or during integrated with global economy so as
monsoon season. to create large number of jobs and to
sustain its growth momentum.
3. Role of dams in flood control is
underestimated in India. 2. Economic liberalization would cause
large economic growth which would
Which of the above assumptions is/are reduce poverty and create sufficient
valid? employment in the long run.
(a) 1 and 2 only Which of the above assumptions is/are
(b) 2 only valid?
(c) 3 only (a) 1 only
(d) 1, 2 and 3 (b) 2 only
(c) Both 1 and 2
Passage–5 (d) Neither 1 nor 2
Economic liberalization in India was shaped largely
by the economic problems of the government than
by the economic priorities of the people or by the 2019
long-term development objectives. Thus, there
were limitations in conception and design which
have been subsequently validated by experience. Directions for the following 7 (seven) items:
Jobless growth, persistent poverty and rising Read the following six passages and answer the
inequality have mounted as problems since items that follow each passage. Your answers to
economic liberalization began. And all these years these items should be based on the passages only.
later, four quiet crises confront the economy;
agriculture, infrastructure, industrialization and
education as constraints on the country’s future Passage -1

9
COMPREHENSION

What stands in the way of the widespread and Passage – 2


careful adoption of ‘Genetic Modification (GM)’ Most invasive species are neither terribly success-
technology is an ‘Intellectual Property Rights’ ful nor very harmful. Britain’s invasive plants are
regime that seeks to create private monopolies for not widespread, not spreading especially quickly,
such technologies. If GM technology is largely and often less of a nuisance than vigorous natives
corporate driven, it seeks to maximize profits and such as bracken. The arrival of new species almost
that too in the short run. That is why corpo- always increases biological diversity in a region;
rations make major investments for herbi- in many cases, a flood of newcomers drives no
cide-tolerant and pest-resistant crops. Such prop- native species to extinction. One reason is that in-
erties have only a short window, as soon enough, vaders tend to colonise disturbed habitats like
pests and weeds will evolve to overcome such polluted lakes and post-industrial wasteland,
resistance. This suits the corporations. The Na- where little else lives. They are nature’s oppor-
tional Farmers Commission pointed out that pri- tunists.
ority must be given in genetic modification to the
incorporation of genes that can help impart resis-
tance to drought, salinity and other stresses. 3. Which one of the following is the most
logical and rational inference that can be
made from the above passage?
1. Which one of the following is the most
logical, rational and crucial message con- (a) Invasive species should be used to
veyed by the above passage? rehabilitate desert areas and waste-
lands of a country.
(a) Public research institutions should
take the lead in GM technology and (b) Laws against the introduction of for-
prioritise the technology agenda. eign plants are unnecessary.
(b) Developing countries should raise (c) Sometimes, the campaigns against
this issue in WTO and ensure the foreign plants are pointless.
abolition of Intellectual Property (d) Foreign plants should be used to in-
Rights. crease the biodiversity of a country.
(c) Private corporations should not be
allowed to do agribusiness in India,
Passage – 3
particularly the seed business.
Diarrhoeal deaths among Indian children are
(d) Present Indian circumstances do not
mostly due to food and water contamination. Use
favour the cultivation of genetically
of contaminated groundwater and unsafe chemi-
modified crops.
cals in agriculture, poor hygiene in storage and
handling of food items to food cooked and dis-
2. On the basis of the above passage, the fol- tributed in unhygienic surroundings; there are
lowing assumptions have been made: myriad factors that need regulation and monitor-
ing. People need to have awareness of adultera-
1. The issue of effects of natural calami- tion and ways of complaining to the relevant au-
ties on agriculture is not given due thorities. Surveillance of food-borne diseases in-
consideration by GM technology com- volves a number of government agencies and en-
panies. tails a good training of inspection staff. Consider-
2. In the long run, GM technology will ing the proportion of the urban population that
not be able to solve agricultural prob- depends on street food for its daily meals, invest-
lems arising due to global warming. ing in training and education of street vendors is
Which of the above assumptions is/are of great significance.
valid?
(a) 1 only 4. On the basis of the above passage, the fol-
(b) 2 only lowing assumptions have been made:

(c) Both 1 and 2 1. Food safety is a complex issue that


calls for a multipronged solution.
(d) Neither 1 nor 2
2. Great investments need to be made
in developing the manpower for sur-

10
COMPREHENSION

veillance and training. based on technological progress.


3. India needs to make sufficient legis- 2. much of modern Indian economy
lation for governing food processing does not nurture sufficient symbiotic
industry. relationship with labour-intensive,
Which of the above assumptions is/are natural resource-based livelihoods.
valid? 3. service sector in India is not very
(a) 1 and 2 only labour-intensive.

(b) 3 only 4. literate rural population is not will-


ing to enter organised sector.
(c) 1 and 3 only
Which of the statements given above are
(d) 1, 2 and 3 correct ?
(a) 1 and 2 only
Passage – 4 (b) 3 and 4 only
The interests of working and poor people have (c) 1, 2 and 3 only
historically been neglected in the planning of our
cities. Our cities are increasingly intolerant, un- (d) 1, 2, 3 and 4
safe and unlivable places for large numbers of citi-
zens and yet we continue to plan via the old ways Passage - 6
— the static Development Plan — that draws ex-
clusively from technical expertise, distanced from India has banking correspondents, who help bring
people’s live experiences and needs, and actively people in the hinterland into the banking fold. For
excluding large number of people, places, activi- them to succeed, banks cannot crimp on costs.
ties and practices that are an integral part of the They also cannot afford to ignore investing in fi-
city. nancial education and literacy. Banking correspon-
dents are way too small to be viewed as a sys-
temic risk. Yet India’s banking regulator has re-
5. The passage seems to argue stricted them to serving only one bank, perhaps
(a) against the monopoly of builders and to prevent arbitrage. Efforts at banking outreach
the interests of elite groups. may succeed only if there are better incentives at
work for such last-mile workers and also those
(b) against the need for global and smart providers who ensure not just basic bank accounts
cities. but also products such as accident and life insur-
(c) in favour of planning cities mainly for ance and micro pension schemes.
working class and poor people.
(d) in favour of participation of peoples’ 7. Which one of the following is the most
groups in city planning. logical, rational and crucial inference that
can be derived from the above passage?
Passage – 5 (a) Efforts to bring people in India’s hin-
terland into the banking system are
A vast majority of Indians are poor, with barely
not successful.
10 percent employed in the organised sector. We
are being convinced that vigorous economic (b) For meaningful financial inclusion,
growth is generating substantial employment. But India’s banking system needs more
this is not so. When our economy was growing at number of banking correspondents
3 percent per year, employment in the organised and other such last-mile workers.
sector was growing at 2 percent per year. As the (c) Meaningful financial inclusion in In-
economy began to grow at 7 - 8 percent per year, dia requires that banking correspon-
the rate of growth of employment in the organised dents have diverse skills.
sector actually declined to 1 percent per year.
(d) Better banking outreach would be
impossible unless each banking cor-
6. The above passage seems to imply that respondent is allowed to serve a num-
ber of banks.
1. most of modern economic growth is

11
COMPREHENSION

9. Which one of the following is the most


Directions for the following 8 (eight) items: logical and rational corollary to the above
passage?
Read the following six passages and answer the
items that follow each passage. Your answers to (a) Globalization is not in the interests
these items should be based on the passages only. of India as it undermines its socio-
economic structures.
(b) India should be careful to protect its
Passage – 1 digital sovereignty in global trade
Low-end IoT (Internet of Things) devices are talks.
cheap commodity items : addressing security (c) India should charge monopoly rents
would add to the cost. This class of items is pro- from multinational companies in ex-
liferating with new applications; many home ap- change for Big Data.
pliances, thermostats, security and monitoring
devices and personal convenience devices are part (d) The loss of Big Data from India is
of the IoT. So are fitness trackers, certain medical proportional to the degree/value of
implants and computer-like devices in automo- its foreign trade.
biles. The IoT is expected to expand exponentially
— but new security challenges are daunting. 10. Which of the following is most definitively
implied by the above passage?
8. Which one of the following statements is (a) Big Data is the key resource in the
the most logical and rational inference that digital space.
can be made from the above passage? (b) Big economies create Big Data.
(a) Development of enabling technolo- (c) Access to Big Data is the prerogative
gies in India can be a big boost to its of developed countries.
manufacturing sector.
(d) Access to and possession of Big Data
(b) India is not yet fully ready to adopt is a characteristic of developed coun-
IoT in view of the imminent security tries.
challenges.
(c) Life becomes more comfortable with
the development of cheap low-end
IoT devices. Passage – 3
(d) As we go digital, we must recognise The rural poor across the world, including India,
the huge threat to Internet security have contributed little to human-induced climate
from some IoT devices. change, yet they are on the frontline in coping with
its effects. Farmers can no longer rely on histori-
cal averages for rainfall and temperature, and the
Passage – 2 more frequent and extreme weather events, such
With the digital phenomenon restructuring most as droughts and floods, can spell disaster. And
social sectors, it is little surprise that global trade there are new threats, such as sea level rise and
negotiations are now eyeing the digital area in an the impact of melting glaciers on water supply.
attempt to pre-emptively colonise it. Big Data is How significant are small farms ? As many as two
freely collected or mined from developing coun- billion people worldwide depend on them for their
tries, and converted into digital intelligence in food and livelihood. Small-holder farmers in In-
developed countries. This intelligence begins to dia produce 41 percent of the country’s food
control different sectors and extract monopoly grains, and other food items that contribute to
rents. A large foreign company providing cab ser- local and national food security.
vice, for instance, is not a network of cars and
drivers; it is digital intelligence about commuting,
public transport, roads, traffic, city events, per- 11. What is the most logical and rational cor-
sonal behavioural characteristics of commuters and ollary to the above passage?
drivers and so on. (a) Supporting small farmers is an impor-
tant part of any agenda regarding
environmentally sustainable develop-

12
COMPREHENSION

ment. (a) 1 and 2 only


(b) Poor countries have little role to play (b) 3 only
in the mitigation of global warming. (c) 1 and 3 only
(c) Due to a large number of farmer (d) 1, 2 and 3
households, India will not have food
security problem in the foreseeable
future. Passage – 5
(d) Only small-holder farmers in India Access to schooling for those coming of school age
can ensure food security. is close to universal, but access to quality exhibits
a sharp gradient with socio-economic status. Quo-
tas for the weaker sections in private schools is a
12. The above passage implies that provision introduced by the Right of Children to
1. There is a potential problem of food Free and Compulsory Education Act, 2009. The
insecurity in India. quotas have imposed a debate on issues of social
2. India will have to strengthen its di- integration and equity in education that private
saster management capabilities. actors had escaped by and large. The idea of egali-
tarian education system with equality of opportu-
Which of the above assumptions is/are nity as its primary goal appears to be outside the
valid? space that private school principals inhabit. There-
(a) 1 only fore, the imposition of the quotas has led to resis-
tance, sometimes justified.
(b) 2 only
(c) Both 1 and 2
14. With reference to the above passage, the
(d) Neither 1 nor 2
following assumptions have been made:
1. Making equality of opportunity a re-
Passage – 4 ality is the fundamental goal of the
A changing climate, and the eventual efforts of Indian education system.
governments (however reluctant) to deal with it, 2. The present Indian school system is
could have a big impact on investors’ returns. unable to provide egalitarian educa-
Companies that produce or use large amounts of tion.
fossil fuels will face higher taxes and regulatory
3. Abolition of private schools and es-
burdens. Some energy producers may find it im-
tablishment of more government
possible to exploit their known reserves, and be
schools is the only way to ensure
left with “stranded assets” — deposits of oil and
egalitarian education.
coal that have to be left in the ground. Other in-
dustries could be affected by the economic dam- Which of the above assumptions is/are
age caused by more extreme weather — storms, valid?
floods, heat waves and droughts. (a) 1 and 2 only
(b) 2 only
13. On the basis of the above passage, the fol- (c) 2 and 3 only
lowing assumptions have been made:
(d) 3 only
1. Governments and companies need to
be adequately prepared to face the
climate change. Passage – 6
2. Extreme weather events will reduce A majority of the TB infected in India are poor
the economic growth of governments and lack sufficient nutrition, suitable housing and
and companies in future. have little understanding of prevention. TB then
3. Ignoring climate change is a huge risk devastates families, makes the poor poorer, par-
for investors. ticularly affects women and children, and leads
to ostracisation and loss of employment. The truth
Which of the above assumptions is/are is that even if TB does not kill them, hunger and
valid? poverty will. Another truth is that deep-seated

13
COMPREHENSION

stigma, lack of counselling, expensive treatment (a) A better regulatory system will help
and lack of adequate support from providers and India achieve the size of economy
family, coupled with torturous side-effects appropriate to its population.
demotivate patients to continue treatment — with (b) In a competitive global economy, In-
disastrous health consequences. dia must use regulations strategically.
(c) Regulations in India do not favour its
15. Which one of the following is the most integration with today’s hyper com-
logical, rational and crucial message con- petitive global economy.
veyed by the above passage? (d) Job creation and economic growth
(a) TB is not a curable disease in Indian should be dominant considerations in
circumstances. developing India’s regulatory system.
(b) Curing TB requires more than diag-
nosis and medical treatment. 17. On the basis of the above passage, the
(c) Government’s surveillance mecha- following assumptions have been made :
nism is deficient; and poor people In today’s global economy,
have no access to treatment.
1. regulations are not effectively used
(d) India will be free from diseases like to protect local markets.
TB only when its poverty alleviation
programmes are effectively and suc- 2. social and environmental concerns are
cessfully implemented. generally ignored by the govern-
ments across the world while imple-
menting the regulations.
Directions for the following 7 (seven) items : Which of the above assumptions is/are
Read the following five passages and answer the items valid ?
that follow each passage. Your answers to these items (a) 1 only
should be based on the passages only.
(b) 2 only
Passage - 1
(c) Both 1 and 2
India’s economic footprint, given its population,
still remains small compared to the US, the Euro- (d) Neither 1 nor 2
pean Union or China. It has much to learn from
other economies, yet must implement solutions
Passage – 2
that fit its unique circumstances. India especially
needs an effective long-term regulatory system In a study, scientists compared the microbiomes
based on collaboration rather than the current top- of poorly nourished and well nourished infants
down approach. Regulations seek desirable out- and young children. Gut microbes were isolated
comes yet are repeatedly used as political tools to from faecal samples of malnourished and healthy
push one agenda or another. Often, regulations children. The microbiome was “immature” and less
fail to consider impacts on jobs and economic diverse in malnourished children compared to the
growth — or less restrictive alternatives. Regula- better developed “mature” microbiome found in
tions may be used to protect local markets at the healthy children of the same age. According to
expense of more widely shared prosperity in the some studies, the chemical composition of mother’s
future. Additionally, regulations inevitably result milk has shown the presence of a modified sugar
in numerous unintended consequences. In today’s (sialylated oligosaccharides). This is not utilized
hyper competitive global economy, regulations by the baby for its own nutrition. However, the
need to be viewed as “weapons” that seek cost- bacteria constituting the infant’s microbiome thrive
justified social and environmental benefits while on this sugar which serves as their food. Malnour-
improving the economic well-being of most citi- ished mothers have low levels of this sugar in their
zens. milk. Consequently, the microbiomes of their in-
fants fail to mature. That in turn, leads to mal-
nourished babies.
16. Which one of the following is the most
logical, rational and crucial inference that
can be derived from the above passage ? 18. Which one of the following is the most

14
COMPREHENSION

logical, rational and crucial inference that sedimentation in some areas.


can be derived from the above passage ? 3. Melting glaciers can reduce marine
(a) If malnourished condition in children biodiversity in some areas.
is caused by gut bacteria, it cannot Which of the above assumptions is/are
be treated. valid?
(b) The guts of malnourished babies (a) 1 and 2 only
should be inoculated with mature
microbiomes. (b) 3 only
(c) Babies of malnourished mothers (c) 2 and 3 only
should be fed with dairy milk forti- (d) 1, 2 and 3
fied with sialylated oligosaccharides
instead of mother’s milk.
Passage - 4
(d) Research on benign effects of gut bac-
teria on nutrition has policy implica- A research team examined a long-term owl roost.
tions. Owls prey on small mammals and the excreted
remains of those meals that accumulated over the
time, provide us an insight into the composition
19. On the basis of the above passage, the and structure of small mammals over the past mil-
following assumptions have been made : lennia. The research suggested that when the Earth
1. Processed probiotic foods are a solu- went through a period of rapid warming about
tion to treat the children suffering 13,000 years ago, the small mammal community
from malnutrition due to immature was stable and resilient. But, from the last quar-
gut bacteria composition. ter of the nineteenth century, human-made
changes to the environment had caused an enor-
2. The babies of malnourished mothers mous drop in biomass and energy flow. This dra-
generally tend to be malnourished. matic decline in energy flow means modern eco-
Which of the above assumptions is/are systems are not adapting as easily as they did in
valid? the past.
(a) 1 only
(b) 2 only 21. On the basis of the above passage, the
(c) Both 1 and 2 following assumptions have been made :

(d) Neither 1 nor 2 1. Global warming is a frequently oc-


curring natural phenomenon.
2. The impending global warming will
Passage - 3 not adversely affect small mammals.
Temperatures have risen nearly five times as rap- 3. Humans are responsible for the loss
idly on the Western Antarctic Peninsula than the of the Earth’s natural resilience.
global average over the past five decades. Re-
searchers have now found that melting glaciers Which of the above assumptions is/are
are causing a loss of species diversity among valid?
benthos in the coastal waters off the Antarctic (a) 1 and 2 only
Peninsula, impacting an entire seafloor ecosystem. (b) 3 only
They believe increased levels of suspended sedi-
ment in water to be the cause of the dwindling (c) 2 and 3 only
biodiversity in the coastal region. (d) 1, 2 and 3

20. On the basis of the above passage, the Passage – 5


following assumptions have been made :
Food varieties extinction is happening all over the
1. Regions of glaciers warm faster than world — and it is happening fast. For example, of
other regions due to global warming. the 7,000 apple varieties that were grown during
2. Global warming can lead to seafloor the nineteenth century, fewer than a hundred re-
main. In the Philippines, thousands of varieties of

15
COMPREHENSION

rice once thrived; now only up to a hundred are 24. On the basis of the above passage, the
grown there. In China, 90 percent of the wheat following assumptions have been made :
varieties cultivated just a century ago have disap- 1. Removal of economic discrimina-
peared. Farmers in the past painstakingly bred and tion leads to removal of social dis-
developed crops well suited to the peculiarities of crimination.
their local climate and environment. In the recent
past, our heavy dependence on a few high yield- 2. Democratic polity is the best way to
ing varieties and technology-driven production repair historical wrongs.
and distribution of food is causing the dwindling Which of the above assumptions is/are
of diversity in food crops. If some mutating crop valid?
disease or future climate change decimates the few
(a) 1 only
crop plants we have come to depend on to feed
our growing population, we might desperately (b) 2 only
need some of those varieties we have let go ex- (c) Both 1 and 2
tinct.
(d) Neither 1 nor 2

22. On the basis of the above passage, the


following assumptions have been made : Passage - 2

1. Humans have been the main reason Education plays a great transformatory role in life,
for the large scale extinction of plant particularly so in this rapidly changing and glo-
species. balizing world. Universities are the custodians of
the intellectual capital and promoters of culture
2. Consumption of food mainly from and specialized knowledge. Culture is an activity
locally cultivated crops ensures crop of thought, and receptiveness to beauty and hu-
diversity man feelings. A merely well informed man is only
3. The present style of production and a bore on God’s earth. What we should aim at is
distribution of food will finally lead producing men who possess both culture and ex-
to the problem of food scarcity in the pert knowledge. Their expert knowledge will give
near future. them a firm ground to start from and their cul-
ture will lead them as deep as philosophy and as
4. Our food security may depend on our
high as art. Together it will impart meaning to
ability to preserve the locally culti-
human existence.
vated varieties of crops.
Which of the above assumptions are valid?
25. On the basis of the above passage, the
(a) 1 and 3
following assumptions have been made :
(b) 2 and 4
1. A society without well educated
(c) 2 and 3 people cannot be transformed into a
(d) 1 and 4 modern society.
2. Without acquiring culture, a person’s
education is not complete.
23. What is the main idea that we can infer
from the passage ? Which of the above assumptions is/are
valid ?
(a) Untouchability in India has not been
taken seriously by political theorists. (a) 1 only
(b) Historical injustice is inevitable in any (b) 2 only
society and is always beyond repair. (c) Both 1 and 2
(c) Social discrimination and deprivation (d) Neither 1 nor 2
have their roots in bad economies.
(d) It is difficult, if not impossible, to re-
Passage – 3
pair every manifestation of historical
injustice. Soil, in which nearly all our food grows, is a liv-
ing resource that takes years to form. Yet it can
vanish in minutes. Each year 75 billion tonnes of

16
COMPREHENSION

fertile soil is lost to erosion. That is alarming — (d) 1, 2 and 3


and not just for food producers. Soil can trap huge
quantities of carbon dioxide in the form of organic
carbon and prevent it from escaping into the at- Passage - 5
mosphere. Climate change may actually benefit some plants
by lengthening growing seasons and increasing
carbon dioxide. Yet other effects of a warmer
26. On the basis of the above passage, the world, such as more pests, droughts, and flood-
following assumptions have been made : ing, will be less benign. How will the world adapt
1. Large scale soil erosion is a major rea- ? Researchers project that by 2050, suitable crop-
son for widespread food insecurity lands for four commodities — maize, potatoes, rice
in the world. and wheat — will shift, in some cases pushing
2. Soil erosion is mainly anthropogenic. farmers to plant new crops. Some farmlands may
benefit from warming, but others won’t. Climate
3. Sustainable management of soils alone does not dictate yields; political shifts, glo-
helps in combating climate change. bal demand, and agricultural practices will influ-
Which of the above assumptions is/are ence how farms fare in the future.
valid ?
(a) 1 and 2 only 28. Which one of the following is the most
(b) 3 only logical and rational inference that can be
made from the above passage ?
(c) 2 and 3 only
(a) Farmers who modernize their meth-
(d) 1, 2 and 3
ods and diversify their fields will be
in an advantageous position in future.
Passage - 4 (b) Climate change will adversely affect
Inequality is visible, even statistically measurable the crop diversity.
in many instances, but the economic power that (c) Shifting major crops to new croplands
drives it is invisible and not measurable. Like the will lead to a great increase in the
force of gravity, power is the organising principle total area under cultivation and thus
of inequality, be it of income, or wealth, gender, an increase in overall agricultural pro-
race, religion and region. Its effects are seen in a duction.
pervasive manner in all spheres, but the ways in
(d) Climate change is the most important
which economic power pulls and tilts visible eco-
factor affecting the agricultural
nomic variables remain invisibly obscure.
economy in the future.

27. On the basis of the above passage, the


Passage - 6
following assumptions have been made :
A bat’s wings may look like sheets of skin. But
1. Economic power is the only reason
underneath, a bat has the same five fingers as an
for the existence of inequality in a
orangutan or a human, as well as a wrist connected
society.
to the same cluster of wrist bones connected to
2. Inequality of different kinds, income, the same long bones of the arm. What can be more
wealth, etc. reinforces power. curious than that the hand of a man, formed for
3. Economic power can be analysed grasping, that of a mole for digging, the leg of the
more through its effects than by di- horse, the paddle of the porpoise, and the wing
rect empirical methods. of the bat, should all be constructed on the same
pattern ?
Which of the above assumptions is/are
valid ?
(a) 1 and 2 only 29. Which one of the following is the most
logical, scientific and rational inference
(b) 3 only that can be made from the above passage
(c) 1 and 3 only ?

17
COMPREHENSION

(a) Different species having similar struc- (b) 3 and 4


ture of hands is an example of (c) 1 and 3
biodiversity.
(d) 2 and 4
(b) Limbs being used by different spe-
cies for different kinds of work is an
example of biodiversity. 2018
(c) Man and the aforementioned animals
having similar structure of limbs is an
example of coincidence in evolution. Directions for the following 3 (three) items :
(d) Man and the aforementioned animals Read the following two passages and answer the
have a shared evolutionary history. items that follow. Your answers to these items
should be based on the passages only.

Passage - 7
Passage—1
Around 56 million years ago, the Atlantic Ocean
had not fully opened and animals, perhaps includ- The quest for cheap and plentiful meat has resulted
ing our primate ancestors, could walk from Asia in factory farms where more and more animals
to North America through Europe and across are squeezed into smaller lots in cruel and shocking
Greenland. Earth was warmer than it is today, conditions. Such practices have resulted in many
but as the Palaeocene epoch gave way to Eocene, of the world’s health pandemics such as the avian
it was about to get much warmer still -r-rapidly flu. Worldwide, livestock are increasingly raised
and radically. The cause was a massive geologi- in cruel, cramped conditions, where animals spend
cally sudden release of carbon. During this period their short lives under artificial light, pumped full
called Palaeocene - Eocene Thermal Maximum or of antibiotics and growth hormones, until the day
PETM, the carbon injected into the atmosphere was they are slaughtered. Meat production is water-
roughly the amount that would be injected today intensive. 15000 litres of water is needed for every
if humans burned all the Earth’s reserves of coal, kilogram of meat compared with 3400 litres for
oil and natural gas. The PETM lasted for about rice, 3300 litres for eggs and 255 litres for a
1,50,000 years, until the excess carbon was reab- kilogram of potatoes.
sorbed. It brought on drought, floods, insect
plagues and a few extinctions. Life on Earth sur- 1. What is the most rational and crucial
vived — indeed, it prospered — but it was drasti- message given by the passage?
cally different.
(a) Mass production of meat through
industrial farming is cheap and is
30. Based on the above passage, the follow- suitable for providing protein
ing assumptions have been made : nutrition to poor countries.
(b) Meat-producing industry violates the
1. Global warming has a bearing on the laws against cruelty to animals.
planet’s biological evolution. ^ (c) Mass production of meat through
2. Separation of land masses causes the industrial farming is undesirable and
release of huge quantities of carbon should be stopped immediately.
into the atmosphere. (d) Environmental cost of meat
3. Increased warming of Earth’s atmo- production is unsustainable when it
is produced through industrial
sphere can change the composition of
its flora and fauna. farming.

4. The present man-made global warm-


ing will finally lead to conditions simi- Passage—2
lar to those which happened 56 mil- A male tiger was removed from Pench Tiger
lion years ago. Reserve and was relocated in Panna National Park.
Which of the assumptions given above Later, this tiger trekked toward his home 250
are valid? miles away. The trek of this solitary tiger highlights
a crisis. Many wildlife reserves exist as islands of
(a) 1 and 2 fragile habitat in a vast sea of humanity, yet tigers

18
COMPREHENSION

can range over a hundred miles, seeking prey, like India, which would be one of the hardest hit
mates and territory. Nearly a third of India’s tigers by climate change, given its need to finance
live outside tiger reserves, a situation that is development. Most countries do indeed treat
dangerous for both human and animal. Prey and climate change as real threat and are striving to
tigers can only disperse if there are recognized address it in a more comprehensive and integrated
corridors of land between protected areas to allow manner with the limited resources at their disposal.
unmolested passage.
4. With reference to the above passage, the
2. Which of the following is the most following assumptions have been made :
rational and crucial message given by the 1. Climate change is not a challenge for
passage? developed countries.
(a) The conflict between man and 2. Climate change is a complex policy
wildlife cannot be resolved, no matter issue and also a development issue
what efforts we make. for many countries.
(b) Safe wildlife corridors between 3. Ways and means of finance must be
protected areas is an essential aspect found to enable developing countries
of conservation efforts. to enhance their adaptive capacity.
(c) India needs to declare more Which of the above assumptions is/are
protected areas and set up more tiger valid?
reserves. (a) 1 and 2 only
(d) India’s National Parks and Tiger (b) 3 only
Reserves need to be professionally (c) 2 and 3 only
managed. (d) 1, 2 and 3

3. With reference to the above passage, the Passage—2


following assumptions have been made :
Cooking with biomass and coal in India is now
1. The strategy of conservation of
recognized to cause major health problems, with
wildlife by relocating them from one
women and children in poor populations facing
protected area to another is not often
the greatest risk. There are more than 10 lakh
successful.
premature deaths each year from household air
2. India does not have suitable
pollution due to polluting cooking fuels with
legislation to save the tigers, and its
another 1-5 lakh due to their contribution to
conservation efforts have failed
general outdoor air pollution in the country.
which forced the tigers to live outside
Although the fraction of the Indian population
protected areas.
using clean cooking fuels, such as LPG, natural gas
Which of the above assumptions is/are and electricity, is slowly rising, the number using
valid? polluting solid fuels as their primary cooking fuel
(a) 1 only has remained static for nearly 30 years at about 70
(b) 2 only crore.
(c) Both 1 and 2
(d) Neither 1 nor 2
5. Which of the following is the most crucial
and logical inference that can be made
Directions for the following 8 (eight) items : from the above passage?
Read the following eight passages and answer the (a) Rural people are giving up the use of
items that follow. Your answers to these items polluting solid fuels due to their
should be based on the passages only. increasing awareness of health
hazards.
(b) Subsidizing the use of clean cooking
Passage—1
fuels will solve the problem of India’s
All actions to address climate change ultimately indoor air pollution.
involve costs. Funding is vital in order for (c) India should increase its import of
countries like India to design and implement natural gas and produce more
adaptation and mitigation plans and projects. The electricity.
problem is more severe for developing countries

19
COMPREHENSION

(d) Access to cooking gas can reduce Being a member of the WTO, India is bound by
premature deaths in poor the agreements that have been signed and ratified
households. by its members, including itself. According to
Article 6 of the Agriculture Agreement, providing
minimum support prices for agricultural products
Passage—3
is considered distorting and is subject to limits.
Scientific knowledge has its dangers, but so has The subsidy arising from ‘minimal supports’ cannot
every great thing. Over and beyond the dangers exceed 10 per cent of the value of agricultural
with which it threatens the present, it opens up as production for developing countries. PDS in India
nothing else can, the vision of a possible happy entails minimum support prices and public
world; a world without poverty, without war, stockholding of food grains. It is possible that, in
with little illness. Science, whatever unpleasant some years, the subsidy to producers will exceed
consequences it may have by the way, is in its very 10 per cent of the value of agricultural production.
nature a liberator.

8. What is the crucial message conveyed by


6. Which one of the following is the most the above passage?
important implication of the passage? (a) India should revise its PDS.
(a) A happy world is a dream of science. (b) India should not be a member of
(b) Science only can build a happy world, WTO.
but it is also the only major threat. (c) For India, food security collides with
(c) A happy world is not possible trade.
without science. (d) India provides food security to its
(d) A happy world is not at all possible poor.
with or without science.

Passage—6
Passage—4
India’s educational system is modelled on the mass
The Arctic’s vast reserves of fossil fuel, fish and education system that developed in the 19th
minerals are now accessible for a longer period in century in Europe and later spread around the
a year. But unlike Antarctica, which is protected world. The goal of the system is to condition
from exploitation by the Antarctic Treaty framed children as ‘good’ citizens and productive
during the Cold War and is not subject to territorial workers. This suited the industrial age that needed
claims by any country, there is no legal regime the constant supply of a compliant workforce with
protecting the Arctic from industrialization, a narrow set of capabilities. Our educational
especially at a time when the world craves for institutes resemble factories with bells, uniforms
more and more resources. The distinct possibility and batch-processing of learners, designed to get
of ice-free summer has prompted countries with learners to conform. But, from an economic point
Arctic coastline to scramble for great chunks of of view, the environment today is very different.
the melting ocean. It is a complex, volatile and globally interconnected
world.
7. Which one of the following is the most
important implication of the passage? 9. With reference to the above passage, the
(a) India can have territorial claims in the following assumptions have been made :
Arctic territory and free access to its 1. India continues to be a developing
resources. country essentially due to its faulty
(b) Melting of summer ice in the Arctic education system.
leads to changes in the geopolitics. 2. Today’s learners need to acquire
(c) The Arctic region will solve the new-age skill-sets.
world’s future problem of resource 3. A good number of Indians go to some
crunch. developed countries for education
(d) The Arctic region has more resources because the educational systems there
than Antarctica. are a perfect reflection of the societies
in which they function.
Passage—5 Which of the above assumptions is/are

20
COMPREHENSION

valid? in the world.


(a) 1 and 3 only (c) More and more genetically modified
(b) 2 only crops only can save the world from
(c) 2 and 3 only impending shortages of food.
(d) 1, 2 and 3 (d) Asia and North America will be
worst sufferers from climate change
and the consequent shortage of food.
Passage—7
The practice of dieting has become an epidemic;
Directions for the following 4 (four) items :
everyone is looking out for a way to attain that
perfect body. We are all different with respect to
our ethnicity, genetics, family history, gender, age, Read the following passage and answer the four
physical and mental and spiritual health status, items that follow. Your answers to these items
lifestyles and preferences. Thereby we also differ should be based on the passage only.
in what foods we tolerate or are sensitive to. So
we really cannot reduce so many complexities into
one diet or diet book. This explains the failure of Passage
diets across the world in curbing obesity. Unless It is no longer enough for us to talk about
the reasons for weight gain are well understood providing for universal access to education. Making
and addressed and unless habits are changed available schooling facilities is an essential
permanently, no diet is likely to succeed. prerequisite, but is insufficient to ensure that all
children attend school and participate in the
learning process. The school may be there, but
10. What is the most logical and rational
children may not attend or they may drop out after
inference that can be made from the above
a few months. Through school and social mapping,
passage?
we must address the entire gamut of social,
(a) Obesity has become an epidemic all economic, cultural and indeed linguistic and
over the world. pedagogic issues, factors that prevent children
(b) A lot of people are obsessed with from weaker sections and disadvantaged groups,
attaining a perfect body. as also girls, from regularly attending and
(c) Obesity is essentially an incurable complementing elementary education. The focus
disease. must be on the poorest and most vulnerable since
(d) There is no perfect diet or one these groups are the most disempowered and at
solution for obesity. the greatest risk of violation or denial of their right
to education.
Passage—8
Monoculture carries great risks. A single disease The right to education goes beyond free and
or pest can wipe out swathes of the world’s food compulsory education to include quality education
production, an alarming prospect given that its for all. Quality is an integral part of the right to
growing and wealthier population will eat 70% education. If the education process lacks quality,
more by 2050. The risks are magnified by the children are being denied their right. The Right
changing climate. As the planet warms and of Children to Free and Compulsory Education
monsoon rains intensify, farmlands in Asia will Act lays down that the curriculum should provide
flood. North America will suffer more intense for learning through activities, exploration and
droughts, and crop diseases will spread to new discovery. This places an obligation on us to
latitudes. change our perception of children as passive
receivers of knowledge, and to move beyond the
convention of using textbooks as the basis of
11. Which of the following is the most
examinations. The teaching-learning process must
logical, rational and crucial message given
become stress-free; and a massive programme for
by the passage?
curricular reform should be initiated to provide
(a) Preserving crop genetic diversity is for a child-friendly learning system, that is more
an insurance against the effects of relevant and empowering. Teacher
climate change. accountability systems and processes must ensure
(b) Despite great risks, monoculture is that children are learning, and that their right to
the only way to ensure food security

21
COMPREHENSION

learn in a child-friendly environment is not passage?


violated. Testing and assessment systems must be (a) The Right to Education now is a
reexamined and redesigned to ensure that these Fundamental Right.
do not force children to struggle between school (b) The Right to Education enables the
and tuition centres, and bypass childhood. children of poor and weaker sections
of the society to attend schools.
(c) The Right to Free and Compulsory
12. According to the passage, which of
Education should include quality
the following is/are of paramount
education for all.
importance under the Right to Education?
(d) The Government as well as parents
1. Sending of children to school by all should ensure that all children attend
parents schools.
2. Provision of adequate physical
infrastructure in schools
Directions for the following 4 (four) items :
3. Curricular reforms for developing
child-friendly learning system Read the following four passages and answer the
Select the correct answer using the code items that follow. Your answers to these items
given below. should be based on the passages only.
(a) 1 only
(b) 1 and 2 only Passage—1
(c) 3 only
Global population was around 1.6 billion in 1990—
(d) None of the above
today it is around 7.2 billion and growing. Recent
estimates on population growth predict a global
13. With reference to the above passage, the population of 9.6 billion in 2050 and 109 billion in
following assumptions have been made : 2100. Unlike Europe and North America, where
1. The Right to Education guarantees only three to four per cent of population is engaged
teachers’ accountability for the in agriculture, around 47 per cent of India’s
learning process of children. population is dependent upon agriculture. Even if
2. The Right to Education guarantees India continues to do well in the service sector
100% enrolment of children in the and the manufacturing sector picks up, it is
schools. expected that around 2030 when India overtakes
3. The Right to Education intends to China as the world’s most populous country,
take full advantage of demographic nearly 42 per cent of India’s population will still
dividend. be predominantly dependent on agriculture.
Which of the above assumptions is/are
valid? 16. Which of the following is the most logical
(a) 1 only and rational inference that can be made
(b) 2 and 3 only from the above passage?
(c) 3 only (a) Prosperity of agriculture sector is of
(d) 1, 2 and 3 critical importance to India.
(b) Indian economy greatly depends on
14. According to the passage, which one of the its agriculture.
following is critical in bringing quality in (c) India should take strict measures to
education? control its rapid population growth.
(a) Ensuring regular attendance of (d) India’s farming communities should
children as well as teachers in school switch over to other occupations to
(b) Giving pecuniary benefits to teachers improve their economic conditions.
to motivate them
(c) Understanding the socio-cultural Passage—2
background of children
Many pathogens that cause foodborne illnesses are
(d) Inculcating learning through activities
unknown. Food contamination can occur at any
and discovery
stage from farm to plate. Since most cases of food
poisoning go unreported, the true extent of global
15. What is the essential message in this foodborne illnesses is unknown. Improvements in

22
COMPREHENSION

international monitoring have led to greater public body and the spirit; though I seek all this, yet I do
awareness, yet the rapid globalization of food not wish to cut myself off from that past
production increases consumers’ vulnerability by completely. I am proud of that great inheritance
making food harder to regulate and trace. “We that has been and is, ours and I am conscious that
have the world on our plates”, says an official of I too, like all of us, am a link in that unbroken
WHO. chain which goes back to the dawn of history in
the immemorial past of India.
17. Which of the following is the most logical
corollary to the above passage? 19. The author wants India to rid herself of
(a) With more options for food come certain past bonds because
more risks. (a) he is not able to see the relevance of
(b) Food processing is the source of all the past
foodborne illnesses. (b) there is not much to be proud of
(c) We should depend on locally (c) he is not interested in the history of
produced food only. India
(d) Globalization of food production (d) they obstruct her physical and
should be curtailed. spiritual growth

Passage—3 Directions for the following 7 (seven) items :


I am a scientist, privileged to be somebody who Read the following four passages and answer the
tries to understand nature using the tools of items that follow. Your answers to these items
science. But it is also clear that there are some really should be based on the passages only.
important questions that science cannot really
answer, such as : Why is there something instead
of nothing? Why are we here? In those domains, I Passage—1
have found that faith provides a better path to ‘Desertification’ is a term used to explain a process
answers. I find it oddly anachronistic that in today’s of decline in the biological productivity of an
culture there seems to be a widespread ecosystem, leading to total loss of productivity.
presumption that scientific and spiritual views are While this phenomenon is often linked to the arid,
incompatible. semi-arid and sub-humid ecosystems, even in the
humid tropics, the impact could be most dramatic.
Impoverishment of human-impacted terrestrial
18. Which of the following is the most logical ecosystems may exhibit itself in a variety of ways
and rational inference that can be made : accelerated erosion as in the mountain regions
from the above passage? of the country, salinization of land as in the semi-
(a) It is the faith and not science that can arid and arid ‘green revolution’ areas of the
finally solve all the problems of country, e.g., Haryana and western Uttar Pradesh,
mankind. and site quality decline—a common phenomenon
(b) Science and faith can be mutually due to general decline in tree cover and
complementary if their proper monotonous monoculture of rice/wheat across the
domains are understood. Indian plains. A major consequence of
(c) There are some very fundamental deforestation is that it relates to adverse alterations
questions which cannot be answered in the hydrology and related soil and nutrient
by either science or faith. losses. The consequences of deforestation
(d) In today’s culture, scientific views are invariably arise out of site degradation through
given more importance than spiritual erosive losses. Tropical Asia, Africa and South
views. America have the highest levels of erosion. The
already high rates for the tropics are increasing at
Passage—4 an alarming rate (e.g., through the major river
systems—Ganga and Brahmaputra, in the Indian
Though I have discarded much of past tradition context), due to deforestation and ill-suited land
and custom, and am anxious that India should rid management practices subsequent to forest
herself of all shackles that bind and contain her clearing. In the mountain context, the declining
and divide her people, and suppress vast numbers moisture retention of the mountain soils, drying
of them, and prevent the free development of the

23
COMPREHENSION

up of the underground springs and smaller rivers 22. With reference to ‘desertification’, as
in the Himalayan region could be attributed to described in the passage, the following
drastic changes in the forest cover. An indirect assumptions have been made :
consequence is drastic alteration in the upland- 1. Desertification is a phenomenon in
lowland interaction, mediated through water. The tropical areas only.
current concern the tea planter of Assam has is 2. Deforestation invariably leads to
about the damage to tea plantations due to floods and desertification.
frequent inundation along the flood-plains of Which of the above assumptions is/are
Brahmaputra, and the damage to tea plantation valid?
and the consequent loss in tea productivity is due
(a) 1 only
to rising level of the river bottom because of
(b) 2 only
siltation and the changing course of the river
(c) Both 1 and 2
system. The ultimate consequences of site
(d) Neither 1 nor 2
desertification are soil degradation, alteration in
available water and its quality, and the consequent
decline in food, fodder and fuel-wood yields Passage—2
essential for the economic well-being of rural A diversity of natural assets will be needed to
communities. cope with climate change and ensure productive
agriculture, forestry, and fisheries. For example,
20. According to the passage, which of the crop varieties are needed that perform well under
following are the consequences of decline drought, heat, and enhanced C02. But the private-
in forest cover? sector and farmer-led process of choosing crops
favours homogeneity adapted to past or current
1. Loss of topsoil
conditions, not varieties capable of producing
2. Loss of smaller rivers
consistently high yields in warmer, wetter, or
3. Adverse effect on agricultural
drier conditions. Accelerated breeding
production
programmes are needed to conserve a wider pool
4. Declining of groundwater
of genetic resources of existing crops, breeds, and
Select the correct answer using the code their wild relatives. Relatively intact ecosystems,
given below. such as forested catchments, mangroves, wetlands,
(a) 1, 2 and 3 only can buffer the impacts of climate change. Under a
(b) 2, 3 and 4 only changing climate, these ecosystems are themselves
(c) 1 and 4 only at risk, and management approaches will need to
(d) 1, 2, 3 and 4 be more proactive and adaptive. Connections
between natural areas, such as migration
21. Which of the following is/are the correct corridors, may be needed to facilitate species
inference/ inferences that can be made from movements to keep up with the change in climate.
the passage?
1. Deforestation can cause changes in the 23. With reference to the above passage,
course of rivers. which of the following would assist us in
2. Salinization of land takes place due coping with the climate change?
to human activities only. 1. Conservation of natural water
3. Intense monoculture practice in plains sources
is a major reason for desertification 2. Conservation of wider gene pool
in Tropical Asia, Africa and South 3. Existing crop management
America. practices
Select the correct answer using the code 4. Migration corridors
given below. Select the correct answer using the code
(a) 1 only given below.
(b) 1 and 2 only (a) 1, 2 and 3 only
(c) 2 and 3 only (b) 1, 2 and 4 only
(d) None of the above is a correct (c) 3 and 4 only
inference (d) 1, 2, 3 and 4

24
COMPREHENSION

24. With reference to the above passage, the (a) leadership can be taught through war
following assumptions have been made : experience only
1. Diversification of livelihoods acts as (b) leadership can be acquired as well as
a coping strategy for climate change. taught
2. Adoption of monocropping practice (c) the results of training show that more
leads to the extinction of plant people acquire leadership than are
varieties and their wild relatives. expected
Which of the above assumptions is/are (d) despite rigorous instruction, very few
valid? leaders are produced
(a) 1 only
(b) 2 only 2017
(c) Both 1 and 2
(d) Neither 1 nor 2
Directions for the following 8 (eight) items :
Read the following eight passages and answer the
Passage—3
items that follow the passages. Your answers to
Today, the top environmental challenge is a these items should be based on the passages only.
combination of people and their aspirations. If the
aspirations are more like the frugal ones we had Passage—1
after the Second World War, a lot more is possible What climate change will undeniably do is cause
than if we view the planet as a giant shopping mall. or amplify events that hasten the reduction of
We need to get beyond the fascination with glitter resources. Competition over these diminishing
and understand that the planet works as a resources would ensue in the form of political or
biological system. even violent conflict. Resource-based conflicts
have rarely been overt and are thus difficult to
isolate. Instead they take on veneers that appear
25. Which of the following is the most crucial
more politically palatable. Conflicts over resources
and logical inference that can be made
like water are often cloaked in the guise of identity
from the above passage?
or ideology.
(a) The Earth can meet only the basic
needs of humans for food, clothing 1. What does the above passage imply?
and shelter. (a) Resource-based conflicts are always
(b) The only way to meet environmental politically motivated.
challenge is to limit human (b) There are no political solutions to
population. resolve environmental and resource-
(c) Reducing our consumerism is very based conflicts.
much in our own interest. (c) Environmental issues contribute to
(d) Knowledge of biological systems can resource stresses and political
only help us save this planet. conflict.
(d) Political conflict based on identity or
Passage—4 ideology cannot be resolved.
Some people believe that leadership is a quality
Passage—2
which you have at birth or not at all. This theory
The man who is perpetually hesitating which of
is false, for the art of leadership can be acquired
the two things he will do first, will do neither.
and can indeed be taught. This discovery is made
The man who resolves, but suffers his resolution
in time of war and the results achieved can surprise
to be changed by the first counter-suggestion of a
even the instructors. Faced with the alternatives
friend—who fluctuates from opinion to opinion
of going left or right, every soldier soon grasps
and veers from plan to plan—can never accomplish
that a prompt decision either way is better than
anything. He will at best be stationary and
an endless discussion. A firm choice of direction
probably retrograde in all. It is only the man who
has an even chance of being right while to do
first consults wisely, then resolves firmly and then
nothing will be almost certainly wrong.
executes his purpose with inflexible perseverance,
undismayed by those petty difficulties which daunt
26. The author of the passage holds the view a weaker spirit—that can advance to eminence in
that any line.

25
COMPREHENSION

(c) People cannot change their old


2. The keynote that seems to be emerging habits.
from the passage is that (d) People have neither civic sense nor
(a) we should first consult wisely and sense of privacy.
then resolve firmly
(b) we should reject suggestions of
friends and remain unchanged Passage - 5
(c) we should always remain broad- In the last two decades, the world’s gross domestic
minded product (GDP) has increased by 50 per cent,
(d) we should be resolute and whereas inclusive wealth has increased by a mere
achievement-oriented 6 per cent. In recent decades, GDP-driven
economic performance has only harmed inclusive
Passage—3 wealth like human capital; and natural capital like
During the summer in the Arctic Ocean, sea ice forests, land and water. While the world’s human
has been melting earlier and faster, and the winter capital which stands at 57 per cent of total inclusive
freeze has been coming later. In the last three wealth grew by only 8 per cent, the natural capital
decades, the extent of summer ice has declined by which is 23 per cent of total inclusive wealth
about 30 per cent. The lengthening period of declined by 30 per cent worldwide in the last two
summer melt threatens to undermine the whole decades.
Arctic food web, atop which stand polar bears.
5. Which of the following is the most crucial
3. Which among the following is the most inference from the above passage?
crucial message conveyed by the above (a) More emphasis should be laid on the
passage? development of natural capital.
(a) Climate change has caused Arctic (b) The growth driven by GDP only is
summer to be short but temperature neither desirable nor sustainable.
to be high. (c) The economic performance of the
(b) Polar bears can be shifted to South countries of the world is not
Pole to ensure their survival. satisfactory.
(c) Without the presence of polar bears, (d) The world needs more human capital
the food chains in Arctic region will under the present circumstances.
disappear.
(d) Climate change poses a threat to the Passage—6
survival of polar bears. By 2020, when the global economy is expected to
run short of 56 million young people, India, with
Passage—4 its youth surplus of 47 million, could fill the gap.
Why do people prefer open defecation and not It is in this context that labour reforms are often
want toilets or, if they have them, only use them cited as the way to unlock double-digit growth in
sometimes? Recent research has shown two critical India. In 2014, India’s labour force was estimated
elements: ideas of purity and pollution, and not to be about 40 per cent of the population, but 93
wanting pits or septic tanks to fill because they per cent of this force was in unorganized sector.
have to be emptied. These are the issues that Over the last decade, the compound annual growth
nobody wants to talk about, but if we want to rate (CAGR) of employment has slowed to 0.5 per
eradicate the practice of open defecation, they cent, with about 14 million jobs created during last
have to be confronted and dealt properly. year when the labour force increased by about 15
million.
4. Which among the following is the most
crucial message conveyed by the above 6. Which of the following is the most rational
passage? inference from the above passage?
(a) The ideas of purity and pollution are (a) India must control its population
so deep-rooted that they cannot be growth so as to reduce its
removed from the minds of the unemployment rate.
people. (b) Labour reforms are required in India
(b) People have to perceive toilet use and to make optimum use of its vast
pit-emptying as clean and not labour force productively.
polluting. (c) India is poised to achieve the double-

26
COMPREHENSION

digit growth very soon. religion we may belong, are equally the children
(d) India is capable of supplying the of India with equal rights, privileges and
skilled young people to other obligations. We cannot encourage communalism
countries. or narrow-mindedness, for no nation can be great
whose people are narrow in thought or action.
Passage—7
The very first lesson that should be taught to us 9. The challenge the author of the above
when we are old enough to understand it, is that passage throws to the public is to achieve
complete freedom from the obligation to work is (a) a high standard of living, progress
unnatural, and ought to be illegal, as we can escape and privileges
our share of the burden of work only by throwing (b) equal privileges, fulfilment of
it on someone else’s shoulders. Nature ordains that destiny and political tolerance
the human race shall perish of famine if it stops (c) spirit of adventure and economic
working. We cannot escape from this tyranny. The parity
question we have to settle is how much leisure we (d) hard work, brotherhood and
can afford to allow ourselves. national unity

7. The main idea of the passage is that Passage—2


(a) it is essential for human beings to “The individual, according to Rousseau, puts his
work person and all his power in common under the
(b) there should be a balance between supreme direction of the General Will and in our
work and leisure corporate capacity we receive each member as an
(c) working is a tyranny which we have indivisible part of the whole.”
to face
(d) human’s understanding of the 10. In the light of the above passage, the
nature of work is essential nature of General Will is best described as
(a) the sum total of the private wills of
Passage—8 the individuals
There is no harm in cultivating habits so long as (b) what is articulated by the elected
they are not injurious. Indeed, most of us are little representatives of the individuals
more than bundle of habits. Take away our habits (c) the collective good as distinct from
and the residuum would hardly be worth private wills of the individuals
bothering about. We could not get on without (d) the material interests of the
them. They simplify the mechanism of life. They community
enable us to do a multitude of things
automatically, which, if we had to give fresh and Passage—3
original thought to them each time, would make In a democratic State, where a high degree of
existence an impossible confusion. political maturity of the people obtains, the conflict
between the will, of the sovereign law-making
8. The author suggests that habits body and the organized will of the people seldom
(a) tend to make our lives difficult occurs.
(b) add precision to our lives
(c) make it easier for us to live 11. What does the above passage imply?
(d) tend to mechanize our lives (a) In a democracy, force is the main
phenomenon in the actual exercise of
Directions for the following 7 (seven) items : sovereignty.
Read the following seven passages and answer the (b) In a mature democracy, force to a
items that follow the passages. Your answers to great extent is the main phenomenon
these items should be based on the passages only. in the actual exercise of sovereignty.
Passage—1 (c) In a mature democracy, use of force
We have hard work ahead. There is no resting for is irrelevant in the actual exercise of
any of us till we redeem our pledge in full, till we sovereignty.
make all the people of India what destiny intends (d) In a mature democracy, force is
them to be. We are citizens of a great country, on narrowed down to a marginal
the verge of bold advance, and we have to live up phenomenon in the actual exercise of
to that high standard. All of us, to whatever sovereignty.

27
COMPREHENSION

been the chief source of human


Passage—4 progress.
A successful democracy depends upon Which of these assumptions is/are valid?
widespread interest and participation in politics, (a) 1 only
in which voting is an essential part. To deliberately (b) 2 only
refrain from taking such an interest, and from (c) Both 1 and 2
voting, is a kind of implied anarchy; it is to refuse (d) Neither 1 nor 2
one’s political responsibility while enjoying the
benefits of a free political society. Passage—7
There is more than a modicum of truth in the
12. This passage relates to assertion that “a working knowledge of ancient
(a) duty to vote history is necessary to the intelligent interpretation
(b) right to vote of current events”. But the sage who uttered these
(c) freedom to vote words of wisdom might well have added
(d) right to participate in politics something on the benefits of studying particularly
the famous battles of history for the lessons they
Passage—5 contain for those of us who lead or aspire to
In a free country, the man who reaches the position leadership. Such a study will reveal certain qualities
of leader is usually one of outstanding character and attributes which enabled the winners to win—
and ability. Moreover, it is usually possible to and certain deficiencies which caused the losers
foresee that he will reach such a position, since to lose. And the student will see that the same
early in life one can see his qualities of character. pattern recurs consistently, again and again,
But this is not always true in the case of a dictator; throughout the centuries.
often he reaches his position of power through
chance, very often through the unhappy state of 15. With reference to the above passage, the
his country. following assumptions have been made :
1. A study of the famous battles of
13. The passage seems to suggest that history would help us understand
(a) a leader foresees his future position the modern warfare.
(b) a leader is chosen only by a free 2. Studying the history is essential for
country anyone who aspires to be a leader.
(c) leader must see that his country is Which of these assumptions is/are valid?
free from despair (a) 1 only
(d) despair in a country sometimes leads (b) 2 only
to dictatorship (c) Both 1 and 2
(d) Neither 1 nor 2
Passage—6
The greatest blessing that technological progress Directions for the following 8 (eight) items :
has in store for mankind is not, of course, an Read the following seven passages and answer
accumulation of material possessions. The amount the items that follow the passages. Your answers
of these that can be effectively enjoyed by one to these items should be based on the passages
individual in one lifetime is not great. But there is only.
not the same narrow limit to the possibilities of
the enjoyment of leisure. The gift of leisure may Passage—1
be abused by people who have had no experience Disruption of traditional institutions,
of making use of it. Yet the creative use of leisure identifica-tions and loyalties is likely to lead to
by a minority in societies has been the mainspring ambivalent situations. It is possible that some
of all human progress beyond the primitive level. people may renew their identification with
traditional groups whereas others align themselves
14. With reference to the above passage, the with new groups and symbols emergent from
following assumptions have been made : processes of political development. In addition,
1. People always see the leisure time as political development tends to foster group
a gift and use it for acquiring more awareness of a variety of class, tribe, region, clan,
material possessions. language, religion, occupation and others.
2. Use of leisure by some people to
produce new and original things has 16. Which one of the following is the best

28
COMPREHENSION

explanation of the above passage? We live in digital times. The digital is not just
(a) Political development is not a something we use strategically and specifically to
unilinear process for it involves both do a few tasks. Our very perception of who we
growth and decay. are, how we connect to the world around us, and
(b) Traditional societies succeed in the ways in which we define our domains of life,
resisting positive aspects of political labour and language are hugely structured by the
development. digital technologies. The digital is everywhere
(c) It is impossible for traditional and, like air, invisible. We live within digital
societies to break away from systems, we live with intimate gadgets, we interact
lingering loyalties. through digital media, and the very presence and
(d) Sustenance of traditional loyalties is imagination of the digital has dramatically
conducive to political development. restructured our lives. The digital, far from being
a tool, is a condition and context that defines the
Passage—2 shapes and boundaries of our understanding of
There has been a significant trend worldwide the self, the society, and the structure of
towards regionalism in government, resulting in governance.
a widespread transfer of powers downwards
towards regions and communities since 1990s. This 18. Which among the following is the
process, which involves the creation of new most logical and essential message
political entities and bodies at a sub-national level conveyed by the above passage?
and an increase in their content and powers, is (a) All problems of governance can be
known as devolution. Devolution has been solved by using digital technologies.
characterized as being made up of three factors— (b) Speaking of digital technologies is
political legitimacy, decentralization of authority speaking of our life and living.
and decentralization of resources. Political (c) Our creativity and imagination
legitimacy here means a mass demand from below cannot be expressed without digital
for the decentralization process, which is able to media.
create a political force for it to take place. In many (d) Use of digital systems is imperative
cases, decentralization is initiated by the upper tier for the existence of mankind in
of government without sufficient political future.
mobilization for it at the grassroots level, and in
such cases the decentralization process often does Passage—4
not fulfil its objectives. The IMF has pointed out that the fast growing
economies of Asia face the risk of falling into
17. Which among the following is the most ‘middle-income trap’. It means that average
logical, rational and critical inference that incomes in these countries, which till now have
can be made from the above passage? been growing rapidly, will stop growing beyond
(a) Emergence of powerful mass leaders a point—a point that is well short of incomes in
is essential to create sub-national the developed West. The IMF identifies a number
political entities and thus ensure of causes of middle-income trap—none of which
successful devolution and is surprising—from infrastructure to weak
decentralization. institutions, to less than favourable macroeconomic
(b) The upper tier of government should conditions. But the broad, overall cause, says IMF,
impose devolution and is a collapse in the growth of productivity.
decentralization on the regional
communities by law or otherwise. 19. Which among the following is the most
(c) Devolution, to be successful, requires logical, rational and critical inference that
a democracy in which there is free can be made from the above passage?
expression of the will of the people (a) Once a country reaches middle-
at lower level and their active income stage, it runs the risk of
participation at the grassroots level. falling productivity which leads to
(d) For devolution to take place, a strong stagnant incomes.
feeling of regionalism in the masses (b) Falling into middle-income trap is a
is essential. general characteristic of fast growing
economies.
Passage—3 (c) There is no hope at all for emerging

29
COMPREHENSION

Asian economies to sustain the


growth momentum. 21. Which of the following is the most rational
(d) As regards growth of productivity, inference from the above passage?
the performance of Asian economies (a) The world will not be able to cope
is not satisfactory. with large-scale migration of climate
refugees.
Passage—5 (b) We must find the ways and means
An innovative India will be inclusive as well as to stop further climate change.
technologically advanced, improving the lives of (c) Climate change will be the most
all Indians. Innovation and R&D can mitigate important reason for the migration
increases in social inequality and relieve the of people in the future.
pressures created by rapid urbanization. The (d) Relation between climate change and
growing divergence in productivity between migration is not yet properly
agriculture and knowledge-intensive understood.
manufacturing and services threatens to increase
income inequality. By encouraging India’s R&D Passage—7
labs and universities to focus on the needs of poor Many farmers use synthetic pesticides to kill
people and by improving the ability of informal infesting insects. The consumption of pesticides
firms to absorb knowledge, an innovation and in some of the developed countries is touching
research agenda can counter this effect. Inclusive 3000 grams/hectare. Unfortunately, there are
innovation can lower the costs of goods and reports that these compounds possess inherent
services and create income – earning opportunities toxicities that endanger the health of the farm
for the poor people. operators, consumers and the environment.
Synthetic pesticides are generally persistent in
20. Which among the following is the most environment. Entering in food chain they destroy
logical and rational assumption that can the microbial diversity and cause ecological
be made from the above passage? imbalance. Their indiscriminate use has resulted
(a) Innovation and R & D is the only way in development of resistance among insects to
to reduce rural to urban migration. insecticides, upsetting of balance in nature and
(b) Every rapidly growing country resurgence of treated populations. ‘Natural pest
needs to minimize the divergence control using the botanical pesticides is safer to
between productivity in agriculture the user and the environment because they break
and other sectors. down into harmless compounds within hours or
(c) inclusive innovation and R&D can days in the presence of sunlight. Plants with
help create an egalitarian society. pesticidal properties have been in nature for
(d) Rapid urbanization takes place only millions of years without any ill or adverse effects
when a country’s economic growth on the ecosystem They are easily decomposed by
is rapid. many microbes common in most soils. They help
in the maintenance of biological diversity of
Passage—6 predators and the reduction of environmental
Climate change is likely to expose a large number contamination and human health hazards-Botanical
of people to increasing environmental risks forcing pesticides formulated from plants are
them to migrate. The international community is biodegradable and their use in crop protection is
yet to recognize this new category of migrants. a practical sustainable alternative.
There is no consensus on the definition and status
of climate refugees owing to the distinct meaning 22. On the basis of the above passage, the
the term refugees carry under international laws. following assumptions have been made :
There are still gaps in understanding how climate 1. Synthetic pesticides should never be
change will work as the root cause of migration. used in modern agriculture.
Even if there is recognition of climate refugees, 2. One of the aims of sustainable
who is going to provide protection? More emphasis agriculture is to ensure minimal
has been given to international migration due to ecological imbalance.
climate change. But there is a need to recognize 3. Botanical pesticides are more
the migration of such people within the countries effective as compared to synthetic
also so that their problems can be addressed pesticides.
properly. Which of the assumptions given above is/

30
COMPREHENSION

are correct? about pollution problems should


(a) 1 and 2 only increase.
(b) 2 only
(c) 1 and 3 only Passage—2
(d) 1,2 and 3 Productive jobs are vital for growth and a good
job is the best form of inclusion. More than half of
23. Which of the following statements is/are our population depends on agriculture, but the
correct regarding biopesticides? experience of other countries suggests that the
1. They are not hazardous to human number of people dependent on agriculture will
health. have to shrink if per capita incomes in agriculture
2. They are persistent in environment. are to go up substantially. While industry is
3. They are essential to maintain the creating jobs, too many such jobs are low-
biodiversity of any ecosystem. productivity non-contractual jobs in the
Select the correct answer using the code unorganized sector, offering low incomes, little
given below. protection, and no benefits. Service jobs are
(a) 1 only relatively of high productivity, but employment
(b) 1 and 2 only growth in services has been slow in recent years.
(c) 1 and 3 only
(d) 1, 2 and 3 25. Which among the following is the most
logical and rational inference that can be
Directions for the following 7 (seven) items : made from the above passage?
Read the following seven passages and answer (a) We must create conditions for the
the items that follow the passages. Your answers faster growth of highly productive
to these items should be based on the passages service jobs to ensure employment
only. growth and inclusion.
(b) We must shift the farm workers to
Passage—1 the highly productive manufacturing
An air quality index (AQI) is a way to combine and service sectors to ensure the
measurements of multiple air pollutant into a single economic growth and inclusion.
number or rating. This index is ideally kept (c) We must create conditions for the
constantly updated and available in different faster growth of productive jobs
places. The AQI is most useful when lots of outside of agriculture even while
pollution data are being gathered aid when improving the productivity of
pollution levels are normally, but not always, low. agriculture.
In such cases, if pollution levels spike for a few (d) We must emphasize the cultivation
days, the public can quickly take preventive action of high-yielding hybrid varieties
(like staying indoors) in response to an air quality and genetically modified crops to
warning. Unfortunately, that is not urban India. increase the per capita income in
Pollution levels in many large Indian cities are so agriculture.
high that they remain well above any health or
regulatory standard for large part of the year. If Passage—3
our index stays in the ‘Red/Dangerous’ region day A landscape-scale approach to land use can
after day, there is not much any one can do, other encourage greater biodiversity outside protected
than getting used to ignoring it. areas. During hurricane ‘Mitch’ in 1998, farms using
ecoagricultural practices suffered 58 per cent, 70
24. Which among the following is the most per cent and 99 per cent less damage in Honduras,
logical and rational inference that can be Nicaragua and Guatemala, respectively, than farms
made from the above passage? using conventional techniques. In Costa Rica,
(a) Our governments are not responsible vegetative windbreaks and fencerows boosted
enough to keep our cities pollution farmers’ income from pasture and coffee while also
free. increasing bird diversity. Bee pollination is more
(b) There is absolutely no need for air effective when agricultural fields are closer to
quality indices in our country. natural or seminatural habitat, a finding that
(c) Air quality index is not helpful to the matters because 87 per cent of the world’s 107
residents of many of our large cities. leading crops depend on animal pollinators. In
(d) In every city, public awareness Costa Rica, Nicaragua and Colombia silvopastoral

31
COMPREHENSION

systems that integrate trees with pastureland are Passage—5


improving the sustainability of cattle production, Over the last decade, Indian agriculture has become
and diversifying and increasing farmers’ income. more robust with record production of food grains
and oilseeds. Increased procurement,
26. Which among the following is the most consequently, has added huge stocks of food grains
logical and rational inference that can be in the granaries. India is one of the world’s top
made from the above passage? producers of rice, wheat, milk, fruits and
(a) Agricultural practices that enhance vegetables. India is still home to a quarter of all
biodiversity can often increase farm undernourished people in the world. On an
output and reduce the vulnerability average, almost half of the total expenditure of
to disasters. nearly half of the households is on food.
(b) All the countries of the world should
be encouraged to replace 28. Which among the following is the most
ecoagriculture with conventional logical corollary to the above passage?
agriculture. (a) Increasing the efficiency of farm-to-
(c) Ecoagriculture should be permitted fork value chain is necessary to
in protected areas without reduce the poverty and
destroying the biodiversity there. malnutri-tion.
(d) The yield of food crops will be very (b) Increasing the agricultural
high if ecoagricultural practices are productivity will automatically
adopted to cultivate them. eliminate the poverty and
malnutrition in India.
Passage—4 (c) India’s agricultural productivity is
The medium term challenge for Indian already great and it is not necessary
manufacturing is to move from lower to higher to increase it further.
tech sectors, from lower to higher value-added (d) Allocation of more funds for social
sectors, and from lower to higher productivity welfare and poverty alleviation
sectors. Medium tech industries are primarily programmes will ultimately
capital intensive and resource processing; and high eliminate the poverty and
tech industries are mainly capital and technology malnutrition in India.
intensive. In order to push the share of
manufacturing in overall GDP to the projected 25 Passage—6
per cent, Indian manufacturing needs to capture The States are like pearls and the Centre is the
the global market in sectors showing a rising trend thread which turns them into a necklace; if the
in demand. These sectors are largely high thread snaps, the pearls are scattered.
technology and capital intensive.
29. Which one of the following views
27. Which among the following is the most corroborates the above statement?
logical and rational inference that can be (a) A strong Centre and strong States
made from the above passage? make the federation strong.
(a) India’s GDP displays high value- (b) strong Centre is a binding force for
added and high productivity levels national integrity.
in medium tech and resource (c) A strong Centre is a hindrance to
processing industries. State autonomy.
(b) Promotion of capital and technology (d) State autonomy is a prerequisite for
intensive manufacturing is not a federation.
possible in India.
(c) India should push up the public Passage—7
investments and encourage the Really I think that the poorest he that is in England
private investments in research and has a life to live, as the greatest he, and therefore
development, technology truly, I think it is clear that every man that is to
upgradation and skill development. live under a government ought first by his own
(d) India has already gained a great consent to put himself under the government, and
share in global markets in sectors I do think that the poorest man in England is not
showing a rising trend in demand. at all bound in a strict sense to that government
that he has not had a voice to put himself under.

32
COMPREHENSION

promise of sustainable production that is high-yielding


30. The above statement argues for and emit low levels of greenhouse gases, but these are
(a) distribution of wealth equally to all still in the R & D stage.
(b) rule according to the consent of the
governed
1. What is/are the present constraint/constraints
(c) rule of the poor
(d) expropriation of the rich
in using biomass as fuel for power generation?
1. Lack of sustainable supply of biomass
2. Biomass production competes with food
2016 production
3. Bio-energy may not always be low carbon
on a life-cycle basis
Directions for the following 5 (five) items :
Select the correct answer using the code given
Read the following two passages and answer the items below :
that follow each passage. Your answers to these items (a) 1 and 2 only
should be based on the passages only. (b) 3 only
(c) 2 and 3 only
Passage — 1 (d) 1, 2 and 3
Biomass as fuel for power, heat, and transport has the
highest mitigation potential of all renewable sources. 2. Which of the following can lead to food security
It comes from agriculture and forest residues as well problem ?
as from energy crops. The biggest challenge in using 1. Using agricultural and forest residues as
biomass residues is a long-term reliable supply feedstock for power generation
delivered to the power plant at reasonable costs; the 2. Using biomass for carbon capture and
key problems are logistical constraints and the costs storage
of fuel collection. Energy crops, if not managed 3. Promoting the cultivation of energy crops
properly, compete with food production and may have Select the correct answer using the code given
undesirable impacts on food prices. Biomass production
below :
is also sensitive to the physical impacts of a changing
climate. (a) 1 and 2 only
(b) 3 only
(c) 2 and 3 only
Projections of the future role of biomass are probably (d) 1, 2 and 3
overestimated, given the limits to the sustainable
biomass supply, unless breakthrough technologies
3. In the context of using biomass, which of the
substantially increase productivity. Climate-energy
following is/are the characteristic/
models project that biomass use could increase nearly
characteristics of the sustainable production of
four-fold to around 150 - 200 exajoules, almost a quarter
biofuel ?
of world primary energy in 2050. However the
1. Biomass as a fuel for power generation
maximum sustainable technical potential of biomass
could meet all the primary energy
resources (both residues and energy crops) without
requirements of the world by 2050
disruption of food and forest resources ranges from
2. Biomass as a fuel for power generation
80 - 170 exajoules a year by 2050, and only part of this
does not necessarily disrupt food and forest
is realistically and economically feasible. In addition,
resources
some climate models rely on biomass-based carbon
3. Biomass as a fuel for power generation
capture and storage, an unproven technology, to
could help in achieving negative emissions,
achieve negative emissions and to buy some time
given certain nascent technologies
during the first half of the century.
Select the correct answer using the code given
Some liquid biofuels such as corn-based ethanol, mainly below :
for transport, may aggravate rather than ameliorate (a) 1 and 2 only
carbon emissions on a life-cycle basis. Second (b) 3 only
generation biofuels, based on ligno-cellulosic feedstocks (c) 2 and 3 only
- such as straw, bagasse, grass and wood - hold the (d) 1, 2 and 3

33
COMPREHENSION

4. With reference to the passage, following By killing transparency and competition, crony
assumptions have been made : capitalism is harmful to free enterprise, opportunity and
1. Some climate-energy models suggest that economic growth. Crony capitalism, where rich and
the use of biomass as a fuel for power the influential are alleged to have received land and
generation helps in mitigating greenhouse natural resources and various licences in return for
gas emissions payoffs to venal politicians, is now a major issue to be
2. It is not possible to use biomass as a fuel tackled. One of the greatest dangers to growth of
for power generation without disrupting developing economies like India is the middle-income
food and forest resources trap where crony capitalism creates oligarchies that
Which of these assumptions is/are valid? slow down the growth.
(a) 1 only
(b) 2 only 6. Which among the following is the most logical
(c) Both 1 and 2 corollary to the above passage ?
(d) Neither 1 nor 2 (a) Launching more welfare schemes and
allocating more finances for the current
Passage-2 schemes are urgently needed
We are witnessing a dangerous dwindling of (b) Efforts should be made to push up
biodiversity in our food supply. The green revolution is economic growth by other means and
a mixed blessing. Over time farmers have come to provide licences to the poor
rely heavily on broadly adapted, high yield crops to the (c) Greater transparency in the functioning of
exclusion of varieties adapted to the local conditions. the government and promoting the financial
Monocropping vast fields with the same genetically inclusion are needed at present
uniform seeds helps boost yield and meet immediate (d) We should concentrate more on
hunger needs. Yet high-yield varieties are also developing manufacturing sector than
genetically weaker crops that require expensive service sector
chemical fertilizers and toxic pesticides. In our focus
on increasing the amount of food we produce today,
we have accidentally put ourselves at risk for food Passage-2
shortages in future.
Climate adaptation may be rendered ineffective if
policies are not designed in the content of other
5. Which among the following is the most logical development concerns. For instance, a comprehensive
and critical inference that can be made from strategy that seeks to improve food security in the
the above passage ? context of climate change may include a set of
(a) In our agricultural practices, we have coordinated measures related to agricultural extension,
become heavily dependent on expensive crop diversification, integrated water and pest
chemical fertilizers and toxic pesticides management and agricultural information services.
only due to green revolution Some of these measures may have to do with climate
(b) Monocropping vast fields with high-yield changes and others with economic development.
varieties is possible due to green revolution
(c) Monocropping with high-yield varieties is
7. What is the most logical and rational inference
the only way to ensure food security to
that can be made from the above passage ?
millions
(a) It is difficult to pursue climate adaptation
(d) Green revolution can pose a threat to
in the developing countries
biodiversity in food supply and food
(b) Improving food security is a far more
security in the long run
complex issue than climate adaptation
(c) Every developmental activity is directly or
Directions for the following 8 (eight) items : indirectly linked to climate adaptation
Read the following eight passages and answer the item (d) Climate adaptation should be examined in
that follows each passage. Your answers to these items tandem with other economic development
should be based on the passages only. options

Passage-1 Passage-3

34
COMPREHENSION

Understanding of the role of biodiversity in the and discontinue the use of dry latrines. A more sustained
hydrological cycle enables better policy-making. The and rigorous campaign needs to be launched towards
term biodiversity refers to the variety of plants, animals, the right to sanitation on a very large scale. This should
microorganisms, and the ecosystems in which they primarily focus on the abolition of manual scavenging.
occur. Water and biodiversity are interdependent. In
reality, the hydrological cycle decides how biodiversity 10. With reference to the above passage, consider
junctions. In turn, vegetation and soil drive the the following statements :
movement of water. Every glass of water we drink 1. Urban sanitation problems can be fully
has, at least in part, passed through fish, trees, bacteria, solved by the abolition of manual
soil and other organisms. Passing through these scavenging only
ecosystems, it is cleansed and made fit for consumption. 2. There is a need to promote greater
The supply of water is a critical service that the awareness on safe sanitation practices in
environment provides. urban areas
Which of the statements given above is/are
8. Which among the following is the most critical correct?
inference that can be made from the above (a) 1 only
passage ? (b) 2 only
(a) Biodiversity sustains the ability of nature (c) Both 1 and 2
to recycle water (d) Neither 1 nor 2
(b) We cannot get potable water without the
existence of living organisms Passage-6
(c) Wants, animals and microorganisms
To understand the nature and quantity of Government
continuously interact among themselves
proper for man, it is necessary to attend to his character.
(d) Living organisms could not have come into
As nature created him for social life, she fitted him for
existence without hydrological cycle
the station she intended. In all cases she made his
natural wants greater than his individual powers. No
Passage-4 one man is capable, without the aid of society, of
In the last decade, the banking sector has been supplying his own wants; and those wants, acting upon
restructured with a high degree of automation and every individual, impel the whole of them into society.
products that mainly serve middle-class and upper
middle-class society. Today there is a need for a new
11. Which among the following is the most logical
agenda for the banking and non-banking financial
and rational inference that can be made from
services that does not exclude the common man.
the above passage ?
(a) Nature has created a great diversity in
9. Which one of the following is the message that human society
is essentially implied in the above passage ? (b) Any given human society is always short
(a) Need for more automation and more of its wants
products of banks (c) Social life is a specific characteristic of
(b) Need for a radical restructuring of our man
entire public finance system (d) Diverse natural wants forced man
(c) Need to integrate banking and non- towards social system
banking institutions
(d) Need to promote financial inclusion Passage-7
The nature of the legal imperatives in any given state
Passage-5 corresponds to the effective demands that state
Safe and sustainable sanitation in slums has encounters, and that these, in their turn, depend, in a
immeasurable benefits to women and girls in terms of general way, upon the manner in which economic
their health, safety, privacy and dignity. However, power is distributed in the society which the state
women do not feature in most of the schemes and controls.
policies on urban sanitation. The fact that even now
the manual scavenging exists, only goes to show that
12. The statement refers to :
not enough has been done to promote pour-flush toilets
(a) the antithesis of Politics and Economics

35
COMPREHENSION

(b) the interrelationship of Politics and vegetables.


Economics
(c) the predominance of Economics over 14. What is the critical message conveyed by the
Politics above passage ?
(d) the predominance of Politics over (a) Our increasing demand for foods sourced
Economics from animals puts a greater burden on our
natural resources
Passage-8 (b) Diets based on grains, nuts, fruits and
About 15 per cent of global greenhouse gas emissions vegetables are best suited for health in
come from agricultural practices. This includes nitrous developing countries
oxide from fertilizers; methane from livestock, rice (c) Human beings change their food habits
production, and manure storage; and carbon dioxide from time to time irrespective of the health
(CO2) from burning biomass, but this excludes CO2, concerns
emissions from soil management practices, savannah (d) From a global perspective, we still do not
burning and deforestation. Forestry, land use, and land- know which type of diet is best for us
use change account for another 17 per cent of
greenhouse gas emissions each year, three quarters of Passage-2
which come from tropical deforestation. The remainder All humans digest mother’s milk as infants, but until
is largely from draining and burning tropical peatland. cattle began being domesticated 10,000 years ago,
About the same amount of carbon is stored in the children once weaned no longer needed to digest milk.
world’s peatlands as is stored in the Amazon rainforest. As a result, they stopped making the enzyme lactase,
which breaks down the sugar lactose into simple
13. Which among the following is the most logical sugars. After humans began herding cattle, it became
and rational inference that can be made from tremendously advantageous to digest milk, and lactose
the above passage ? tolerance evolved independently among cattle herders
(a) Organic farming should immediately in Europe, the middle East and Africa. Groups not
replace mechanised and chemical dependant on cattle, such as the Chinese and Thai,
dependant agricultural practices all over remain lactose intolerant.
the world
(b) It is imperative for us to modify our land 15. Which among the following is the most logical
use practices in order to mitigate climate assumption that can be made from the above
change passage ?
(c) There are no technological solutions to the (a) About 10,000 years ago, the domestication
problem of greenhouse gas emissions of animals took place in some parts of the
(d) Tropical areas are the chief sites of carbon world
sequestration (b) A permanent change in the food habits of
a community can bring about a genetic
Directions for the following 8 (eight) items : Read change in its members
the following five passages and answer the items that (c) Lactose tolerant people only are capable
follow each passage. Your answers to these items of getting simple sugars in their bodies
should be based on the passages only. (d) People who are not lactose tolerant cannot
digest any dairy product
Passage-1
As we look to 2050, when we will need to feed two Passage-3
billion more people, the question of which diet is best “The conceptual difficulties in National Income
has taken on new urgency. The foods we choose to comparisons between underdeveloped and
eat in the coming decades will have dramatic industrialised countries are particularly serious because
ramifications for the planet. Simply put, a diet that a part of the national output in various underdeveloped
revolves around meat and dairy, a way of eating that is countries is produced without passing through the
on the rise throughout the developing world, will take a commercial channels.”
greater toll on the world’s resources than one that
revolves around unrefined grains, nuts, fruits and 16. In the above statement, the author implies that:

36
COMPREHENSION

(a) the entire national output produced and enduring basis; it is not so much and not always, of a
consumed in industrialized countries passes constraint for an open, and growing economy, which
through commercial channels has adequate exchange surpluses to buy food abroad.
(b) the existence of a non-commercialized For the world as a whole, supply-demand balance is
sector in different underdeveloped always an inescapable prerequisite for warding off
countries renders the national income hunger and starvation. However, global availability of
comparisons over countries difficult adequate supply does not necessarily mean that food
(c) no part of national output should be would automatically move from countries of surplus to
produced and consumed without passing countries of deficit if the latter lack in purchasing power.
through commercial channels The uneven distribution of hunger, starvation, under-
(d) a part of the national output being produced or malnourishment, etc., at the world-level, thus owes
and consumed without passing through itself to the presence of empty-pocket hungry mouths,
commercial channels is a sign of overwhelmingly confined to the underdeveloped
underdevelopment economies. Inasmuch as ‘a two-square meal’ is of
elemental significance to basic human existence, the
Passage-4 issue of worldwide supply of food has been gaining
An increase in human-made carbon dioxide in the significance, in recent times, both because the quantum
atmosphere could initiate a chain reaction, between and the composition of demand has been undergoing
plants and microorganisms that would unsettle one of big changes, and because, in recent years, the
the largest carbon reservoirs on the planet -soil. In a capabilities of individual countries to generate
study, it was found that the soil, which contains twice uninterrupted chain of food supplies have come under
the amount of carbon present in all plants and Earth’s strain. Food production, marketing and prices,
atmosphere combined, could become increasingly especially price-affordability by the poor in the poor in
volatile as people add more carbon dioxide to the the developing world, have become global issues that
atmosphere. This is largely because of increased plant need global thinking and global solutions.
growth. Although a greenhouse gas and a pollutant,
carbon dioxide also supports plant growth. As trees 18. According to the above passage, which of the
and other vegetation flourish in a carbon dioxide-rich following are the fundamental solutions for the
future, their roots could stimulate, microbial activity in world food security problem ?
soil that may in turn accelerate the decomposition of 1. Setting up more agro-based industries
soil carbon and its release into the atmosphere as 2. Improving the price affordability by the
carbon dioxide. poor
3. Regulating the conditions of marketing
17. Which among the following is the most logical 4. Providing food subsidy to one and all
corollary to the above passage ? Select the correct answer using the code given
(a) Carbon dioxide is essential for the survival below:
of microorganisms and plants (a) 1 and 2 only
(b) Humans are solely responsible for the (b) 2 and 3 only
release of carbon dioxide into the (c) 1, 3 and 4 only
atmosphere (d) 1, 2, 3 and 4
(c) Microorganisms and soil carbon are mainly
responsible for the increased plant growth 19. According to the above passage, the biggest
(d) Increasing green cover could trigger the challenge to world agriculture is :
release of carbon trapped in soil (a) to find sufficient land for agriculture and
to expand food processing industries
Passage-5 (b) to eradicate hunger in underdeveloped
countries
Historically, the biggest challenge to world agriculture
(c) to achieve a balance between the
has been to achieve a balance between demand for
production of food and non-food items
and supply of food. At the level of individual countries,
(d) to achieve a balance between demand for
the demand-supply balance can be a critical policy issue
and supply of food
for a closed economy, especially if it is a populous
economy and its domestic agriculture is not growing
sufficiently enough to ensure food supplies, on an

37
COMPREHENSION

20. According to the above passage, which of the service that is not totally subservient to the political
following helps/help in reducing hunger and executive but will have the strength to function in larger
starvation in the developing economies ? public interest. The need to balance internal and external
1. Balancing demand and supply of food accountability is thus built into the Constitution. The
2. Increasing imports of food issue is where to draw the line. Over the years, the
3. Increasing purchasing power of the poor emphasis seems to have tilted in favour of greater
4. Changing the food consumption patterns internal accountability of the civil services to the political
and practices leaders of the day who in turn are expected to be
Select the correct answer using the code given externally accountable to the society at large through
below : the election process. This system for seeking
(a) 1 only accountability to society has not worked out, and has
(b) 2, 3 and 4 only led to several adverse consequences for governance.
(c) 1 and 3 only
(d) 1, 2, 3 and 4 Some special measures can be considered for improving
accountability in civil services. Provisions of articles
21. The issue of worldwide supply of food has 311 and 312 should be reviewed and laws and
gained importance mainly because of: regulations framed to ensure external accountability
1. overgrowth of the population worldwide of civil services. The proposed Civil Services Bill seeks
2. sharp decline in the area of food production to address some of these requirements. The respective
3. limitation in the capabilities for sustained roles of professional civil services and the political
supply of food executive should be defined so that professional
Select the correct answer using the code given managerial functions and management of civil services
below : are depoliticized. For this purpose, effective statutory
(a) 1 and 2 only civil service boards should be created at the centre
(b) 3 only and in the states. Decentralization and devolution of
(c) 2 and 3 only authority to bring government and decision making
(d) 1, 2 and 3 closer to the people also helps to enhance accountability.

Directions for the following 6 ( six ) items: 22. According to the passage, which of the following
Read the following two passages and answer the items factor/factors led to the adverse consequences
that follow each passage. Your answers to these items for governance/public administration ?
should be based on the passages only. 1. Inability of civil services to strike a balance
between internal and external
Passage-1 accountabilities
2. Lack of sufficient professional training to
Accountability, or the lack of it, in governance generally,
the officers of All India Services
and civil services, in particular, is a major factor
3. Lack of proper service benefits in civil
underlying the deficiencies in governance and public
services
administration. Designing an effective framework for
4. Lack of Constitutional provisions to define
accountability has been a key element of the reform
the respective roles of professional civil
agenda. A fundamental issue is whether civil services
services vis-a-vis political executive in this
should be accountable to the political executive of the
context
day or to society at large. In other words, how should
Select the correct answer using the code given
internal and external accountability be reconciled ?
below :
Internal accountability is sought to be achieved by
(a) 1 only
internal performance monitoring, official supervision
(b) 2 and 3 only
by bodies like the Central Vigilance Commission and
(c) 1 and 4 only
Comptroller and Auditor General, and judicial review
(d) 2, 3 and 4
of executive decisions. Articles 311 and 312 of the
Indian Constitution provide job security and safeguards
to the civil services, especially the All India Services. 23. With reference to the passage, the following
The framers of the Constitution had envisaged that assumptions have been made :
provision of these safeguards would result in a civil 1. Political executive is an obstacle to the
accountability of the civil services to the

38
COMPREHENSION

society 1. Human relationships are derived from their


2. In the present framework of Indian polity, religious traditions
the political executive is no longer 2. Human beings can be duty bound only if
accountable to the society they believe in god
Which of these assumptions is/are valid? 3. Religious traditions are essential to
(a) 1 only practice and understand justice
(b) 2 only Which of these assumption(s) is/are valid ?
(c) Both 1 and 2 (a) 1 only
(d) Neither 1 nor 2 (b) 2 and 3 only
(c) 1 and 3 only
24. Which one of the following is the essential (d) 1, 2 and 3
message implied by this passage?
(a) Civil services are not accountable to the 27. Which one of the following is the crux of this
society they are serving passage ?
(b) Educated and enlightened persons are not (a) Our duties to one another derive from our
taking up political leadership religious traditions
(c) The framers of the Constitution did not (b) Having relationship to the divine principle
envisage the problems being encountered is a great virtue
by the civil services (c) Balance between rights and duties is
(d) There is a need and scope for reforms to crucial to the delivery of justice in a society
improve the accountability of civil services (d) Religious concept of rights is primarily
derived from our relationship to god
25. According to the passage, which one of the
following is not a means of enhancing internal
accountability of civil services ? 2015
(a) Better job security and safeguards
(b) Supervision by Central Vigilance
Commission Directions for the following 7 (seven) items :
(c) Judicial review of executive decisions Read the following four passages and answer the items
(d) Seeking accountability through enhanced that follow. Your answers to these items should be
participation by people in decision making based on the passages only.
process

Passage-2 Passage — 1
In general, religious traditions stress our duty to god, India has suffered from persistent high inflation.
or to some universal ethical principle. Our duties to Increase in administered prices, demand and supply
one another derive from these. The religious concept imbalances, imported inflation aggravated by rupee
of rights is primarily derived from our relationship to depreciation, and speculation — have combined to keep
this divinity or principle and the implication it has on high inflation going. If there is an element common to
our other relationships. This correspondence between all of them, it is that many of them are the outcomes of
rights and duties is critical to any further understanding economic reforms. India’s vulnerability to the effects
of justice. of changes in international prices has increased with
trade liberalisation. The effort to reduce subsidies has
resulted in a continuous increase in the prices of
But, for justice to be practiced; virtue, rights and” duties
commodities that are administered.
cannot remain formal abstractions. They must be
grounded in a community (common unity) bound
together by a sense of common union (communion). 1. What is the most logical, rational and crucial
Even as a personal virtue, this solidarity is essential to message that is implied in the above passage?
the practice and understanding of justice. (a) Under the present circumstances, India
should completely avoid all trade
26. With reference to the passage, the following liberalisation policies and all subsidies.
assumptions have been made : (b) Due to its peculiar socio-economic
situation, India is not yet ready for trade

39
COMPREHENSION

liberalisation process. (c) Old values, ideas and traditions persist


(c) There is no solution in sight for the despite the dynamic nature of human
problems of continuing poverty and society.
inflation in India in the near future. (d) Constitutional guarantee of freedom of
(d) Economic reforms can often create a high speech is not in the interest of society.
inflation economy.
Passage — 4
Passage — 2 Climate change is a complex policy issue with major
No Right is absolute, exclusive or inviolable. The Right implications in terms of finance. All actions to address
of personal property, similarly, has to be perceived in climate change ultimately involve costs. Funding is vital
the larger context of its assumed legitimacy. The Right for countries like India to design and implement
of personal property should unite the principle of liberty adaptation and mitigation plans and projects. Lack of
with that of equality, and both with the principle of funding is a large impediment to implementing
cooperation. adaptation plans. The scale and magnitude of the
financial support required by developing countries to
enhance their domestic mitigation and adaptation
2. In the light of the argument in the above passage, actions are a matter of intense debate
which one of the following statements is the
most convincing explanation? in the multilateral negotiations under the United Nations
(a) The Right of personal property is a Natural Framework Convention on Climate Change
Right duly supported by statutes and (UNFCCC). The Convention squarely puts the
scriptures. responsibility for provision of financial support on the
(b) Personal property is a theft and an developed countries, taking into account their
instrument of exploitation. The Right of contribution to the stock of greenhouse gases (GHGs)
personal property is therefore violative of in the atmosphere. Given the magnitude of the task
economic justice. and the funds required, domestic finances are likely to
(c) The Right of personal property is violative fall short of the current and projected needs of the
of distributive justice and negates the developing countries. Global funding through the
principle of cooperation. multilateral mechanism of the Convention will enhance
(d) The comprehensive idea of economic their domestic capacity to finance the mitigation efforts.
justice demands that the Right of each
person to acquisition of property has to be 4. According to the passage, which of the following
reconciled with that of others. is/are a matter of intense debate in the multilateral
negotiations under UNFCCC regarding the role
Passage — 3 of developing countries in climate change?
The conflict between man and State is as old as State 1. The scale and size of required financial
history. Although attempts have been made for support.
centuries to bring about a proper adjustment between 2. The crop loss due to climate change in
the competing claims of State and the individual, the the developing countries.
solution seems to be still far off. This is primarily 3. To enhance the mitigation and adaptation
because of the dynamic nature of human society where actions in the developing countries.
old values and ideas constantly yield place to new ones. Select the correct answer using the code given
It is obvious that if individuals are allowed to have below:
absolute freedom of speech and action, the result would (a) 1 only
be chaos, ruin and anarchy. (b) 2 and 3 only
(c) 1 and 3 only
(d) 1, 2 and 3
3. The author’s viewpoint can be best summed up
in which of the following statements? 5. In this passage, the Convention puts the
(a) The conflict between the claims of State responsibility for the provision of financial
and individual remains unresolved. support on the developed countries because of
(b) Anarchy and chaos are the obvious results 1. their higher level of per capita incomes.
of democratic traditions. 2. their large quantum of GDP.

40
COMPREHENSION

3. their large contribution to the stock of 8. Which among the following is the most logical
GHGs in the atmosphere. corollary to the above passage?
Select the correct answer using the code given (a) Government should allocate more funds
below: to poverty alleviation programmes and
(a) 1 only increase food subsidies to the poor
(b) 1 and 2 only communities.
(c) 3 only (b) Poverty and climate impacts reinforce
(d) 1, 2 and 3 each other and therefore we have to re-
imagine our food systems.
6. With regards to developing countries, it can be (c) All the countries of the world must unite
inferred from the passage that climate change in fighting poverty and malnutrition and
is likely to have implications on their treat poverty as a global problem,
1. domestic finances. (d) We must stop unsustainable agricultural
2. capacity for multilateral trade. practices immediately and control food
Select the correct answer using the code given prices.
below:
(a) 1 only Passage —2
(b) 2 only
(c) Both 1 and 2 The Global Financial Stability Report finds that the share
(d) Neither 1 nor 2 of portfolio investments from advanced economies in
the total debt and equity investments in emerging
economies has doubled in the past decade to 12 percent.
7. Which one of the following is essentially The phenomenon has implications for Indian policy
discussed in the passage? makers as foreign portfolio investments in the debt and
(a) Conflict between developed and equity markets have been on the rise. The phenomenon
developing countries regarding support for is also flagged as a threat that could compromise global
mitigation financial stability in a chain reaction, in the event of
(b) Occurrence of climate change due to United States Federal Reserve’s imminent reversal of
excessive exploitation of natural resources its “Quantitative Easing” policy.
by the developed countries
(c) Lack of political will on the part of all the
countries to implement adaptation plans 9. Which among the following is the most rational
(d) Governance problems of developing and critical inference that can be made from
countries as a result of climate change the above passage?
(a) Foreign portfolio investments are not good
Directions for the following 7 (seven) items: for emerging economies.
Read the following six passages and answer the items (b) Advanced economies undermine the global
that follow. Your answers to these items should be financial stability.
based on the passages only. (c) India should desist from accepting foreign
portfolio investments in the future.
(d) Emerging economies are at a risk of shock
Passage — 1 from advanced economies.
Climate change is already making many people hungry
all over the world, by disrupting crop yields and pushing Passage — 3
up prices. And it is not just food but nutrients that are
becoming scarcer as the climate changes. It is the Open defecation is disastrous when practised in very
poorest communities that will suffer the worst effects densely populated areas, where it is impossible to keep
of climate change, including increased hunger and away human faeces from crops, wells, food and
malnutrition as crop production and livelihoods are children’s hands. Groundwater is also contaminated
threatened. On the other hand, poverty is a driver of by open defecation. Many ingested germs and worms
climate change, as desperate communities resort to spread diseases. They prevent the body from absorbing
unsustainable use of resources to meet current needs. calories and nutrients. Nearly one-half of India’s
children remain malnourished. Lakhs of them die from
preventable conditions. Diarrhoea leaves Indians’

41
COMPREHENSION

bodies smaller on average than those of people in some to all people in developing countries like India and more
poorer countries where people eat fewer calories. so, in rural areas. Savings help poor households manage
Underweight mothers produce stunted babies prone volatility in cash flow, smoothen consumption, and build
to sickness who may fail to develop their full cognitive working capital. Poor households without access to a
potential. The germs released into environment harm formal savings mechanism encourage immediate
rich and poor alike, even those who use latrines. spending temptations.

10. Which among the following is the most critical 12. With reference to the above passage, consider
inference that can be made from the above the following statements:
passage? 1. Indian financial institutions do not offer
(a) The Central and State governments in any financial instruments to rural
India do not have enough resources to households to mobilise their savings.
afford a latrine for each household. 2. Poor households tend to spend their
(b) Open defecation is the most important earnings/savings due to lack of access to
public health problem of India. appropriate financial instruments.
(c) Open defecation reduces the human Which of the statements given above is/are
capital of India’s workforce. correct?
(d) Open defecation is a public health problem (a) 1 only
in all developing countries. (b) 2 only
(c) Both 1 and 2
Passage — 4 (d) Neither 1 nor 2
We generally talk about democracy but when it comes
to any particular thing, we prefer a belonging to our 13. What is the crucial message conveyed in the
caste or community or religion. So long as we have passage?
this kind of temptation, our democracy will remain a (a) Establish more banks
phoney kind of democracy. We must be in a position to (b) Increase the Gross Domestic Product
respect a man as a man and to extend opportunities (GDP) growth rate
for development to those who deserve them and not to (c) Increase the interest rate of bank deposits
those who happen to belong to our community or race. (d) Promote financial inclusion
This fact of favouritism has been responsible for much
discontent and ill-will in our country. Passage — 6

11. Which one of the following statements best Governments may have to take steps which would
sums up the above passage? otherwise be an infringement on the Fundamental
(a) Our country has a lot of diversity with its Rights of individuals, such as acquiring a person’s land
many castes, communities and religions. against his will, or refusing permission for putting up a
(b) True democracy could be established by building, but the larger public interest for which these
providing equal opportunities to all. are done must be authorized by the people (Parliament).
(c) So far none of us have actually understood Discretionary powers to the administration can be done
the meaning of democracy. away with. It is becoming more and more difficult to
(d) It will never be possible for us to establish keep this power within limits as the government has
truly democratic governance in our many number of tasks to perform. Where discretion
country. has to be used, there must be rules and safeguards to
prevent misuse of that power. Systems have to be
Passage — 5 devised which minimise, if not prevent, the abuse of
discretionary power. Government work must be
The existence/establishment of formal financial conducted within a framework of recognised rules and
institutions that offer safe, reliable, and alternative principles, and decisions should be similar and
financial instruments is fundamental in mobilising predictable.
savings. To save, individuals need access to safe and
reliable financial institutions, such as banks, and to 14. Which among the following is the most logical
appropriate financial instruments and reasonable assumption that can be made from the above
financial incentives. Such access is not always available passage?

42
COMPREHENSION

(a) Government should always be given wide 16. Which one of the following statements best
discretionary power in all matters of sums up the above passage?
administration. (a) The population of the world is growing
(b) The supremacy of rules and safeguards very fast.
should prevail as opposed to the influence (b) Food security is a perennial problem only
of exclusive discretion of authority. in developing countries.
(c) Parliamentary democracy is possible only (c) The world does not have enough resources
if the Government has wider discretionary to meet the impending food scarcity.
power. (d) Food security is increasingly a collective
(d) None of the above statements is a logical challenge.
assumption that can be made from this
passage. Passage — 3

Directions for the following 8 (eight) items: Many people in India feel that if we cut our defence
Read the following six passages and answer the items expenditure on weapon-building, we can create a
that follow. Your answers to these items should be climate of peace with our neighbours, subsequently
based on the passages only. reducing the conflict or creating a no-war situation.
People who proclaim such ideas are either the victims
Passage — 1 of war or the propagators of false argument.
Human history abounds in claims and theories confining
the right of governing to a few select citizens. Exclusion 17. With reference to the above passage, which of
of the many is justified on the ground that human beings the following is the most valid assumption ?
may be rightfully segregated for the good of society (a) Building of weapons systems by us has
and viability of the political process. instigated our neighbours to wage wars
against us.
15. Which one of the following statements is least (b) The greater spending on weapon-building
essential as a part of the argument in the above by us would lessen the possibility of armed
passage? conflict with our neighbours.
(a) Man seeks control over external things (c) It is necessary to have state of the art
affecting him. weapons systems for national security.
(b) In society, there are ‘super’ and ‘sub’ (d) Many people in India believe that we are
human beings. wasting our resources on weapon-
(c) Exceptions to universal citizen participation building.
are conducive to systemic efficacy.
(d) Governing implies recognition of
disparities in individual capacities. Passage —4

Passage — 2 India accounts for nearly a fifth of the world’s child


deaths. In terms of numbers, it is the highest in the
By 2050, the Earth’s population will likely have swelled world — nearly 16 lakhs every year. Of these, more
from seven to nine billion people. To fill all those than half die in the first month of life. Officials believe
stomachs — while accounting for shifting consumption that the reason for this is the absence of steps to
patterns, climate change, and a finite amount of arable propagate basic health practices relating to breast
land and potable water — some experts say food feeding and immunisation. Also the large reproductive
production will have to double. How can we make the population of 2.6 crore remains bereft of care during
numbers add up ? Experts say higher yielding crop the critical phases of pregnancy and post-delivery.
varieties and more efficient farming methods will be Added to this is the prevalence of child marriages,
crucial. So will waste reduction. Experts urge cities to anaemia among young women and lack of focus on
reclaim nutrients and water from waste streams and adolescent sanitation, all of which impact child death
preserve farmland. Poor countries, they say, can rates.
improve crop storage and packaging and rich nations
could cut back on resource-intensive foods like meat. 18. Which is the critical inference that can be made
from the above passage?

43
COMPREHENSION

(a) A lot of Indians are illiterate and hence do speech or writing. I have thus been spared many a
not recognize the value of basic health mishap and waste of time. Experience has taught me
practices. that silence is part of the spiritual discipline of a votary
(b) India has a very huge population and the of truth. Proneness to exaggerate, to suppress or
government alone cannot manage public modify the truth, wittingly or unwittingly, is a natural
health services. weakness of man, and silence is necessary in order to
(c) Universalization and integration of surmount it. A man of few words will rarely be
maternal health and child health services thoughtless in his speech; he will measure every word.
can effectively address the problem. We find so many people impatient to talk. There is no
(d) The nutrition of women in child bearing chairman of a meeting who is not pestered with notes
age does not affect child mortality rate. for permission to speak. And whenever the permission
is given the speaker generally exceeds the time-limit,
Passage — 5 asks for more time, and keeps on talking without
permission. All this talking can hardly be said to be of
Foods travel more than the people who eat them. any benefit to the world. It is so much waste of time.
Grocery stores and supermarkets are loaded with My shyness has been in reality my shield and buckler.
preserved and processed foods. This, however, often It has allowed me to grow. It has helped me in my
leads to environmental threats, such as pollution discernment of truth.
generated by long distance food transportation and
wastage of food during processing and transportation, 20. The author says that a thoughtless word hardly
destruction of rain forests, reduced nutritional content, ever escapes his tongue or pen. Which one of
increased demand for preservation and packaging. Food the following is not a valid reason for this?
insecurity also increases as the produce comes from (a) He has no intention to waste his time.
regions that are not feeding their own population (b) He believes in the economy of words.
properly. (c) He believes in restraining his thoughts.
(d) He has hesitancy in his speech.
19. With reference to the above passage, which of
the following statements is/are true? 21. The most appropriate reason for the author to
1. Consuming regionally grown food and not be spared many a mishap is that
depending on long travelled food is a part (a) he hardly utters or writes a thoughtless
of eco-friendly behaviour. word.
2. Food processing industry puts a burden on (b) he is a man of immense patience.
our natural resources. (c) he believes that he is a spiritual person.
Select the correct answer using the code given (d) he is a votary of truth.
below:
(a) 1 only 22. For the author, silence is necessary in order to
(b) 2 only surmount
(c) Both 1 and 2 (a) constitutional shyness.
(d) Neither 1 nor 2 (b) hesitancy in speech.
(c) suppression of thoughts.
Passage — 6 (d) tendency to overstate.

I must say that beyond occasionally exposing me to Directions for the following 8 (eight) items:
laughter, my constitutional shyness has been of no Read the following seven passages and answer the
disadvantage whatever. In fact I can see that, on the items that follow. Your answers to these items should
contrary, it has been all to my advantage. My hesitancy be based on the passages only.
in speech, which was once an annoyance, is now a
pleasure. Its greatest benefit has been that it has taught Passage — 1
me the economy of words. I have naturally formed
the habit of restraining my thoughts. And I can now The richer States have a responsibility to cut down
give myself the certificate that a thoughtless word carbon emissions and promote clean energy
hardly ever escapes my tongue or pen. I do not investments. These are the States that got electricity,
recollect ever having had to regret anything in my grew faster and now have high per capita income,

44
COMPREHENSION

making them capable of sharing India’s burden of security. In other words, to strengthen his natural right
becoming eco-friendly. Delhi, for example, can help to exist and work without injury to himself or others.
by generating its own clean electricity using solar The object of government is not to change men from
rooftop panels or even help poor States finance their rational beings into beasts or puppets. It should enable
clean energy projects. It is no secret that State them to develop their minds and bodies in security, and
Electricity Boards, which control 95% of the distribution to employ their reason unshackled.
network, are neck-deep in losses. These losses further
discourage State utilities from adopting renewable 25. Which among the following is the most logical
energy as it is more expensive than fossil fuels. and rational inference that can be made from
the above passage?
23. Which among the following is the most logical (a) The true aim of government is to secure
and rational assumption that can be made the citizens their social and political
from the above passage? freedom.
(a) The richer States must lead in the (b) The primary concern of government is to
production and adoption of renewable provide absolute social security to all its
energy. citizens.
(b) The poor States always have to depend (c) The best government is the one that allows
on rich States for electricity. the citizens to enjoy absolute liberty in all
(c) The State Electricity Boards can improve matters of life.
their finances by undertaking clean energy (d) The best government is the one that
projects. provides absolute physical security to the
(d) The high economic disparity between the people of the country.
rich and poor States is the major cause of
high carbon emissions in India.
Passage —4
Passage —2
Our municipal corporations are understaffed. The issue
Set against a rural backdrop, ‘Stench of kerosene’ is of skills and competencies of the staff poses an even
the story of a couple, Guleri and Manak, who have greater challenge. Urban services delivery and
been happily married for several years but do not have infrastructure are complex to plan and execute. They
a child. Manak’s mother is desperate to have a require a high degree of specialization and
grandchild to carry on the family name. Hence, she professionalism. The current framework within which
gets Manak remarried in Guleri’s absence. Manak, who municipal employees, including senior management, are
acts as a reluctant but passive spectator, is meanwhile, recruited does not adequately factor in the technical
informed by a friend that Guleri, on hearing about her and managerial competencies required. Cadre and
husband’s second marriage, poured kerosene on her recruitment rules only specify the bare minimum in
clothes and set fire to them. Manak is heartbroken academic qualifications. There is no mention of
and begins to live as if he were a dead man. When his managerial or technical competencies, or of relevant
second wife delivers a son, Manak stares at the child work experience. This is the case with most municipal
for a long time and blurts out, “Take him away! He corporations. They also suffer from weak organisation
stinks of kerosene.” design and structure.

24. This is a sensitive issue-based story which tries 26. Which among the following is the most logical
to sensitise the readers about and rational assumption that can be made
(a) Male chauvinism and infidelity from the above passage?
(b) Love and betrayal (a) The task of providing urban services is a
(c) Lack of legal safeguards for women complex issue which requires the
(d) Influence of patriarchal mindset organisational expansion of municipal
bodies all over the country.
Passage — 3 (b) Our cities can provide better quality of life
The ultimate aim of government is not to rule or control if our local government bodies have
by fear, nor to demand obedience, but conversely, to adequate staff with required skills and
free every man from fear, that he may live in all possible competencies.

45
COMPREHENSION

(c) Lack of skilled staff is due to the absence (b) The paper-based system of payments is
of institutions which offer the requisite more efficient than electronic payment in
skills in city management. the present scenario.
(d) Our country is not taking advantage of the (c) The goal of electronic wage payments was
demographic dividend to manage the not to eliminate mediation by village
problems associated with rapid leaders.
urbanization. (d) It is essential to provide financial literacy
to the rural poor.

Passage - 5 Passage -7
Individuals, groups and leaders who promote human
Flamingos in large flocks in the wild are social and development operate under strong institutional,
extremely loyal. They perform group mating dances. structural and political constraints that affect policy
Parents are very fond of their chicks, gathering them options. But experience suggests broad principles for
into crèches for protection while both males and shaping an appropriate agenda for human development.
females fly off to search for food. One important finding from several decades of human
development experience is that focusing exclusively
27. Which among the following is the most logical on economic growth is problematic. While we have
corollary to the above passage? good knowledge about how to advance health and
(a) Mass nesting in all species of birds is education, the causes of growth are much less certain
essential to ensure complete survival of and growth is often elusive. Further, an unbalanced
their offspring. emphasis on growth is often associated with negative
(b) Only birds have the capacity to develop environmental consequences and adverse distributional
social behaviour and thus can do mass effects. The experience of China, with its impressive
nesting to raise their chicks in safety. growth record, reflects these broader concerns and
(c) Social behaviour in some species of birds underlines the importance of balanced approaches that
increases the odds of survival in an unsafe emphasize investments in the non-income aspects of
world. human development.
(d) All species of birds set up crèches for their
chicks to teach them social behaviour and 29. With reference to the above passage, consider
loyalty. the following statements:
1. In developing countries, a strong
Passage – 6 institutional framework is the only
requirement for human development and
Vast numbers of Indian citizens without bank accounts policy options.
live in rural areas, are financially and functionally 2. Human development and economic
illiterate, and have little experience with technology. A growth are not always positively inter-
research study was conducted in a particular area in related.
which electronic wage payments in Mahatma Gandhi 3. Focusing only on human development
National Rural Employment Guarantee Scheme should be the goal of economic growth.
(MGNREGS) are meant to go directly to the poor. It Which of the above statements is/are correct?
was observed that recipients often assume that the (a) 1 only
village leader needs to mediate the process, as was (b) 2 and 3 only
the case under the previous paper-based system. (c) 2 only
Among households under this research study area who (d) 1, 2 and 3
claimed to have at least one bank account, over a third
reported still receiving MGNREGS wages in cash 30. With reference to the above passage, the
directly from a village leader. following assumptions have been made:
1. Higher economic growth is essential to
28. What is the most logical, rational and crucial ensure reduction in economic disparity.
message that is implied in the above passage? 2. Environmental degradation is sometimes
(a) MGNREGS should be extended only to a consequence of economic growth.
those who have a bank account.

46
COMPREHENSION

Which of the above is/are valid assumption/ economy to develop even faster and also to spread the
assumptions? benefits of this growth more widely than has been done
(a) 1 only thus far. Before going into details of the kinds of micro-
(b) 2 only structural changes that we need to conceptualize and
(c) Both 1 and 2 then proceed to implement, it is worthwhile elaborating
(d) Neither 1 nor 2 on the idea of inclusive growth that constitutes the
defining concept behind this Government’s various
31. The mangroves can shed tons of leaves per acre economic policies and decisions. A nation interested in
every year; fungi and bacteria break down this inclusive growth views the same growth differently
leaf litter and consume it, they then are depending on whether the gains of the growth are
consumed by tiny worms and crustaceans, which heaped primarily on a small segment or shared widely
in turn feed small fish, which feed larger fish by the population. The latter is cause for celebration
and birds and crocodiles. but not the former. In other words, growth must not be
Which among the following is the most logical treated as an end in itself but as an instrument for
inference of the above statement? spreading prosperity to all. India’s own past experience
(a) Coastal areas cannot have food chains and the experience of other nations suggests that
without mangroves. growth is necessary for eradicating poverty but it is
(b) Mangroves are an essential component of not a sufficient condition. In other words, policies for
all marine ecosystems. promoting growth need to be complemented with
(c) Mangroves have a crucial role in some of policies to ensure that more and more people join in
the coastal food chains. the growth process and, further, that there are
(d) The composition of marine flora and fauna mechanisms in place to redistribute some of the gains
is largely determined by mangroves. to those who are unable to partake in the market
process and, hence, get left behind.
32. “By liberty I mean the eager maintenance of
that atmosphere in which men have the A simple way of giving this idea of inclusive growth a
opportunity to be their best selves.” sharper form is to measure a nation’s progress in terms
Which one of the following expresses the view of the progress of its poorest segment, for instance the
implied in the above statement? bottom 20 per cent of the population. One could
(a) Liberty is the absence of restraint on measure the per capita income of the bottom quintile
human action. of the population and also calculate the growth rate of
(b) Liberty is what law permits people to income; and evaluate our economic success in terms
perform of these measures that pertain to the poorest segment.
(c) Liberty is the ability to do what one This approach is attractive because it does not ignore
desires. growth like some of the older heterodox criteria did. It
(d) Liberty is the maintenance of conditions simply looks at the growth of income of the poorest
for the growth of human personality. sections of the population. It also ensures that those
who are outside of the bottom quintile do not get ignored.
If that were done, then those people would in all
2014 likelihood drop down into the bottom quintile and so
would automatically become a direct target of our
policies. Hence the criterion being suggested here is a
Directions for the following 5 (five) items: statistical summing up of the idea of inclusive growth,
Read the following two passages and answer the items which, in turn, leads to two corollaries to wish that
that follow each passage. Your answers to these items India must strive to achieve high growth and that we
should be based on the passages only. must work to ensure that the weakest segments benefit
from the growth.
Passage–1
1. The author’s central focus is on
In recent times, India has grown fast not only (a) applauding India’s economic growth not
compared to its own past but also in comparison with only against its own past performance, but
other nations. But there cannot be any room for against other nations.
complacency because it is possible for the Indian (b) emphasizing the need for economic growth

47
COMPREHENSION

which is the sole determinant of a for petrol pricing is announced every fortnight
country’s prosperity. or month, by
(c) emphasizing inclusive growth where gains 1.promoting its sales.
of growth are shared widely by the 2.undertaking innovation.
population. 3.cutting costs.
(d) emphasizing high growth. 4.selling its equity shares at higher prices.
Which of the statements given above is/are I
2. The author supports policies which will help correct?
(a) develop economic growth. (a) 1 only
(b) better distribution of incomes irrespective (b) 2 and 3
of rate of growth. (c) 3 and 4
(c) develop economic growth and redistribute (d) 1, 2 and 4
economic gains to those getting left behind.
(d) put an emphasis on the development of 5. Consider the following statements; According
the poorest segments of society. to the passage, private oil companies re-enter
the oil producing market if
3. Consider the following statements: 1. a transparent rule-based petrol pricing
According to the author, India’s economy has exists.
grown but there is no room for complacency as 2. there is no government interference in the
1. growth eradicates poverty. oil producing market.
2. growth has resulted in prosperity for all. 3. subsidies are given by the government.
Which of the statements given above is/are 4. regulations of anti-trust are removed.
correct? Which of the statements given above are
(a) 1 only correct?
(b) 2 only (a) l and 2
(c) Both 1 and 2 (b) 2 and 3
(d) Neither 1 nor 2 (c) 3 and 4
(d) 2 and 4
Passage – 2
Directions for the following 6 (six) items:
It is easy, for the government to control State-owned Read the following two passages and answer the items
companies through nods and winks. So what really that follow each passage. Your answers to these items
needs to be done as a first step is to ‘put petrol pricing should be based on the passages only.
on a transparent formula — if the price of crude is x
and the exchange rate y, then every month or fortnight, Passage –1
the government announces a maximum price of petrol, Climate change poses potentially devastating
which anybody can work out from the x and the y. effects on India’s agriculture. While the overall
The rule has to be worked out to make, sure that the parameters of climate change are increasingly accepted
oil-marketing companies can, in general, cover their — a 1°C average temperature increase over the next
costs. This will mean that if one company can innovate 30 years, sea level rise of less than 10 cm in the same
and cut costs, it will make greater profits. Hence, firms period, and regional monsoon variations and
will be more prone to innovate and be efficient under corresponding droughts — the impacts in India are likely
this system. Once the rule is announced, there should to be quite site and crop specific. Some crops may
be no interference by the government. If this is done respond favourably to the changing conditions, others
for a while, private companies will re-enter this market. may not. This emphasizes the need to promote
And once a sufficient number of them are in the fray, agricultural research and create maximum flexibility
we can remove the rule-based pricing and leave it truly in the system to permit adaptations.
to the market (subject to, of course, the usual The key ingredient for “drought proofing” is the
regulations of anti-trust and other competition laws). managed recharge of aquifers. To ensure continued
yields of important staple crops (e.g. wheat), it may
4. Consider the following statements: also be necessary to shift the locations where these
According to the passage, an oil company can crops are grown, in response to temperature changes
make greater profits, if a transparent formula as well as to water availability. The latter will be a key

48
COMPREHENSION

factor in making long term investment decisions. technological solutions for harnessing these resources.
For example, water runoff from the Himalayas is
One particular trajectory for carrying out stringent
predicted to increase over the next 30 years as glaciers
mitigation of greenhouse gas emissions assessed by
melt, but then decline substantially thereafter. It will
the Intergovernmental Panel on Climate Change
be critical to provide incentives to plan for these large-
(IPCC) clearly shows the need for ensuring that global
scale shifts in agro-ecological conditions.
emissions of greenhouse gases peak no later than 2015
India needs to make long term investment in
and reduce rapidly thereafter. The cost associated with
research and development in agriculture. India is likely
such a trajectory is truly modest and would amount, in
to experience changed weather patterns in future.
the estimation of IPCC, to not more than 3 percent of
the global GDP in 2030. In other words, the level of
6. Consider the following statements:
prosperity that the world would have reached without
Climate change may force the shifting of
mitigation would at worst be postponed by a few months
locations of the existing crops due to
or a year at the most. This is clearly not a very high
1. melting of glaciers.
price to pay for protecting hundreds of millions of people
2. water availability and temperature
from the worst risks associated with climate change.
suitability at other locations.
Any such effort, however, would require lifestyles to
3. poor productivity of crops.
change appropriately also. Mitigation of greenhouse
4. wider adaptability of crop plants.
gas emissions is not a mere technological fix, and clearly
Which of the statements given above are
requires changes in lifestyles and transformation of a
correct?
country’s economic structure, whereby effective
(a) 1, 2 and 3
reduction in emissions is brought about, such as through
(b) 2 and 3 only
the consumption of much lower quantities of animal
(c) 1 and 4 only
protein. The Food and Agriculture Organization (FAO)
(d) 1, 2, 3 and 4
has determined that the emissions from the livestock
sector amount to 18 percent of the total. The reduction
7. According to the passage, why is it important to
of emissions from this source is entirely in the hands
promote agricultural research in India?
of human beings, who have never questioned the
(a) To predict variations in monsoon patterns
impacts that their dietary habits of consuming more
and to manage water resources
and more animal protein are bringing about. Mitigation
(b) To make long term investment decisions
overall has huge co-benefits, such as lower air pollution
for economic growth.
and health benefits, higher energy security and greater
(c) To facilitate wider adaptability of crops
employment.
(d) To predict drought conditions and to
recharge aquifers
8. According to the passage, which of the following
Passage —2 would help in the mitigation of greenhouse
gases?
It is essential that we mitigate the emissions of
1. Reducing the consumption of meat
greenhouse gases and thus avoid some of the worst
2. Rapid economic liberalization
impacts of climate change that would take place in
3. Reducing the consumerism
coming years and decades. Mitigation would require a
4. Modern management practices of
major shift in the way we produce and consume energy.
livestock
A shift away from overwhelming dependence on fossil
Select the correct answer using the code given
fuels is now long overdue, but unfortunately,
below :
technological development has been slow and
(a) 1, 2 and 3
inadequate largely because government policies have
(b) 2, 3 and 4
not promoted investments in research and development,
(c) 1 and 3 only
myopically as a result of relatively low prices of oil. It
(d) 2 and 4 only
is now, therefore, imperative for a country like India
treating the opportunity of harnessing renewable energy
9. Why do we continue to depend• on the fossil
on a large scale as a national imperative. This country
fuels heavily?
is extremely well endowed with solar, wind and
1. Inadequate technological development
biomass sources of energy. Where we have lagged,
2. Inadequate funds for research and
unfortunately, is in our ability to develop and to create

49
COMPREHENSION

development conserving all the representative systems.


3. Inadequate availability of alternative
sources of energy Further, it needs to be emphasized that the
Select the correct answer using the code given endemics with restricted distribution, and most often
below: with specialized habitat requirements, are among the
(a) 1 only most vulnerable elements. In this respect the Himalayan
(b) 2 and 3 only biodiversity hotspot, with rich endemic diversity, is
(c) 1 and 3 only vulnerable to climate change. The threats include
(d) 1, 2 and 3 possible loss of genetic resources and species, habitats
and concomitantly a decrease in ecosystem services.
10. According to the passage, how does the Therefore, conservation of endemic elements in
mitigation of greenhouse gases help us? representative ecosystems/habitats assumes a great
1. Reduces expenditure on public health significance while drawing conservation plans for the
2. Reduces dependence on livestock region.
3. Reduces energy requirements
4. Reduces rate of global climate change Towards achieving the above, we will have to shift
Select the correct answer using the code given toward contemporary conservation approaches, which
below : include a paradigm of landscape level interconnectivity
(a) 1, 2 and 3 between protected area systems. The concept
(b) 1, 3 and 4 advocates a shift from the species-habitat focus to an
(c) 2, 3 and 4 inclusive focus on expanding the biogeographic range
(d) 1 and 4 only so that natural adjustments to climate change can
proceed without being restrictive.
11. What is the essential message of the passage?
(a) We continue to depend on fossil fuels 12. Consider the following statements:
heavily According to the passage, the adverse impact
(b) Mitigation of the greenhouse gases is of climate change on an ecosystem can be a
imperative 1. permanent disappearance of some of its
(c) We must invest in research and flora and fauna.
development 2. permanent disappearance of ecosystem
(d) People must change their lifestyle itself.
Which of the statements given above is/are
Directions for the following 8 (eight) items: correct?
Read the following two passages and answer the items (a) 1 only
that follow each passage. Your answers to these items (b) 2 only
should be based on the passages only. (c) Both 1 and 2
(d) Neither 1 nor 2
Passage - 1
The Himalayan ecosystem is highly vulnerable to 13. Which one of the following statements best
damage, both due to geological reasons and on account implies the need to shift toward contemporary
of the stress caused by increased pressure of conservation approach?
population, exploitation of natural resources and other (a) Exploitation of natural resources causes
related challenges. These aspects may be exacerbated a stress on the Himalayan ecosystem.
due to the impact of climate change. It is possible that (b) Climate change alters precipitation
climate change may adversely impact the Himalayan patterns, causes episodes of drought and
ecosystem through increased temperature, altered biotic interference.
precipitation patterns, episodes of drought and biotic (c) The rich hiodiversity, including endemic
influences. This would not only impact the very diversity, makes the Himalayan region a
sustenance of the indigenous communities in uplands biodiversity hotspot.
but also the life of downstream dwellers across the (d) The Himalayan biogeographic region
country and beyond. Therefore, there is an urgent need should be enabled to adapt to climate
for giving special attention to sustain the Himalayan change smoothly.
ecosystem. This would require conscious efforts for

50
COMPREHENSION

14. What is the most important message conveyed in the role of the State, fiscal reforms should be
by the passage? undertaken to have generally low levels of taxation
(a) Endemism is a characteristic feature of and government expenditure should be kept to the
Himalayan region. minimum to abide by the principle of fiscal prudence.
(b) Conservation efforts should emphasize on All these are policy actions on the domestic front and
biogeographic ranges rather than on some are not directly related to the core items of the
species or habitats. globalization agenda, namely free international flow of
(c) Climate change has adverse impact on the goods and finance.
Himalayan ecosystem.
(d) Without Himalayan ecosystem, the life of 16. According to the passage, under the
the communities of uplands and globalization, government interventions are
downstreams will have no sustenance. viewed as processes leading to
(a) distortions and inefficiency in the economy.
15. With reference to the passage, the following (b) optimum use of resources.
assumptions have been made: (c) more profitability to industries.
1. To maintain natural ecosystems, (d) free play of market forces with regard to
exploitation of natural resources should be industries.
completely avoided.
2. Not only anthropogenic but also natural 17. According to the passage, the basic philosophy
reasons can adversely affect ecosystems. of globalization is to
3. Loss of endemic diversity leads to the (a) give absolute freedom to producers to
extinction of ecosystems. determine prices and production.
Which of the above assumptions is/are correct? (b) give freedom to producers to evolve
(a) 1 and 2 distribution patterns.
(b) 2 only (c) give absolute freedom to markets to
(c) 2 and 3 determine prices, production and
(d) 3 only employment.
(d) give freedom to producers to import and
Passage 2 export.
It is often forgotten that globalization is not only
about policies on international economic relationships 18. According to the passage, which of the following
and transactions, but has equally to do with domestic is/are necessary for ensuring globalization?
policies of a nation. Policy changes necessitated by 1. Privatization of public enterprises
meeting the internationally set conditions (by WTO 2. Expansionary policy of public expenditure
etc.) of free trade and investment flows obviously affect 3. Free play of market forces to determine
domestic producers and investors. But the basic wages and employment
philosophy underlying globalization emphasizes absolute 4. Privatization of social services like
freedom to markets to determine prices and production education and health
and distribution patterns, and view government Select the correct answer using the code given
interventions as processes that create distortions and below:
bring in inefficiency. Thus, public enterprises have to (a) 1 only
be privatized through disinvestments and sales; sectors (b) 2 and 3 only
and activities hitherto reserved for the public sector (c) 1, 3 and 4
have to be opened to the private sector. This logic (d) 2, 3 and 4
extends to the social services like education and health.
Any restrictions on the adjustments in workforce by 19. According to the passage, in the process of
way of retrenchment of workers should also be globalization the State should have
removed and exit should be made easier by removing (a) expanding role.
any restrictions on closures. Employment and wages (b) reducing role.
should be governed by free play of market forces, as (c) statutory role.
any measure to regulate them can discourage (d) none of the above roles.
investment and also create inefficiency in production.
Above all, in line with the overall philosophy of reduction Directions for the following 7 (seven) items:

51
COMPREHENSION

Read the following two passages and answer the items 4.ignores resultant gross inequity.
that follow each passage. Your answers to these items Which of the statements given above is/are
should be based on the passages only. correct ?
(a) 1 only
Passage -1 (b) 2 and 3
Many nations now place their faith in capitalism (c) 1 and 4
and governments choose it as the strategy to create (d) 4 Only
wealth for their people. The spectacular economic
growth seen in Brazil, China and India after the Passage–2
liberalisation of their economies is proof of its enormous
potential and success. However, the global banking Net profits are only 22% of their total assets for
crisis and the economic recession have left many central public sector undertakings, lower than for the
bewildered. The debates tend to focus on free market private corporate sector. While the public sector or
operations and forces, their efficiency and their ability the State-led entrepreneurship played an important role
for self correction. Issues of justice, integrity and in triggering India’s industrialization, our evolving
honesty are rarely elaborated to highlight the failure of development needs, comparatively less-than-
the global banking system. The apologists of the system satisfactory performance of the public sector
continue to justify the success of capitalism and argue enterprises, the maturing of our private sector, a much
that the recent crisis was a blip. larger social base now available for expanding
Their arguments betray an ideological bias with entrepreneurship and the growing institutional
the assumptions that an unregulated market is fair and capabilities to enforce competition policies would
competent, and that the exercise of private greed will suggest that the time has come to review the role of
be in the larger public interest. public sector.
Few recognize the bidirectional relationship What should the portfolio composition of the
between capitalism and greed; that each reinforces government be? It should not remain static all times.
the other. Surely, a more honest conceptualisation of The airline industry works well as a purely private affair.
the conflicts of interest among the rich and powerful At the opposite end, rural roads, whose sparse traffic
players who have benefited from the system, their makes tolling unviable, have to be on the balance-sheet
biases and ideology is needed; the focus on the wealth of the State. If the government did not own rural roads,
creation should also highlight the resultant gross inequity. they would not exist. Similarly, public health capital in
our towns and cities will need to come from the public
20. The apologists of the “Free Market System”, sector. Equally, preservation and improvement of forest
according to the passage, believe in cover will have to be a new priority for the public sector
(a) market without control by government assets.
authorities. Take the example of steel. With near-zero tariffs,
(b) market without protection by the India is a globally competitive market for the metal.
government. Indian firms export steel into the global market, which
(c) ability of market to self correct. demonstrates there is no gap in technology. Indian
(d) market for free goods and services. companies are buying up global steel companies, which
shows there is no gap in capital availability. Under these
21. With reference to “ideological bias”, the passage conditions, private ownership works best.
implies that Private ownership is clearly desirable in regulated
(a) free market is fair but not competent. industries, ranging from finance to infrastructure, where
(b) free market is not fair but competent. a government agency performs the function of
(c) free market is fair and competent. regulation and multiple competing firms are located in
(d) free market is neither fair nor biased. the private sector. Here, the simple and clean solution
— government as the umpire and the private sector as
22. “The exercise of private greed will be in the the players is what works best. In many of these
larger public interest” from the passage industries, we have a legacy of government ownership,
1.refers to the false ideology of capitalism. where productivity tends to be lower, fear of
2.underlies the righteous claims of the free bankruptcy is absent, and the risk of asking for money
market. from the tax payer is ever present. There is also the
3.shows the benevolent face of capitalism. conflict of interest between government as an owner

52
COMPREHENSION

and as the regulator. The formulation and in this context.


implementation of competition policy will be more
vigorous and fair if government companies are out of
action.
2013

23. According to the passage, what is/are the Directions for the following 8 (eight) items:
reason/reasons for saying that the time has come Read the following four passages and answer the items
to review the role of public sector? that follow each passage. Your answers to these items
1. Now public sector has lost its relevance should be based on the passages only.
in the industrialization process.
2. Public sector does not perform Passage — 1
satisfactorily.
The subject of democracy has become severely
3. Entrepreneurship in private sector is
muddled because of the way the rhetoric surrounding
expanding.
it has been used in recent years. There is, increasingly,
4. Effective competition policies are available
an oddly confused dichotomy between those who want
now.
to ‘impose’ democracy on countries in the non-Western
Which of the statements given above is/are
world (in these countries’‘own interest’, of course) and
correct in the given context?
those who are opposed to such ‘imposition’ (because
(a) 1 and 3 only
of the respect for the countries’‘own ways’). But the
(b) 2 only
entire language of ‘imposition’, used by both sides, is
(c) 2, 3 and 4 only
extraordinarily inappropriate since it makes the implicit
(d) 1, 2, 3 and 4
assumption that democracy belongs exclusively to the
West, taking it to be a quintessentially ‘Western’ idea
24. According to the passage, rural roads should be
which has originated and flourished only in the West.
in the domain of public sector only. Why?
(a) Rural development work is the domain of But the thesis and the pessimism it generates about
government only. the possibility of democratic practice in the world would
(b) Private sector cannot have monetary gains be extremely hard to justify. There were several
in this. experiments in local democracy in ancient India. Indeed,
(c) Government takes money from tax payers in understanding the roots of democracy in the world,
and hence it is the responsibility of we have to take an interest in the history of people
government only. participation and public reasoning in different parts of
(d) Private sector need not have any social the world. We have to look beyond thinking of
responsibility. democracy only in terms of European and American
evolution. We would fail to understand the pervasive
25. The portfolio composition of the government demands for participatory living, on which Aristotle
refers to spoke with far-reaching insight, if we take democracy
(a) Public sector assets quality. to be a kind of a specialized cultural product of the
(b) Investment in liquid assets. West.
(c) Mix of government investment in different It cannot, of course, be doubted that the institutional
industrial sectors. structure of the contemporary practice of democracy
(d) Buying Return on Investment yielding is largely the product of European and American
capital assets. experience over the last few centuries. This is
extremely important to recognize since these
26. The author prefers government as the umpire developments in institutional formats were immensely
and private sector as players because innovative and ultimately effective. There can be little
(a) Government prescribes norms for a fair doubt that there is a major ‘Western’ achievement here.
play by the private sector.
(b) Government is the ultimate in policy 1. Which of the following is closest to the view of
formulation. democracy as mentioned in the above passage?
(c) Government has no control over private (a) The subject of democracy is a muddle due
sector players. to a desire to portray it as a Western
(d) None of the above statements is correct concept, ‘alien’ to non-Western countries.

53
COMPREHENSION

(b) The language of imposition of democracy often becomes the centre of discussion only after the
is inappropriate. There is, however, a need exposure of a large scam.
to consider this concept in the backdrop
of culture of ‘own ways’ of non-Western
3. According to the passage, which of the following
society.
should be the practice/practices in good
(c) While democracy is not essentially a corporate governance?
Western idea belonging exclusively to the 1. Companies should always comply with
West, the institutional structure of current labour and tax laws of the land.
democratic practices has been their 2. Every company in the country should have
contribution. a government representative as one of the
(d) None of the statements (a), (b) and (c) independent directors on the board to
given above is correct. ensure transparency.
3. The manager of a company should never
2. With reference to the passage, the following invest his personal funds in the company.
assumptions have been made: Select the correct answer using the codes given
1. Many of the non-Western countries are below :
unable to have democracy because they (a) 1 only (b) 2 and 3 only
take democracy to be a specialized (c) 1 and 3 only (d) 1, 2 and 3
cultural product of the West.
2. Western countries are always trying to 4. According to the passage, which of the following
impose democracy on non-Western is/are the major benefit/benefits of good
countries. corporate governance?
Which of the above is/are valid assumption / 1. Good corporate governance leads to
assumptions ? increase in share price of the company.
(a) 1 only (b)2 only 2. A company with good corporate
(c) Both 1 and 2 (d)Neither 1 nor 2 governance always increases its business
turnover rapidly.
Passage —2 3. Good corporate governance is the main
criterion for foreign institutional investors
Corporate governance is based on principles such as
when they decide to buy a company.
conducting the business with all integrity and fairness,
Select the correct answer using the codes given
being transparent with regard to all transactions,
below :
making all the necessary disclosures and decisions,
(a) 1 only (b) 2 and 3 only
complying with all the laws of the land, accountability
and responsibility towards the stakeholders and (c) 1 and 3 only (d) 1, 2 and 3
commitment to conducting business in an ethical
manner. Another point which is highlighted on Passage — 3
corporate governance is the need for those in control Malnutrition most commonly occurs between the ages
to be able to distinguish between what are personal of six months and two years. This happens despite the
and corporate funds while managing a company. child’s food requirements being less than that of an
Fundamentally, there is a level of confidence that is older child. Malnutrition is often attributed to poverty,
associated with a company that is known to have good but it has been found that even in households where
corporate governance. The presence of an active group adults eat adequate quantities of food, more than 50
of independent directors on the board contributes a per cent of children-under-five do not consume enough
great deal towards ensuring confidence in the market. food. The child’s dependence on someone else to feed
Corporate governance is known to be one of the criteria him/her is primarily responsible for the malnutrition.
that foreign institutional investors are increasingly Very often the mother is working and the responsibility
depending on when deciding on which companies to of feeding the young child is left to an older sibling. It
invest in. It is also known to have a positive influence is therefore crucial to increase awareness regarding
on the share price of the company. Having a clean the child’s food needs and how to satisfy them.
image on the corporate governance front could also
make it easier for companies to source capital at more 5. According to the passage, malnutrition in children
reasonable costs. Unfortunately, corporate governance can be reduced

54
COMPREHENSION

(a) if the children have regular intake of food.


(b) after they cross the age of five. 7. The need for policy intervention to mitigate risks
(c) if the food needs of younger children are in agriculture is because
known. (a) farmers are extremely risk-averse.
(d) if the responsibility of feeding younger (b) farmers do not know how to mitigate risks.
children is given to adults. (c) the methods adopted by farmers and
existing risk sharing institutions are not
6. According to the author, poverty is not the main adequate.
cause of malnutrition, but the fact that (d) majority of farmers depend on rain-fed
1. taking care of younger ones is not a agriculture.
priority for working mothers.
2. awareness of nutritional needs is not 8. Which of the following observations emerges
propagated by the Public Health from the above passage ?
authorities. (a) One can identify a single policy that can
Select the correct answer using the code given reduce risk without any side-effect.
below : (b) No single risk-specific policy is sufficient
(a) 1 only (b) 2 only to reduce agricultural risk.
(c) Both 1 and 2 (d) Neither 1 nor 2 (c) Policies which affect risk indirectly can
eliminate it.
Passage — 4 (d) Government’s policy intervention can
mitigate agricultural risk completely.
A number of empirical studies find that farmers are
risk-averse, though only moderately in many cases. 9. Consider the following statements
There is also evidence to show that farmers’ risk (i) A primary group is relatively smaller in
aversion results in cropping patterns and input use size.
designed to reduce risk rather than to maximize income. (ii) Intimacy is an essential characteristic of
Farmers adopt a number of strategies to manage and a primary group.
cope with agricultural risks. These include practices (iii) A family may be an example of a primary
like crop and field diversification, non-farm employment, group.
storage of stocks and strategic migration of family In the light of the above statements, which one
members. There are also institutions ranging from share of the following is true ?
tenancy to kinship, extended family and informal credit (a) All families are primary groups.
agencies. One major obstacle to risk sharing by farmers (b) All primary groups are families.
is that the same type of risks can affect a large number (c) A group of smaller size is always a primary
of farmers in the region. Empirical studies show that group.
the traditional methods are not adequate. Hence there (d) Members of a primary group know each
is a need for policy interventions, especially measures other intimately.
that cut across geographical regions.
Policies may aim at tackling agricultural risks directly Directions for the following 4 (four) items:
or indirectly. Examples of risk-specific policies are crop Read the following passage and answer the four items
insurance, price stabilization and the development of that follow. Your answers to these items should be
varieties resistant to pests and diseases. Policies which based on the passage only.
affect risk indirectly are irrigation, subsidized credit and
access to information. No single risk-specific policy is Passage
sufficient to reduce risk and is without side-effects, Financial markets in India have acquired greater depth
whereas policies not specific to risk influence the and liquidity over the years. Steady reforms since 1991
general situation and affect risks only indirectly. Crop have led to growing linkages and integration of the
insurance, as a policy measure to tackle agricultural Indian economy and its financial system with the global
risk directly, deserves careful consideration in the Indian economy. Weak global economic prospects and
context and in many other developing countries — continuing uncertainties in the international financial
because the majority of farmers depend on rain-fed markets therefore, have had their impact on the
agriculture and in many areas yield variability is the emerging market economies. Sovereign risk concerns,
predominant cause of their income instability. particularly in the Euro area, affected financial markets

55
COMPREHENSION

for the greater part of the year, with the contagion of critical to ensure overall financial stability
Greece’s sovereign debt problem spreading to India because Indian financial system is
and other economies by way of higher-than normal (a) controlled by the Government of India.
levels of volatility. (b) less integrated with banks.
The funding constraints in international financial markets (c) controlled by Reserve Bank of India.
could impact both the availability and cost of foreign (d) dominated by banks.
funding for banks and corporates. Since the Indian
financial system is bank dominated, banks’ ability to 13. Risk and liquidity management assumes more
withstand stress is critical to overall financial stability. importance in the Indian banking system in
Indian banks, however, remain robust, notwithstanding future due to
a decline in capital to risk-weighted assets ratio and a 1. further globalization
rise in non-performing asset levels in the recent past. 2. more consolidation and deregulation of the
Capital adequacy levels remain above the regulatory financial system.
requirements. The financial market infrastructure
3. further diversification of financial system.
continues to function without any major disruption. With
further globalization, consolidation, deregulation, and 4. more financial inclusion in the economy.
diversification of the financial system, the banking Select the correct answer using the code given
business may become more complex and riskier. Issues below :
like risk and liquidity management and enhancing skill (a) l, 2 and 3
therefore assume greater significance. (b) 2, 3 and 4
(c) 1 and 2 only
10. According to the passage, the financial markets (d) 3 and 4 only
in the emerging market economies including
India had the adverse impact in recent years Directions for the following 2 (two) items:
due to Read the ‘following passage and answer the two items
1. weak global economic prospects. that follow. Your answers to these items should be
2. uncertainties in the international financial based on the passage only.
markets.
3. sovereign risk concerns in the Euro area. Passage
4. bad monsoons and the resultant crop loss. Crude mineral oil comes out of the earth as a thick
Select the correct answer using the code given brown or black liquid with a strong smell. It is a complex
below: mixture of many different substances, each with its
(a) 1 and 2 only own individual qualities. Most of them are combinations
(b) l, 2 and 3 of hydrogen and carbon in varying proportions. Such
hydrocarbons are also found in other forms such as
(c) 2 and 3 only
bitumen, asphalt and natural gas. Mineral oil originates
(d) 2, 3 and 4 from the carcasses of tiny animals and from plants
that live in the sea. Over millions of years, these dead
11. The Indian financial markets are affected by creatures form large deposits under the sea-bed; and
global changes mainly due to the ocean currents cover them with a blanket of sand and
(a) increased inflow of remittances from silt. As this mineral hardens, it becomes sedimentary
abroad. rock and effectively shuts out the oxygen, so preventing
(b) enormous increase in the foreign the complete decomposition of the marine deposits
exchange reserves. underneath. The layers of sedimentary rock become
(c) growing global linkages and integration of thicker and heavier. Their pressure produces heat,
the Indian financial markets. which transforms the tiny carcasses into crude oil in a
(d) contagion of Greece’s sovereign debt process that is still going on today.
problem.
14. Mineral oil deposits under the sea do not get
12. According to the passage, in the Indian financial completely decomposed because they
system, banks’ ability to withstand stress is (a) are constantly washed by the ocean
currents

56
COMPREHENSION

(b) become rock and prevent oxygen from (a) 1 only (b) 1 and 2
entering them. (c) 2 and 3 (d) 1, 2 and 3
(c) contain a mixture of hydrogen and carbon.
(d) are carcasses of organisms lying in saline Directions for the following 7 (seven) items:
conditions Read the following two passages and answer the items
that follow each passage. Your answers to these items
15. Sedimentary rock leads to the formation of oil should be based on the passages only.
deposits because
(a) there are no saline conditions below it. Passage — 1
(b) it allows some dissolved oxygen to enter The law in many parts of the world increasingly restricts
the dead organic matter below it. the discharge of agricultural slurry into watercourses.
The simplest and often the most economically sound
(c) weight of overlying sediment layers
practice returns the material to the land as semisolid
causes the production of heat.
manure or as sprayed slurry. This dilutes its
(d) it contains the substances that catalyze concentration in the environment to what might have
the chemical reactions required to change occurred in a more primitive and sustainable type of
dead organis into oil. agriculture and converts pollutant into fertilizer. Soil
microorganisms decompose the organic components
Directions for the following 2 (two) items: of sewage and slurry and most of the mineral nutrients
Read the following passage and answer the two items become available to be absorbed again by the
that follow. Your answers to these items should be vegetation.
based on the passage only.
The excess input of nutrients, both nitrogen and
Passage phosphorus — based, from agricultural runoff (and
Ecological research over the last quarter of the century human sewage) has caused many ‘healthy’ oligotrophic
has established the deleterious effects of habitat lakes (low nutrient concentrations, low plant
fragmentation due to mining, highways and such other productivity with abundant water weeds, and clear
intrusions on forests. When a large block of forests water) to change to eutrophic condition where high
gets fragmene into smaller bits, the edges of all these nutrient inputs lead to high phytoplankton productivity
bits come into contact with human activities resulting (sometimes dominated by bloom-forming toxic species).
in the degradation of the entire forests. Continuity of This makes the water turbid, eliminates large plants
forested landscapes and corridors gets disrupted and, in the worst situations, leads to anoxia and fish
affecting several extinction-prone species of wildlife. kills; so called cultural eutrophication. Thus, important
Habitat fragmention is therefore considered as the most ecosystem services are lost, including the provisioning
serious threat to biodiversity conservation Ad hoc service of wild-caught fish and the cultural services
grants of forest lands to mining companies coupled with associated with recreation.
rampant illegal mining is aggravating this threat.
The process of cultural eutrophication of lakes has been
understood for some time. But only recently did
16. What is the central focus of this passage? scientists notice huge ‘dead zones’ in the oceans near
(a) Illegal mining in forests river outlets, particularly those draining large catchment
(b) Extinction of wildlife areas such as the Mississippi in North America and
(c) Conservation of nature the Yangtze in China. The nutrient-enriched water flows
(d) Disruption of habitat through streams, rivers and lakes, and eventually to
the estuary and ocean where the ecological impact
17. What is the purpose of maintaining the continuity may be huge, killing virtually all invertebrates and fish
of forested landscapes and corridors ? in areas up to 70,000 km2 in extent. More than 150 sea
1. Preservation of biodiversity. areas worldwide are now regularly starved of oxygen
as a result of decomposition of algal blooms, fuelled
2. Management of mineral resources.
particularly by nitrogen from agricultural runoff of
3. Grant of forest lands for human activities. fertilizers and sewage from large cities. Oceanic dead
Select the correct answer using the codes given zones are typically associated with industrialized nations
below: and usually lie off countries that subsidize their

57
COMPREHENSION

agriculture, encouraging farmers to increase (b) 1 and 2 only


productivity and use more fertilizer. (c) 2 and 3 only
(d) 1, 2 and 3
18. According to the passage, why should the
discharge of agricultural slurry into watercourses 22. What is the central theme of this passage?
be restricted ? (a) Appropriate legislation is essential to
1. Losing nutrients in this way is not a good protect the environment.
practice economically. (b) Modern agriculture is responsible for the
2. Watercourses do not contain the destruction of environment.
microorganisms that can decompose (c) Improper waste disposal from agriculur can
organic components of agricultural slurry. destroy the aquatic ecosystems.
3. The discharge may lead to the (d) Use of chemical fertilizers is undesirable
eutrophication of water bodies. in agricu1ture.
Select the correct answer using the codes given
below : Passage — 2
(a) 1 only (b) 2 and 3 only The miseries of the world cannot be cured by physical
(c) 1 and 3 only (d) 1, 2 and 3 help only. Until man’s nature changes, his physical
needs will always arise, and miseries will always be
19. The passage refers to the conversion of “pollutant felt, and no amount of physical help will remove them
to fertilizer”. What is pollutant and what is completely. The only solution of the problem is to make
fertilizer in this context ? mankind pure. Ignorance is the mother of evil and of
(a) Decomposed organic component of slurry all the misery we see. Let men have light, let them be
is pollutant and microorganisms in soil pure and spiritually strong and educated; then alone
constitute fertilizer. will misery cease in the world. We may convert every
(b) Discharged agricultural slurry is pollutant house in the country into a charitable asylum, we may
and decomposed slurry in soil is fertilizer. fill the land with hospitals, but human misery will continue
(c) Sprayed slurry is pollutant and until man’s character changes.
watercourse is fertilizer.
(d) None of the above expressions is correct 23. According to the passage, which of the following
in this context. statements is most likely to be true as the reason
for man’s miseries ?
20. According to the passage, what are the effects (a) The poor economic and social conditions
of indiscriminate use of fertilizers? prevailing in society.
1. Addition of pollutants to the soil and water. (b) The refusal on the part of man to change
2. Destruction of decomposer his character.
microorganisms in soil. (c) The absence of physical and material help
3. Nutrient enrichment of water bodies. from his society.
4. Creation of algal blooms. (d) Ever increasing physical needs due to
Select the correct answer from the codes given changing social structure
below :
(a) 1, 2 and 3 only 24. With reference to the passage, the following
(b) 1, 3 and 4 only assumptions have been made :
(c) 2 and 4 only 1. The author gives primary importance to
(d) 1, 2, 3 and 4 physical and material help in eradicating
human misery.
21. What is/are the characteristics of a water body 2. Charitable homes, hospitals, etc. can
with cultural eutrophication? remove human misery to a great extent.
1. Loss of ecosystem services Which of the assumptions is/are valid ?
2. Loss of flora and fauna (a) 1 only
3. Loss of mineral nutrients (b) 2 only
Select the correct answer using the code given (c) Both 1 and 2
below : (d) Neither 1 nor 2
(a) 1 only

58
COMPREHENSION

2012 (d) All the (a), (b) and (c) given above are
correct in this context.
3. Education is a process in which
Directions for the following 15 (fifteen) items:
(a) students are converted into trained
Read the following three passages and answer
professionals.
the items that follow each passage. Your answers
to these items should be based on the passages (b) opportunities for higher income are
only. generated.
(c) individuals develop self-critical awareness
and independence of thought.
Passage - 1
(d) qualifications for upward mobility are
Education, without a doubt, has an important functional,
acquired.
instrumental and utilitarian dimension. This is revealed
when one asks questions such as ‘what is the purpose
of education ?‘. The. answers, too often, are ‘to Passage —2
acquire qualifications for employment/ upward Chemical pesticides lose their role in sustainable
mobility’, ‘wider/higher (in terms of income) agriculture if the pests evolve resistance. The evolution
opportunities’, and ‘to meet the needs for trained human of pesticide resistance is simply natural selection in
power in diverse fields for national development’. But action. It is almost certain to occur when vast numbers
in its deepest sense education is not instrumentalist. of a genetically variable population are killed. One or
That is to say, it is not to be justified outside of itself a few individuals may be unusually resistant (perhaps
because it leads to the acquisition of formal skills or of because they possess an enzyme that can detoxify
certain desired psychological - social attributes. It must the pesticide). If the pesticide is applied repeatedly,
be respected in itself. Education is thus not a each successive generation of the pest will contain a
commodity to be acquired or possessed and then used, larger proportion of resistant individuals. Pests typically
but a process of inestimable importance to individuals have a high intrinsic rate of reproduction, and so a
and society, although it can and does have enormous few individuals in one generation may give rise to
use value. Education then, is a process of expansion hundreds or thousands in the next, and resistance
and conversion, not in the sense of converting or spreads very rapidly in a population.
turning students into doctors or engineers, but the
widening and turning out of the mind - the creation, This problem was often ignored in the past, even though
sustenance and development of self-critical awareness the first case of DDT (dichlorodiphen-
and independence of thought. It is an inner process of yltrichloroethane) resistance was reported as early as
moral - intellectual development. 1946. There is exponential increase in the numbers of
invertebrates that have evolved resistance and in the
number of pesticides against which resistance has
1. What do you understand by the ‘instrumentalist’
evolved. Resistance has been recorded in every family
view of education?
of arthropod pests (including dipterans such as
(a) Education is functional and utilitarian in
mosquitoes and house flies, as well as beetles, moths,
its purposes.
wasps, fleas, lice and mites) as well as in weeds’ and
(b) Education is meant to fulfill human needs. plant pathogens. Take the Alabama leafworm, a moth
(c) The purpose of education is to train the pest of cotton, as an example. It has developed
human intellect. resistance in one or more regions of the world to aidrin,
(d) Education is meant to achieve moral DDT, dieldrin, endrin, lindane and toxaphene.
development.
If chemical pesticides brought nothing but problems,
2. According to the passage, education must be — if their use was intrinsically and acutely
respected in itself because unsustainable — then they would already have fallen
(a) it helps to acquire qualifications for out of widespread use. This has not happened. Instead,
employment. their rate of production has increased rapidly. The ratio
(b) it helps in upward mobility and acquiring of cost to benefit for the individual agricultural
social status. producer has remained in favour of pesticide use. In
(c) it is an inner process of moral and the USA, insecticides have been estimated to benefit
intellectual development. the agricultural products to the tune of around $5 for
every $1 spent.

59
COMPREHENSION

Moreover, in many poorer countries, the prospect of 7. How do pesticides act as agents for the selection
imminent mass starvation, or of an epidemic disease, of resistant individuals in any pest population?
are so frightening that the social and health costs of 1. It is possible that in a pest population the
using pesticides have to be ignored. In general the use individuals will behave differently due to
of pesticides is justified by objective measures such their genetic makeup.
as ‘lives saved’, ‘economic efficiency of food 2. Pests do possess the ability to detoxify
production’ and ‘total food produced’. In these very the pesticides.
fundamental senses, their use may be described as
3. Evolution of pesticide resistance is equally
sustainable. Jr. practice, sustainability depends on
distributed in pest population.
continually developing new pesticides that keep at least
one step ahead of the pests — pesticides that are less Which of the statements given above is/are
persistent, biodegradable and more accurately targeted correct ?
at the pests. (a) 1 only (b) l and 2 only
(c) 3 only (d) 1, 2 and 3
4. “The evolution of pesticide resistance is natural
selection in action.” What does it actually imply? 8. Why is the use of chemical pesticide. generally
(a) It is very natural for man’ organisms to justified by giving the examp1e of poor and
have pesticide resistance. developing countries?
(b) Pesticide resistance among organisms is 1. Developed countries can afford to dc
a universal phenomenon. away with use of pesticides by adapting
to organic farming, but it is imperative for
(c) Some individuals in any given population
poor and developing countries to use
show resistance after the application of
chemical pesticides.
pesticides.
2. In poor and developing countries, the
(d) None of the statements (a), (b) and (c)
pesticide addresses the problem of
given above is correct.
epidemic diseases of crops and eases the
food problem.
5. With reference to the passage, consider the 3. The social and health costs of pesticide
following statements : use are generally ignored in poor and
1. Use of chemical pesticides has become developing countries.
imperative in all the poor countries of the Which of the statements given above is/are
world. correct ?
2. Chemical pesticides should not have any (a) 1 only (b) 1 and 2 only
role in sustainable agriculture.
(c) 2 only (d) 1, 2 and 3
3. One pest can develop resistance to many
pesticides.
9. What does the passage imply?
Which of the statements given above is/are
correct? (a) Alternative options to chemical pesticides
should be promoted.
(a) 1 and 2 only (b) 3 only
(b) Too much use of chemicals is not good
(c) 1 and 3 only (d) 1,2 and 3
for the ecosystem.
(c) There is no scope for the improvement of
6. Though the problems associated with the use of pesticides and making their use
chemical pesticides is known for a long time, sustainable.
their widespread use has not waned. Why?
(d) Both the statements (a) and (b) above are
(a) Alternatives to chemical pesticides do not correct.
exist at all.
(b) New pesticides are not invented at all.
Passage - 3
(c) Pesticides are biodegradable.
Today’s developing economies, use much less energy
(d) None of the statements (a), (b) and (c) per capita than developed countries such as the United
given above is correct. States did at similar incomes, showing the potential
for lower-carbon growth. Adaptation and mitigation

60
COMPREHENSION

need to be integrated into a climate-smart development Select the correct answer using the codes given
strategy that increases resilience, reduces the threat below:
of further global warming, and improves development (a) 1 only (b) 2, 3 and 4 only
outcomes. Adaptation and mitigation measures can (c) 1 and 4 only (d) 1, 2, 3 and 4
advance development, and prosperity can raise
incomes and foster better institutions. A healthier
11. What does low-carbon growth imply in the
population living in better-built houses and with access
present context?
to bank loans and social security is better equipped to
1. More emphasis on the use of renewable
deal with a changing climate and its consequences.
sources of energy.
Advancing robust, resilient development policies that
promote adaptation is needed today because changes 2. Less emphasis on manufacturing sector
in the climate, already begun, will increase even in the and more emphasis on agriculture sector.
short term. 3. Switching over from monoculture
The spread of economic prosperity has always been practices to mixed farming.
intertwined with adaptation to changing ecological 4. Less demand for goods and services.
conditions. But as growth has altered the environment Select the correct answer using the codes given
and as environmental change has accelerated, below :
sustaining growth and adaptability demands greater (a) 1 only
capacity to understand our environment, generate new (b) 2, 3 and 4 only
adaptive technologies and practices, and diffuse them (c) 1 and 4 only
widely. As economic historians have explained, much
(d) None of the above implies low-carbon
of humankind’s creative potential has been directed at
growth
adapting to the changing world. But adaptation cannot
cope with all the impacts related to climate change,
especially as larger changes unfold in the long term. 12. Which of the following conditions is/are
necessary for sustainable economic growth?
Countries cannot grow out of harming way fast enough
1. Spreading of economic prosperity more.
to match the changing climate. And some growth
strategies, whether driven by the government or the 2. Popularising/spreading of adaptive
market, can also add to vulnerability — particularly if technologies widely.
they overexploit natural resources. Under the Soviet 3. Investing on research in adaptation and
development plan, irrigated cotton cultivation expanded mitigation technologies.
in water-stressed Central Asia and led to the near Select the correct answer using the codes given
disappearance of the Aral Sea, threatening the below :
livelihoods of fishermen, herders and farmers. And (a) 1 only (b) 2 and 3 only
clearing mangroves — the natural coastal buffers (c) 1 and 3 only (d) 1, 2 and 3
against storm surges — to make way for intensive
farming or housing development, increases the physical 13. Which of the following inferences can be made
vulnerability of coastal settlements, whether in Guinea from the passage?
or in Louisiana. 1. Rainfed crops should not be cultivated in
irrigated areas.
10. Which of the following conditions of growth can 2. Farming under water-deficient areas
add to vulnerability? should not be a part of development
1. When the growth occurs due to excessive strategy.
exploitation of mineral resources and Select the correct answer using the codes given
forests. below:
2. When the growth brings about a change (a) 1 only (b) 2 only
in humankind’s creative potential. (c) Both 1 and 2 (d) Neither 1 nor 2
3. When the growth is envisaged only for
providing houses and social security to the
14. Consider the following assumptions
people.
1. Sustainable economic growth demands
4. When the growth occurs due to emphasis the use of creative potential of man.
on farming only.

61
COMPREHENSION

2. Intensive agriculture can lead to ecological access to everywhere on the globe, we might expect a
backlash. relatively small number of successful species to become
3. Spread of economic prosperity can dominant in each biome. The extent to which this
adversely affect the ecology and homogenization can happen naturally is restricted by
environment. the limited powers of dispersal of most species in the
With reference to the passage, which of the face of the physical barriers that exist to dispersal. By
above assumptions is/are valid? virtue of the transport opportunities offered by humans,
(a) 1 only (b) 2 and 3 only these barriers have been breached by an ever-
increasing number of exotic species. The effects of
(c) 1 and 3 only (d) 1,2 and 3
introductions have been to convert a hugely diverse
range of local community compositions into something
15. Which one of the following statements much more homogeneous.
constitutes the central theme of this passage?
It would be wrong, however, to conclude that
(a) Countries with greater economic
introducing species to a region will inevitably cause a
prosperity are better equipped to deal with
decline in species richness there. For example, there
the consequences of climate change.
are numerous species of plants, invertebrates and
(b) Adaptation and mitigation should be vertebrates found in continental Europe but absent from
integrated with development strategies. the British Isles (many because they have so far failed
(c) Rapid economic growth should not be to recolonize after the last glaciations). Their
pursued by both developed and developing introduction would be likely to augment British
economies. biodiversity. The significant detrimental effect noted
(d) Some countries resort to overexploitation above arises where aggressive species provide a novel
of natural resources for the sake of rapid challenge to endemic biotas ill-equipped to deal with
development. them.

Directions for the following 11 (eleven) items: 16. With reference to the passage, which of the
Read the following three passages and answer the items following statements is correct?
that follow each passage. Your answers to these items (a) Introduction of exotic species into new
should be based on the passages only. geographical areas always leads to
Passage —1 reduced biodiversity.
(b) Exotic species introduced by man into
Invasions of exotic species into new geographic areas
new areas have always greatly altered the
sometimes occur naturally and without human agency.
native ecosystems.
However, human actions have increased this trickle to
a flood. Human-caused introductions may occur either (c) Man is the only reason to convert a hugely
accidentally as a consequence of human transport, or diverse range of local community
intentionally but illegally to serve some private purpose compositions into more homogeneous
or legitimately to procure some hoped-for public benefit ones.
by bringing a pest under control, producing new (d) None of the statements (a), (b) and (c) is
agricultural products or providing novel recreational correct in this context.
opportunities. Many introduced species are assimilated
into communities without much obvious effect. 17. Why does man introduce exotic species into new
However, some have been responsible for dramatic geographical areas?
changes to native species and natural communities. 1. To breed exotic species with local
For example, the accidental introduction of the brown varieties.
tree snake Boiga irregularis into Guam, an island in the
2. To increase agricultural productivity.
Pacific, has through nest predation reduced 10 endemic
forest bird species ft the point of extinction. 3. For beautification and landscaping.
Which of the above statements is/are Correct ?
One of the major reasons for the world’s great
(a) 1 only (b) 2 and 3 only
biodiversity is the occurrence of centers of endemism
so that similar habitats in different parts of the world (c) 1 and 3 only (d) 1, 2 and 3
are occupied by different groups of species that happen
to have! evolved there. If every species naturally had

62
COMPREHENSION

18. How is homogenization prevented under natural To deal with these issues we have to pay particular
conditions ? attention to both the content of what can be called
(a) Evolution of groups of species specific to development and to the interpretation of democracy
local habitats. (in particular to the respective roles of voting and of
(b) Presence of oceans and mountain ranges. public reasoning). The assessment of development
(c) Strong adaptation of groups of species to cannot be divorced from the lives that people can lead
local physical and climatic conditions. and the real’ freedom that they enjoy. Development
can scarcely be seen merely in terms of enhancement
(d) All the statements (a), (b) and (c) given
of inanimate objects of convenience, such as a rise in
above are correct in this context.
the GNP (or in personal incomes), or industrialization
— important as they may be as means to the real.
19. How have the human beings influenced the ends. Their value must depend on what they do to the
biodiversity? lives and freedom of the people involved, which must
1. By smuggling live organisms. be central to the idea of development.
2. By building highways. If development is understood in a broader way, with a
3. By making ecosystems sensitive so that focus on human lives, then it becomes immediately clear
new species are not allowed. that the relation between development and democracy
4. By ensuring that new species do not have has to be seen partly in terms of their constitutive
major impact on local species. connection, rather than only through their external links.
Which of the statements given above are Even though the question has often been asked whether
correct? political freedom is “conducive to development”, we
(a) 1 and 2 (b) 2 and 3 must not miss the crucial recognition that political
liberties and democratic rights are among the
(c) 1 and 3 (d) 2 and 4
“constituent components” of development. Their
relevance for development does not have to be
20. What can be the impact of invasion of exotic established indirectly through their contribution to the
species on an ecosystem? growth of GNP.
1. Erosion of endemic species.
2. Change in the species composition of the
21. According to the passage, why is a serious
community of the ecosystem.
tension perceived between democracy and
Select the correct answer using the codes given development by the detractors of democracy?
below : (a) Democracy and development are distinct
(a) 1 only (b) 2 only and separate goals.
(c) Both 1 and 2 (d) Neither 1 nor 2 (b) Economic growth can be promoted
successfully without pursuing a
Passage -2 democratic system of governance.
Most champions of democracy have been rather (c) Non-democratic regimes deliver
reticent in suggesting that democracy would itself economic growth faster and far more
promote development and enhancement of social successfully than democratic ones.
welfare — they have tended to see them as good but (d) All the statements (a), (b) and (c) given
distinctly separate and largely independent goals. The above are correct in this context.
detractors of democracy, on the other hand, seemed
to have been quite willing to express their diagnosis of 22. According to the passage, what should be the
what they see as serious tensions between democracy ultimate assessment/aim/view of development?
and development. The theorists of the practical split (a) Rise in the per capita income and industrial
— “Make up your mind: do you want democracy, or growth rates.
instead, do you want development ?” — often came,
(b) Improvement in the Human Development
at least to start with, from East Asian countries, and
Index and GNP.
their voice grew in influence as several of these
countries were immensely successful — through the (c) Rise in the savings and consumption
1970s and 1980s and even later — in promoting trends.
economic growth without pursuing democracy. (d) Extent of real freedom that citizens enjoy.

63
COMPREHENSION

(a) 1 only (b) 2 only


23. What does a “constitutive” connection between (c) Both 1 and 2 (d) Neither 1 nor 2
democracy and development imply ?
(a) The relation between them has to be seen 25. According to the passage, how does a foreign
through external links. investor dominate the relevant domestic market?
(b) Political and civil rights only can lead to 1. Multinational companies get accustomed
economic development. to domestic laws.
(c) Political liberties and democratic rights are 2. Foreign companies establish joint ventures
essential elements of development. with domestic companies.
(d) None of the statements (a), (b) and (c) 3. Affiliates in a particular market sector lose
given above is correct. in this context. their independence as their parent
companies overseas merge.
Passage -3 4. Foreign companies lower the cost of their
The need for Competition Law becomes more evident products as compared to that of products
when foreign direct investment (FDI) is liberalised. The of domestic companies.
impact of FIJI is not always pro-competitive. Very often Which of the statements given above are
FDI takes the form of a foreign corporation acquiring correct?
a domestic enterprise or establishing a joint venture (a) 1 and 2 only (b) 2 and 3 only
with one. By making such an acquisition the foreign (c) 1, 2 and 3 only (d) 1, 2, 3 and 4
investor may substantially lessen competition and gain
a dominant position in the relevant market, thus charging 26. What is the inference from this passage :
higher prices. Another scenario is where the affiliates (a) Foreign investors and multinational
of two separate multinational companies (MNCs) have companies always dominate the domestic
been established in competition with one another in a market.
particular developing economy, following the
(b) It is not in the best interests of the
liberalisation of FDI. Subsequently, the parent
domestic economy to allow mergers of
companies overseas merge. With the affiliates no longer
companies.
remaining independent, competition in the host country
may be virtually eliminated and the prices of the (c) With competition law, it is easy to ensure
products may be artificially inflated. Most of these a level playing field between domestic and
adverse consequences of mergers and acquisitions by foreign firms.
MNCs can be avoided if an effective competition law (d) For countries with open economy, Foreign
is in place. Also, an economy that has implemented an Direct Investment is essential for growth.
effective competition law is in a better position to attract
FDI than one that has not. This is not just because Directions for the following 6 (six) items :
most MNCs are expected to be accustomed to the
Read the following two passages and answer the items
operation of such a law in their home countries and
that follow each passage. Your answers to these items
know how to deal with such concerns but also that
should be based on the passages only.
MNCs expect competition authorities to ensure a level
playing field between domestic and foreign firms. Passage — 1
The poor especially in. market economies, need the
strength that collectivities offer for creating more
24. With reference to the passage, consider the
economic, social and political space for themselves,
following statements :
for enhancing their socio-economic well-being and
1. It is desirable that the impact of Foreign
voice, and as a protection against free market
Direct Investment should be pro-
individualism. It has been argued that a group, approach
competitive.
to fanning, especially in the form of bottom up
2. The entry of foreign investors invariably agricultural production collectivities, offers substantial
leads to the inflated prices in domestic scope for poverty alleviation and empowering the poor
markets. as well as enhancing agricultural productivity. To realise
Which of the statements given above is/are this potential, however, the groups would need to be
correct ? voluntary in nature, small in size, participative in

64
COMPREHENSION

decision making and equitable in work sharing and ‘individuals’ which is a staple to liberal discourse, but
benefit distribution. There are many notable examples as communities or groups: Individuals are getting
of such collectivities to be found in varied contexts, involved in the public sphere not as ‘atomized’
such as in the transition economies. All of them bear individuals but as members of primordial communities
witness to the possibility of successful cooperation drawn on religious or caste identity. Community-identity
under given conditions. And although the gender impact seems to be the governing force. It is not therefore
of the family cooperatives in the transition economies surprising that the so-called peripheral groups continue
are uncertain, the Indian examples women-only groups to maintain their identities with reference to the social
fanning offer considerable potential for benefiting groups (caste, religion or sect) to which they belong
women. while getting involved in the political processes despite
the fact that their political goals remain more or less
identical. By helping to articulate the political voice of
27. Agricultural collectivities such as group based
the marginalized, democracy in India has led to ‘a
farming can provide the rural poor
loosening of social strictures’ and empowered the
1. empowerment.
peripherals to be confident of their ability to improve
2. increased agricultural productivity the socio-economic conditions in which they are placed.
3. safeguard against exploitative markets. This is a significant political process that had led to a
4. surplus production of agricultural silent revolution through a meaningful transfer of power
commodities. from the upper caste elites to various subaltern groups
Select the correct answer using the codes given within the democratic framework of public governance.
below:
(a) 1, 2, 3 and 4 30. According to the passage, what does “deepening
(b) 1, 2 and 3 only of democracy” mean in the Western context?
(c) 2 and 4 only (a) Consolidation of group and class identities.
(d) 1, 3 and 4 only (b) Democratization translated as greater
involvement of people.
28. What does the author imply by “gender impact”? (c) Democratization as greater involvement
(a) Women are doubtful participants in of ‘atomized’ individuals in the public
cooperatives. sphere.
(b) Family cooperatives may not include (d) None of the statements (a), (b) and (c)
women. given above is correct in this context.
(c) Women benefiting from group farming.
(d) Women’s role in transition economies is 31. Greater democratization in India has not
highly restrictive. necessarily led to
(a) the dilution of caste and communal
identities in the public sphere.
29. Consider the following assumptions:
(b) irrelevance of community identity as a
1. It is imperative for transition economies
governing force in Indian politics.
to have agricultural collectivities.
(c) marginalization of elite groups in society.
2. Agricultural productivity can be increased
by group approach to farming. (d) relative unimportance of hereditary
identities over class identities.
With reference to the above passage, which of
these assumption is/are valid ?
(a) 1 only (b) 2 only 32. What is the “silent revolution” that has occurred
(c) Both 1 and 2 (d) Neither I nor 2 in the Indian democratic process?
(a) Irrelevance of caste and class hierarchies
in political processes.
Passage —2
(b) Loosening of social strictures in voting
In a typical Western liberal context, deepening of behaviour and patterns.
democracy invariably leads to consolidation of ‘liberal
(c) Social change through transfer of power
values’. In the Indian context, democratization is
from upper caste elites to subaltern
translated into greater involvement of people not as
groups.

65
COMPREHENSION

(d) All the statements (a), (b) and (c) given Which of the statements given: above are
above are correct in this context. correct?
(a) 1 and 2 only (b) 2 and 3 only
(c) 1 and 4 only (d) 1, 2, 3 and 4
2011
2. According to the passage, the strategy of
Directions for the following 8 (eight) items: inclusive growth can be effected by focusing
Read each of the following two passages and answer on
the items that follow. Your answers to these items (a) Meeting all the needs of every citizen in
should be based on the passages only. the country.
(b) Increasing the regulations over the manu-
Passage-1 facturing sector.
For achieving inclusive growth there is a critical (c) Controlling the distribution of manufac-
need to rethink the role of the State. The early debate tured goods.
among economists, about the size of the Government (d) Delivery of the basic services to the de-
can be misleading. The need of the hour is to have an prived sections of the society.
enabling Government. India is too large and complex
a nation for the State to be able to deliver all that is 3. What constitutes an enabling Government?
needed. Asking the Government to produce all the 1. A large bureaucracy.
essential goods, create all the necessary jobs, and keep 2. Implementation of welfare programmes
a curb on the prices of all goods is to lead to a large through representatives.
cumbersome bureaucracy and widespread corruption. 3. Creating an ethos that helps individual
The aim must be to stay with the objective of enterprise.
inclusive growth that was laid down by the founding 4. Providing resources to those who are un-
fathers of the nation and also to take a more modern derprivileged.
view of what the State can realistically deliver. 5. Offering direct help to the poor regarding
This is what leads to the idea of an enabling State, basic services.
that is, a Government that does not try to directly deliver Select the correct answer from the codes given
to the citizens everything that they need. Instead, it below:
(1) creates an enabling ethos for the market so that (a) 1, 2 and 3 only (b) 4 and 5 only
individual enterprise can flourish and citizens can, for (c) 3, 4 and 5 only (d)1, 2, 3, 4 and 5
the most part, provide for the needs of one another,
and (2) steps in to help those who do not manage to do 4. Why is the State unable to deliver “all that is
well for themselves, for there will always be individuals, needed” ?
no matter what the system, who need support and help. 1. It does not have sufficient bureaucracy.
Hence we need a Government that, when it comes to 2. It does not promote inclusive growth.
the market, sets effective incentive-compatible rules Select the correct’ answer from the codes given
and remains on the sidelines with minimal interference, below :
and, at the same time, plays an important role in (a) 1 only (b) 2 only
directly helping the poor by ensuring that they get basic (c) Both 1 and 2 (d) Neither 1 nor 2
education and health services and receive adequate
nutrition ‘and food.
5. What is the essential message being conveyed
by the author of the passage?
1. According to the passage : (a) The objectives of inclusive growth laid
1. The objective of inclusive growth was laid down by the founding fathers of the na-
down by the founding fathers of the nation. tion should be remembered.
2. Need of the hour is to have an enabling (b) The Government needs to make available
Government. more schools and health services.
3. The Government should engage in maximum (c) The Government needs to establish mar-
interference in market processes. kets and industries to meet the needs of
4. There is a need to change the size of the the poor strata of the society.
Government. (d) There is a need to rethink the role of the
State in achieving inclusive growth.

66
COMPREHENSION

2. To be a creative society, it is imperative to


Passage-2 have potential contradictions and conflicts.
The concept of ‘creative society’ refers to a phase Which of the statements given above is/are
of development of a society in which a large number correct?
of potential contradictions become articulate and (a) 1 only (b) 2 Only
active. This is most evident when oppressed social (c) Both 1 and 2 (d) Neither 1 nor 2
groups get politically mobilised and demand their rights
The upsurge of the peasants and tribals, the Directions for the following 6 (six) items:
movements for regional autonomy and self- Read each of the following two Passages and answer
determination, the environmental movements, and the the items that follow. Your answers to these items
women’s movements in the developing countries are should be based on the passages only.
signs of emergence of creative society in contemporary
times. The forms of social movements and their Passage-3
intensity may vary from country to country and place Ecosystems provide people with a variety of goods
to place within a country. But the very presence of and services; food, clean water, clean air, flood control,
movements for social transformation in various spheres soil stabilization, pollination, climate regulation, spiritual
of a society indicates the emergence of a creative fulfillment and aesthetic enjoyment; to name just a few.
society in a country. Most of these benefits either are irreplaceable or the
6. What does the author imply by “creative technology necessary to replace them is prohibitively
society”? expensive. For example, potable fresh water can be
1 A society where diverse art forms and lit- provided by desalinating sea water, but only at great
erary writings seek incentive. cost.
2. A society where social inequalities are ac- The rapidly expanding human population has
cepted as the norm. greatly modified the Earth’s ecosystems to meet their
3. A society where a large number of con- increased requirements of some of the goods and
tradictions are recognised. services, particularly food, fresh water, timber, fibre
4. A society where the exploited and the op- and fuel. These modifications have contributed
pressed groups grow conscious of their substantially to human well being and economic
human rights and upliftment. development. The benefits have not been equally
Select the correct answer using the codes given distributed. Some people have actually been harmed
below: by these changes. Moreover, short-term increases in
(a) 1, 2 and 3 (b) 4 only some ecosystem goods and services have come at
(c) 3 and 4 (d) 2 and 4 the cost of the long-term degradation of others. For
example, efforts to increase the production of food
7. What according to the passage are the and fibre have decreased the ability of some
manifestations of social movements? ecosystems to provide clean water, regulate flooding
1. Aggressiveness and being incendiary. and support biodiversity.
2. Instigation by external forces.
3. Quest for social equality and individual
9. With reference to the passage, consider the
freedom.
following statements.
4. Urge for granting privileges and self-re-
Expanding human population has an adverse
spect to disparaged sections of the soci-
effect on :
ety.
1. Spiritual fulfilment
Select the correct answer using the codes given
2. Aesthetic enjoyment
below:
3. Potable fresh water
(a) 1 and 3 only (b) 2 and 4 only
4. Production of food and fibre
(c) 3 and 4 only (d) 1, 2, 3 and 4 5. Biodiversity
Which of the statements given above are
8. With reference to the passage, consider the correct?
following statements: (a) 1, 2 and 3 only (b) 2, 4 and 5 only
1. To be a creative society, it is essential to (c) 3 and 5 only (d) 1, 2, 3, 4 and 5
have a variety of social movements.

67
COMPREHENSION

10. The passage mentions that “some people have ately.


actually· been harmed by these changes”. What 3. Man must do his duty.
does it imply? 4. Man should be able to defy convention in
1. The rapid expansion of population has order to be moral.
adversely affected some people. Select the correct answer from the codes given
2. Sufficient efforts have not been made to below :
increase the production of food and fibre. (a) 1 only (b) 1and 3
3. In the short term some people may be (c) 2 and 3 (d) 1 and 4
harmed, but in .the long term everyone will
benefit from modifications in the Earth’s 13. Which of the following statements is the nearest
ecosystems. definition of moral action, according to the
Which of the statements given above is/are writer?
correct? (a) It is a mechanical action based on official
(a) 1 only orders from superiors.
(b) 2 only (b) It is an action based on our sense of dis-
(c) 1 and 3 cretion.
(d) None of the statements given above. (c) It is a clever action based on the clarity of
purpose.
11. With reference to the passage, consider the (d) It is a religious action based on under-
following statements : standing.
1. It is imperative to modify the Earth’s eco-
systems for the well being of mankind. 14. The passage contains a statement “lets himself
2. Technology can never replace all the be swept along like a log of wood by a current.”
goods and services provided by ecosys- Among the following statements, which is/are’
tems. nearest in meaning to this?
Which of the statements given above is/are 1. A person does not use his own reason.
correct? 2. He is susceptible to influence / pressure.
(a) 1 only (b) 2 only 3. He cannot withstand difficulties / chal-
(c) Both 1 and 2 (d) Neither 1 nor 2 lenges.
4. He is like a log of wood.
Passage-4 Select the correct answer using the codes given
below:
A moral act must be our own act; must spring
(a) 1 only (b) 1 and 2
from our own will. If we act mechanically, there is no
moral content in our act. Such action would be moral, (c) 2 and 3 (d) 1 and 4
if we think it proper to act like a machine and do so.
For, in doing so, we use our discrimination. We should Directions for the following 4 (four)items:
bear in mind the distinction between acting Read the following passage and answer the items that
mechanically and acting intentionally. It may be moral follow. Your answers to these items should be based
of a king to pardon a culprit. But the messenger on the passage only.
carrying out the order of pardon plays only a
mechanical part in the king’s moral act. But if the Passage - 5
messenger were to carry out the king’s order A country under foreign domination seeks escape
considering it to be his duty, his action would be a moral from the present in dreams of a vanished age, and
one. How can a man understand morality who does finds consolation in visions of past greatness. That is
not use his own intelligence and power of thought, but a foolish and dangerous pastime in which many of us
lets himself be’ swept along like a log of wood by a indulge. An equally questionable practice for us in India
current? Sometimes a man defies convention and acts is to imagine that we are still spiritually great though
on his own with a view to absolute good. we have come down in the world in other respects.
Spiritual or any other greatness cannot be founded on
12. Which of the following statements best describe/
lack of freedom and opportunity, or on starvation and
describes the thought of the writer?
misery. Many western writers have encouraged that
l. A moral act calls for using our discretion.
notion that Indians are other-worldly. I suppose the
2. Man should react to a situation immedi-
poor and unfortunate in every country become to some

68
COMPREHENSION

extent other-worldly, unless they become


revolutionaries, for this world is evidently not meant Directions for the following 4 (four items):
for them. So also subject peoples. Read the following passage and answer the items that
follow. Your answers to these items should be based
As a man grows to maturity he is not entirely
on the passage only.
engrossed in, or satisfied with, the external objective
Passage-6
world. He seeks also some inner meaning, some
psychological and physical satisfactions. So also with A species that exerts an influence out of proportion
peoples and civilizations as they mature and grow adult. to its abundance in an ecosystem is called a keystone
Every civilization and every people exhibit these parallel species. The keystone species may influence both the
streams of an external life and an internal life. Where species richness of communities and the flow of energy
they meet or keep close to each other, there is an and materials through ecosystems. The sea star
equilibrium and stability. When they diverge conflict Pisaster ochraceus, which lives in rocky intertidal
arises and the crises that torture the mind and spirit. ecosystems on the Pacific coast of North America, is
also an example of a keystone species. Its preferred
15. The passage mentions that “this world is prey is the mussel Mytilus californianus. In the
evidently not meant for them”. It refers to people absence of sea stars, these mussels crowd out other
who competitors in a broad belt of the ‘intertidal zone. By
1. seek freedom from foreign domination. consuming mussels, sea star creates bare spaces that
2. live in starvation and misery. are taken over by a variety of other species.
3. become revolutionaries.
Which of the statements given above is/are A study at the University of Washington
correct ? demonstrated the influence of Pisaster on species,
(a) 1 and 2 (b) 2 only richness by removing sea stars from selected parts of
the intertidal zone repeatedly over a period of five
(c) 2 and 3 (d) 3 only
years. Two major changes occurred in the areas from
which sea stars were removed. First, the lower edge
16. Consider the following assumptions : of the mussel bed extended farther down into the
1. A country under foreign domination can- intertidal zone, showing that sea stars are able to
not indulge in spiritual pursuit. eliminate mussels completely where they are covered
2. Poverty is an impediment in the spiritual with water most of the time. Second, and more
pursuit. dramatically, 28 species of animals and algae
3. Subject peoples may become other- disappeared from the sea star removal zone. Eventually
worldly. only Mytilus, the dominant competitor, occupied the
With reference to the passage, which of the entire substratum. Through its effect on competitive
above assumptions is/are valid? relationships, predation by Pisaster largely determines
(a) 1 and 2 (b) 2 only which species live in these rocky intertidal ecosystems.
(c) 2 and 3 (d) 3 only
19. What is the crux of the passage?
(a) Sea star has a preferred prey.
17. The passage thematically centres on
(b) A preferred prey determines the survival
(a) the state of mind of oppressed people
of a keystone species.
(b) starvation and misery
(c) Keystone species ensures species diver-
(c) the growth of civilization
sity.
(d) body, mind and spirit of people in general.
(d) Sea star is the only keystone species on
the Pacific coast of North America.
18. According to the passage, the torture of the mind
and spirit is caused
20. Reference to the, passage, consider the
(a) by the impact of foreign domination.
following statements:
(b) by the desire to escape from foreign domi-
1. Mussels are generally the dominant
nation and find consolation in visions of
species in intertidal ecosystems.
past greatness.
(c) due to lack of equilibrium between an ex- 2. The survival of sea stars is generally
ternal life and an internal life. determined by the abundance of mussels.
(d) due to one’s inability to be either revolu- Which of the statements given above is / are
tionary or other-worldly. correct ?

69
COMPREHENSION

(a) 1 only (b) 2 only them only a short childhood, if at all. The Right to
(c) Both 1 and 2 (d) Neither 1 nor 2 Education (RTE) has become law at a point in India’s
history when the ghastly practice of female infanticide
21. Which of the following is/are implied by the has resurfaced in the form of foeticide. This is
passage ? “symptomatic of a deeper turmoil” in society which is
1. Mussels are always hard competitors for compounding the traditional obstacles to the education
sea stars. of girls., Tenacious prejudice against the intellectual
2. Sea stars of the Pacific coast have potential of girls runs across our cultural diversity and
reached the climax of their evolution. the system of education has not been able to address
3. Sea stars constitute an important compo- it.
nent in the energy flow in intertidal eco-
system. 23. With reference to the passage, consider the
Which of the statements given above is/are following statements :
correct ? 1. When children are denied education, adult
(a) 1 and 2 (b) 2 only society does not act on behalf of them.
(c) 1 and 3 (d) 3 only 2. Right to Education as a law cannot be
enforced in the country.
22. Consider the following assumptions: Which of the statements given above is/are
1. The food chains / food web in an ecosys- correct ?
tem are influenced by keystone species. (a) 1 only (b) 2 only
2. The presence of keystone species is a (c) Both 1 and 2 (d) Neither 1 nor 2
specific characteristic of aquatic ecosys-
tems. 24. According to the passage, what could be the
3. If the keystone species is completely re- traditional obstacle to the education of girls ?
moved from an ecosystem, it will lead to 1. Inability of parents to fight a legal battle
the collapse of the ecosystem. when the Right to Education is denied to
With reference to the passage, which of the their children.
above assumptions is/are valid? 2. The traditional way of thinking about girls’
(a) 1 only (b) 2 and 3 only role in society.
(c) 1 and 3 only (d) 1, 2 and 3 3. The prejudice against the intelle- ctual
potential of girls.
Directions for the following 5 (five) terms : 4. Improper system of education.
Read the following passage and answer the items that Select the correct answer from the codes given
follow your answers to these items should be based below:
on the passage only. (a) 1 and 2 only (b) 2, 3 and 4 only
Passage-7 (c) 1, 3 and 4 only (d) 1, 2, 3 and 4
Now India’s children have a right to receive at
least eight years of education, the gnawing question is 25. On the basis of the passage, consider the
whether it will remain on paper or become a reality. following statements :
One hardly needs a reminder that this right is different 1. Right to Education is a legal right and not
from the others enshrined in the Constitution, that the a fundamental right.
beneficiary - a six year old child cannot demand it, 2. For realising the goal of universal educa-
nor can she or he fight a legal battle when the right is tion, the education system in the country
denied or violated. In all cases, it is the adult society must be made identical to that of devel-
which must act on behalf of the child. In another oped countries.
peculiarity, where a child’s right to education is denied, Which of the statements given above is/are
no compensation offered later can be adequate or correct ?
relevant. This is so because childhood does not last. If (a) 1 only (b) 2 only
a legal battle fought on behalf of a child is eventually (c) Both 1 and 2 (d) Neither 1 nor 2
won, it may be of little use to the boy or girl because
the opportunity missed at school during childhood
26. Which one of the following statements conveys
cannot serve the same purpose later in life. This may
the key message of the passage ?
be painfully true for girls because our society permits

70
COMPREHENSION

(a) India has declared that education is com-


pulsory for its children.
(b) Adult society is not keen on implementing
the Right to Education.
(c) The Right to Education, particularly of a
girl child, needs to be safeguard.
(d) The system of education should address
the issue of Right to Education.

27. Which one of the following statements conveys


the inference of the passage?
(a) The society has a tenacious prejudice
against. the intellectual potential of girls.
(b) Adults’ cannot be relied upon to fight on
behalf of children for –their Right to Edu-
cation.
(c) The legal fight to get education for chil-
dren is often protracted and prohibitive.
(d) There is no sufficient substitute for edu-
cation received in childhood.

71
COMPREHENSION

ANSWERS OF COMPREHENSION

2020 lead in GM technology and prioritise the


technology agenda.
1. (b) 1, 3 and 5 2. (d) Neither 1 nor 2
2. (a) 1 and 2 3. (d) Foreign plants should be used to increase the
3. (c) 3 only biodiversity of a country.
4. (c) 1 and 3 only
4. (b) Urban local bodies are not efficient enough
to meet the water requirements of our cit- 5. (d) in favour of participation of peoples’ groups
ies. in city planning.
6. (a) 1 and 2 only
5. (b) 2 only
7. (b) For meaningful financial inclusion, India’s
6. (b) 2 and 4 banking system needs more number of
7. (d) Inclusive agricultural growth is key to re- banking correspondents and other such last-
duce poverty in the near future. mile workers.
8. (d) Increased social sector spending is impera- 8. (d) As we go digital, we must recognise the huge
tive for large-scale job creation. threat to Internet security from some IoT
devices.
9. (a) 1 and 2
9. (b) India should be careful to protect its digital
10. (c) Wetlands need to be reinforced as more sovereignty in global trade talks.
than just open sources of water. 10. (a) Big Data is the key resource in the digital
11. (a) Distribute your wealth across different space.
kinds of assets so that your risks would 11. (a) Supporting small farmers is an important part
be minimized. of any agenda regarding environmentally
12. (b) 1 and 3 only sustainable development.
12. (c) Both 1 and 2
13. (c) Biotechnology’s role is not confined to the
current priorities of developing GM crops. 13. (c) 1 and 3 only
14. (b) 2 only
14. (c) India lacks the institutional mechanism to 15. (d) India will be free from diseases like TB only
deal with the failure of banks.
when its poverty alleviation programmes are
15. (c) Getting renewable resources to market effectively and successfully implemented.
too soon may be costly. 16. (b) In a competitive global economy, India must
16. (a) 1 only use regulations strategically.
17. (c) Both 1 and 2 17. (d) Neither 1 nor 2
18. (c) Babies of malnourished mothers should be fed
18. (a) In India, the drainage networks are not
with dairy milk fortified with sialylated
separate for sewerage and storm water.
oligosaccharides instead of mother’s milk.
19. (b) Religious freedom under the Constitution 19. (b) 2 only
is open to State intervention. 20. (b) 3 only
20. (d) Relying on a homogeneous food source is 21. (b) 3 only
not desirable. 22. (b) 2 and 4
21. (d) 3 and 5 only 23. (d) It is difficult, if not impossible, to repair every
manifestation of historical injustice.
22. (d) 1 and 4 only
24. (d) Neither 1 nor 2
23. (b) 2 only 25. (b) 2 only
24. (c) Both 1 and 2 26. (b) 3 only
25. (d) Neither 1 nor 2 27. (b) 3 only
28. (a) Farmers who modernize their methods and
diversify their fields will be in an advantageous
2019 position in future.
1. (a) Public research institutions should take the 29. (d) Man and the aforementioned animals have a

72
COMPREHENSION

shared evolutionary history. oriented


30. (c) 1 and 3 3. (d) Climate change poses a threat to the
survival of polar bears.
4. (b) People have to perceive toilet use and pit-
2018 emptying as clean and not polluting.
1. (d) Environmental cost of meat production is 5. (b) The growth driven by GDP only is neither
unsustainable when it is produced through desirable nor sustainable.
industrial farming. 6. (b) Labour reforms are required in India to
2. (b) Safe wildlife corridors between protected make optimum use of its vast labour force
areas is an essential aspect of conservation productively.
efforts. 7. (c) working is a tyranny which we have to face
3. (d) Neither 1 nor 2 8. (c) make it easier for us to live
4. (b) 3 only 9. (b) equal privileges, fulfilment of destiny
5. (d) Access to cooking gas can reduce and political tolerance
premature deaths in poor households. 10. (a) the sum total of the private wills of the
6. (a) A happy world is a dream of science. individuals
7. (b) Melting of summer ice in the Arctic leads 11. (a) In a democracy, force is the main
to changes in the geopolitics. phenomenon in the actual exercise of
8. (c) For India, food security collides with sovereignty.
trade. 12. (d) right to participate in politics
9. (b) 2 only 13. (d) despair in a country sometimes leads to
10. (d) There is no perfect diet or one solution dictatorship
for obesity. 14. (a) 1 only
11. (d) Asia and North America will be worst 15. (d) Neither 1 nor 2
sufferers from climate change and the 16. (a) Political development is not a unilinear
consequent shortage of food. process for it involves both growth and
12. (c) 3 only decay.
13. (a) 1 only 17. (c) Devolution, to be successful, requires a
14. (d) Inculcating learning through activities and democracy in which there is free expression
discovery of the will of the people at lower level and
15. (c) The Right to Free and Compulsory their active participation at the grassroots
Education should include quality level.
education for all. 18. (b) Speaking of digital technologies is speaking
16. (b) Indian economy greatly depends on its of our life and living.
agriculture. 19. (b) Falling into middle-income trap is a general
17. (a) With more options for food come more characteristic of fast growing economies.
risks. 20. (c) inclusive innovation and R&D can help
18. (b) Science and faith can be mutually create an egalitarian society.
complementary if their proper domains 21. (d) Relation between climate change and
are understood. migration is not yet properly understood.
19. (d) they obstruct her physical and spiritual 22. (b) 2 only
growth 23. (a) 1 only
20. (b) 2, 3 and 4 only 24. (c) Air quality index is not helpful to the
21. (a) 1 only residents of many of our large cities.
22. (d) Neither 1 nor 2 25. (a) We must create conditions for the faster
23. (c) 3 and 4 only growth of highly productive service jobs
24. (b) 2 only to ensure employment growth and
25. (c) Reducing our consumerism is very much inclusion.
in our own interest. 26. (a) Agricultural practices that enhance
26. (b) leadership can be acquired as well as biodiversity can often increase farm output
taught and reduce the vulnerability to disasters.
2017 27. (a) India’s GDP displays high value-added
and high productivity levels in medium tech
1. (c) Environmental issues contribute to and resource processing industries.
resource stresses and political conflict. 28. (a) Increasing the efficiency of farm-to-fork
2. (d) we should be resolute and achievement- value chain is necessary to reduce the

73
COMPREHENSION

poverty and malnutri-tion. 25. (d) Seeking accountability through enhanced


29. (b) strong Centre is a binding force for national participation by people in decision making
integrity. process
30. (b) rule according to the consent of the 26. (b) 2 and 3 only
governed 27. (d) Religious concept of rights is primarily derived
from our relationship to god.
2016
1. (d) 1, 2 and 3 2015
2. (b) 3 only 1. (d) Economic reforms can often create a high in-
3. (b) 3 only flation economy.
4. (a) 1 only
2. (d) The comprehensive idea of economic justice
5. (d) Green revolution can pose a threat to demands that the Right of each person to ac-
biodiversity in food supply and food security in quisition of property has to be reconciled with
the long run that of others.
6. (c) Greater transparency in the functioning of the
3. (a) The conflict between the claims of State and
government and promoting the financial
individual remains unresolved.
inclusion are needed at present
7. (d) Climate adaptation should be examined in 4. (a) 1 only
tandem with other economic development 5. (c) 3 only
options 6. (a) 1 only
8. (a) Biodiversity sustains the ability of nature to 7. (a) Conflict between developed and developing
recycle water countries regarding support for mitigation
9. (d) Need to promote financial inclusion 8. (b) Poverty and climate impacts reinforce each
10. (b) 2 only other and therefore we have to re-imagine our
11. (d) Diverse natural wants forced man towards food systems.
social system 9.(d) Emerging economies are at a risk of shock
12. (c) the predominance of Economics over Politics from advanced economies.
13. (b) It is imperative for us to modify our land use 10. (c) Open defecation reduces the human capital
practices in order to mitigate climate change of India’s workforce.
11 (b) True democracy could be established by
14. (a) Our increasing demand for foods sourced from
providing equal opportunities to all.
animals puts a greater burden on our natural
12. (b) 2 only
resources
13. (d) Promote financial inclusion
15. (b) A permanent change in the food habits of a 14. (b) The supremacy of rules and safeguards should
community can bring about a genetic change prevail as opposed to the influence of exclusive
in its members discretion of authority.
16. (b) the existence of a non-commercialized sector 15.(a) Man seeks control over external things
in different underdeveloped countries renders affecting him.
the national income comparisons over countries 16.(d) Food security is increasingly a collective
difficult challenge.
17. (d) Increasing green cover could trigger the release 17. (b) The greater spending on weapon-building by
of carbon trapped in soil us would lessen the possibility of armed conflict
18. (b) 2 and 3 only with our neighbours.
19. (d) to achieve a balance between demand for and 18. (c) Universalization and integration of maternal
supply of food health and child health services can effectively
20. (c) 1 and 3 only address the problem.
19. (c) Both 1 and 2
21. (b) 3 only
20. (a) He has no intention to waste his time.
22. (c) 1 and 4 only
21. (a) he hardly utters or writes a thoughtless word.
23. (b) 2 only
22. (d) tendency to overstate.
24. (d) There is a need and scope for reforms to 23. (a) The richer States must lead in the production
improve the accountability of civil services and adoption of renewable energy.

74
COMPREHENSION

24. (d) Influence of patriarchal mindset dustrial sectors.


25. (b) The primary concern of government is to 26 (d) None of the above statements is correct in this
provide absolute social security to all its context.
citizens. 2013
26. (b) Our cities can provide better quality of life if
1(b) The language of imposition of democracy is
our local government bodies have adequate
inappropriate. There is, however, a need to
staff with required skills and competencies.
consider this concept in the backdrop of culture
27. (c) Social behaviour in some species of birds
of ‘own ways’ of non-Western society.
increases the odds of survival in an unsafe
2 (d) Neither 1 nor 2
world.
3 (a) 1 only
28.(d) It is essential to provide financial literacy to
4 (a) 1 only
the rural poor.
5 (c) if the food needs of younger children are
29. (c) 2 only
known.
30. (b) 2 only
6 (d) Neither 1 nor 2
31. (c) Mangroves have a crucial role in some of the
7 (c) the methods adopted by farmers and existing
coastal food chains.
risk sharing institutions are not adequate.
32. (d) Liberty is the maintenance of conditions for 8 (b) No single risk-specific policy is sufficient to
the growth of human personality. reduce agricultural risk.
2014 9 (d) Members of a primary group know each other
intimately.
1 (c) emphasizing inclusive growth where gains of
10 (b) l, 2 and 3
growth are shared widely by the population.
11 (c) growing global linkages and integration of the
2 (c) develop economic growth and redistribute
Indian financial markets.
economic gains to those getting left behind.
12 (d) dominated by banks.
3 (d) Neither 1 nor 2
13 (a) l, 2 and 3
4 (b) 2 and 3
14 (b) become rock and prevent oxygen from enter-
5 (a) l and 2
ing them.
6 (a) 1, 2 and 3
15 (c) weight of overlying sediment layers causes the
7 (c) To facilitate wider adaptability of crops
production of heat.
8 (c) 1 and 3 only
16 (d) Disruption of habitat
9 (a) 1 only
17 (a) 1 only
10 (d) 1 and 4 only
18 (c) 1 and 3 only
11 (b) Mitigation of the greenhouse gases is impera-
19 (b) Discharged agricultural slurry is pollutant and
tive
decomposed slurry in soil is fertilizer.
12 (a) 1 only
20 (b) 1, 3 and 4 only
13 (d) The Himalayan biogeographic region should be
21 (b) 1 and 2 only
enabled to adapt to climate change smoothly.
22 (c) Improper waste disposal from agriculur can
14 (b) Conservation efforts should emphasize on bio-
destroy the aquatic ecosystems.
geographic ranges rather than on some spe-
23 (b) The refusal on the part of man to change his
cies or habitats.
character.
15 (c) 2 and 3
16 (a) distortions and inefficiency in the economy. 24 (a) 1 only
17 (c) give absolute freedom to markets to determine
prices, production and employment. 2012
18 (c) 1, 3 and 4 1 (a) Education is functional and utilitarian in its
19 (b) reducing role. purposes.
20 (a) market without control by government authori-
2 (c) it is an inner process of moral and intellectual
ties.
development.
21 (b) free market is not fair but competent.
22 (d) 4 Only 3 (c) individuals develop self-critical awareness and
23 (b) 2 only independence of thought.
24 (b) Private sector cannot have monetary gains in 4 (c) Some individuals in any given population show
this. resistance after the application of pesticides.
25 (c) Mix of government investment in different in-

75
COMPREHENSION

5(b) 3 only 11. (b) 2 only


6 (d) None of the statements (a), (b) and (c) given 12. (d) 1 and 4
above is correct. 13. (b) It is an action based on our sense of discretion.
7(b) l and 2 only 14. (b) 1 and 2
8 (c) 2 only 15. (b) 2 only
9 (b) Too much use of chemicals is not good for the 16. (a) 1 and 2
ecosystem. 17. (a) the state of mind of oppressed people
10 (a) 1 only 18. (c) due to lack of equilibrium between an external
11(d) None of the above implies low-carbon growth. life and an internal life.
12 (b) 2 and 3 only 19. (c) Keystone species ensures species diversity.
13 (b) 2 only 20. (a) 1 only
14 (d) 1, 2 and 3 21. (d) 3 only
15 (b) Adaptation and mitigation should be integrated 22. (a) 1 only
with development strategies 23. (d) Neither 1 nor 2
16 (d) None of the statements (a), (b) and (c) is 24. (b) 2, 3 and 4 only
correct in this context. 25. (d) Neither 1 nor 2
17 (b) 2 and 3 only 26. (d) The system of education should address the
18 (b) Presence of oceans and mountain ranges. issue of Right to Education.
19(a) 1 and 2 27. (d) There is no sufficient substitute for education
20 (c) Both 1 and 2 received in childhood.
21 (a) Democracy and development are distinct and
separate goals.
22 (d) Extent of real freedom that citizens enjoy.
23 (c) Political liberties and democratic rights are
essential elements of development.
24 (a) 1 only
25 (b) 2 and 3 only
26 (c) With competition law, it is easy to ensure a
level playing field between domestic and
foreign firms.
27 (b) 1, 2 and 3 only
28 (c) Women benefiting from group farming.
29 (b) 2 only
30 (c) Democratization as greater involvement of
‘atomized’ individuals in the public sphere.
31 (b) irrelevance of community identity as a
governing force in Indian politics.
32 (c) Social change through transfer of power from
upper caste elites to subaltern groups.

2011
1 (a) 1 and 2 only
2. (d) Delivery of the basic services to the deprived
sections of the society.
3. (c) 3, 4 and 5 only
4. (d) Neither 1 nor 2
5. (d) There is a need to rethink the role of the State
in achieving inclusive growth.
6. (c) 3 and 4
7. (c) 3 and 4 only
8. (a) 1 only
9. (c) 3 and 5 only
10. (a) 1 only

76

You might also like